Cho n là số nguyên dương và thỏa mãn 4 4 1 3 aann giá trị của n là

No 14 SỐ Không phải thực tế mà chính là phép toán là thứ sinh ra các con số và cho chúng tính chất! VÀ LỊCH SỬ PHÁT TRIỂN LOÀI NGƯỜI Nguyễn Lê Anh Xin nhấn mạnh: tôi xem thống kê học là một cách suy nghĩ BẢY TRỤ CỘT THÔNG THÁI CỦA THỐNG KÊ HỌC Nguyễn Văn Tuấn 13. 1. 018 VÀ CÁC CHUYÊN MỤC KHÁC MỘT SỐ BÀI TOÁN TRÊN TÂM ĐƯỜNG TRÒN EULER Trần Quang Hùng TOÁN HỌC VÀ ẢO THUẬT Nguyễn Hùng Sơn NHỮNG CƠ HỘI MỞ RA SAU NGÀY HỘI TOÁN HỌC MỞ Phạm Hy Hưng

NO 14 CHỦ BIÊN: TRẦN NAM DŨNG BIÊN TẬP VIÊN: Dec 018 LÊ VIẾT ÂN LÊ PHÚC LỮ NGUYỄN TẤT THU VÕ QUỐC BÁ CẨN TRẦN QUANG HÙNG NGUYỄN VĂN HUYỆN ĐẶNG NGUYỄN ĐỨC TIẾN

Bạn có tin vào chuyện cổ tích? LỜI NGỎ Cách đây 4 năm, chúng tôi, cùng với những độc giả đầu tiên đã bắt đầu câu chuyện cổ tích của mình bằng việc cho ra đời số đầu tiên tạp chí Epsilon, một tạp chí với mục tiêu vô tư, phát hành online miễn phí, và phục vụ những người yêu thích toán học. Một tạp chí được xây dựng bởi cộng đồng và cho cộng đồng. Câu chuyện cổ tích thử nghiệm ấy đã được lan truyền đi, được đông đảo bạn đọc đón nhận, chia sẻ, và cùng chung sức xây dựng. Câu chuyện của chúng tôi cũng đã có một kết thúc rất đẹp: chúng tôi dừng lại ở con số 13, khi Epsilon đang ở độ chín. Chúng tôi đã dừng lại trong tự hào, để từ đó mở đường cho một người anh em khác, tạp chí Pi. Chúng tôi dừng lại, nhưng một phần nào đó trong mỗi cá nhân chúng tôi vẫn luyến tiếc về những ngày tháng đầy thơ mộng của Epsilon. Rồi thời gian dần trôi, Pi ngày càng vững mạnh, giờ đây đã là một tờ báo quen thuộc với những người yêu toán. Dẫu vậy, trong lòng các bạn yêu toán vẫn nhớ về Epsilon và họ luôn hỏi chúng tôi liệu Epsilon có ngày trở lại? Vào một ngày đẹp trời mùa hè 018, được thôi thúc bởi những giấc mơ đẹp, tổng biên tập Trần Nam Dũng quyết định để người em Epsilon tái xuất, cùng đồng hành với anh Pi của mình. Vậy là tất cả chúng tôi tái hợp. Ban biên tập Epsilon những ngày đầu tiên phần lớn vốn là những cậu trai trẻ, độc thân và có rất nhiều thời gian, giờ đây đều đã có gia đình riêng, đã biết thế nào là những vất vả mưu sinh của cuộc sống. Nhuệ khí của tuổi trẻ đang bắt đầu chuyển mình thành kinh nghiệm của những người đàn ông từng trải. Mặc dù vậy, khi nghe lời hiệu triệu của tổng biên tập, tất cả chúng tôi đều không chút đắn đo, cùng trở lại. Chúng tôi lại cố gắng liên lạc với từng đồng nghiệp để có được những bài viết tốt nhất, lại làm việc thâu đêm để có được những bản biên tập đúng hạn và đúng chất lượng. Tất cả có được bởi chúng tôi tin vào nhau, và tin vào những điều kỳ diệu của những câu chuyện cổ tích. Epsilon trở lại và vẫn giữ vững triết lý của mình: phục vụ cho cộng đồng yêu thích toán, trong đó đặc biệt đề cao tính "khả đọc" của từng bài viết. Epsilon sẽ luôn luôn miễn phí, luôn luôn phát hành online và tương lai chúng tôi sẽ từ từ chuyển hoàn toàn sang 100% online: hiện nay chúng tôi vẫn xuất bản theo kiểu tập tin định dạng pdf, trong tương lai chúng tôi sẽ sử dụng sang những định dạng khác, giúp cho tính khả đọc không chỉ đúng với nội dung mà còn với hình thức trình bày. Epsilon sẽ cố gắng mở rộng hơn đối tượng tác giả, sẽ sử dụng hệ thống bình duyệt bởi những người có chuyên môn. Chúng tôi sẽ tập trung cao hơn về chất lượng, do vậy, chúng tôi sẽ chỉ duy trì xuất bản số mỗi năm, vào ngày 13 tháng 6 và 13 tháng 1. Epsilon 14 vẫn giữ bản sắc của mình các với chuyên mục quen thuộc như bài toán hay - lời giải đẹp, điểm sách, toán học giải trí, lời giải và bình luận các cuộc thi,... và tất nhiên, không thể thiếu những câu chuyện toán học đẹp như cổ tích. Và bây giờ, mời các bạn cùng đọc Epsilon 14.

MỤC LỤC Nguyễn Lê Anh Số và lịch sử phát triển loài người... 6 Vaselin Dimitrov Định lý không điểm tổ hợp - Combinatorial Nullstenllensatz... 16 Nguyễn Hùng Sơn Toán học và Ảo thuật... 4 Ban Biên tập Epsilon Thế nào là tư duy Logic - Hành trình tìm kiếm máy bay mất tích MiG-1U... 9 Võ Nhật Vinh Giới thiệu bài toán tối ưu hai lớp... 49 Nguyễn Song Minh Tính chất Phi Archimedean của Định giá P-Adic... 55 Võ Quốc Bá Cẩn Phương pháp thêm biến trong giải phương trình hàm... 66 Ngô Văn Thái Sáng tạo - Làm chặt... 79 Nguyễn Văn Tuấn Bảy trụ cột thông thái của thống kê học... 93 Trần Quang Hùng Một số bài toán trên tâm đường tròn Euler... 98 Nguyễn Trường Sơn Điểm Humpty-Dumpty trong tam giác và ứng dụng... 109 Lê Viết Ân Một số bổ đề hữu dụng tiếp cận lời giải trong các bài toán hình học... 134 Nguyễn Duy Liên Bài toán hay - Lời giải đẹp... 164 Lê Phúc Lữ Hướng tới kỳ thi VMO 018-019... 168 Nguyễn Trần Hữu Thịnh, Phạm Quốc Thắng, Nguyễn Trường Hải, Võ Thành Đạt, Trần Bá Đạt 4

Trường Đông Toán học miền Nam năm 018 và những bài toán hay... 14 Phạm Hy Hưng Những cơ hội mở ra sau ngày hội toán học mở... 33 Trần Nam Dũng Epsilon, MOD, BME và tinh thần tình nguyện... 39 5

SỐ VÀ LỊCH SỬ PHÁT TRIỂN LOÀI NGƯỜI Nguyễn Lê Anh Tôi thường bắt đầu bài giảng đầu tiên cho sinh viên năm thứ nhất bằng việc thông báo tên của tôi và một số quy định. Những quy định đầu tiên là sinh viên không cần phải xin phép để được vào ra khỏi lớp. Và điều này thì có hàm ý các sinh viên không nên chỉ vì phải đến đúng giờ mà phân tâm khi đi đường và vô tình để bị xảy ra tai nạn giao thông. Tôi còn nói nhiều điều nữa, rồi tôi hỏi - Có đúng là các em đã thi vào trường đại học một cách trung thực không? Gần như ngay lập tức các em đều khẳng định - Thưa thầy, đúng ạ. Tôi cũng nhận ra vẻ kiêu hãnh tự hào trên khuôn mặt của các bạn trẻ mới bước những bước đầu tiên vào giảng đường Đại học. Tôi im lặng và nói. - Các em có cho phép tôi được kiểm tra không? - Vâng. - Có ai trong số các em cho tôi biết 3 là gì không? Thế rồi cả lớp ồn ào, rồi chìm vào sự tĩnh lặng. Tôi gọi một vài sinh viên để họ đứng lên trả lời. Các vị đang dõi theo đọc bài viết này chắc cũng đang tự tìm lấy câu trả lời. Thôi thì đủ các kiểu trả lời. Tôi chờ cho tới khi sinh viên tự nhận thấy là tất cả các câu trả lời của các bạn ấy đều không đúng và tự hiểu ra là không có câu trả lời, rồi tôi nói: - Nếu chắc chắn các em đã thi vào đại học một cách trung thực thì hoặc là bộ Giáo Dục đã không trung thực khi đứng ra làm người cầm cân nảy mực - họ đã không có đủ kiến thức để làm việc ấy - hoặc là trường đại học mà các em đang học không trung thực khi tổ chức thi. Chúng ta sẽ cùng với các em làm cho rõ hơn chuyện này. Thế rồi tôi bắt đầu giảng bài. Tôi nói rằng các con vật không phải là không biết đến các con số, có điều chúng gắn liền con số với sự vật. Khi đưa cho một đứa trẻ nhỏ 5 cái kẹo thì chúng sẽ hình dung ra 5 cái trước mắt chúng. Khi ăn cái kẹo nào là cái kẹo ấy sẽ biến mất đi khỏi sự tưởng tượng. Vậy để kiểm tra một đứa trẻ có khả năng toán học không thì chỉ cần đưa cho nó kẹo, bảo nó cho vào túi, rồi bảo nó ăn, rồi bảo nó cho bạn nó. Rồi bất ngờ hỏi số lượng kẹo còn lại trong túi khi số lượng ấy chỉ vài ba cái. Nó sẽ trả lời đúng cho dù đó là phép toán với các số khá lớn. Những con số như vậy được gọi là số tự nhiên. Nó được dùng để thông báo số lượng các vật thể là kết quả của quá trình đếm. Quá trình đếm bắt đầu là 1, rồi đến,... không thể có 6

số đếm mà không có gì. Phải mãi tới 0 nghìn năm trước CN, do nhu cầu mà, xuất hiện số 0. Số 0 là số không có gì, tức không phải là số. Đó là sản phẩm nhân tạo phi tự nhiên đầu tiên được loài người tạo ra. Sự xuất hiện số 0 đồng nhất với thời điểm xuất hiện văn hóa. Trong việc sử dụng số đếm thì 1 con bò là hợp lý, nhưng 1= con bò là vô nghĩa. Việc sử dụng các con số như 1= cốc nước, =3 đoạn đường... được hiểu là khái niệm số đã thay đổi. Các con số đã bị bỏ đi đặc tính nguyên khi đếm và chuyển sang loại con số thông báo lượng. Người ta hình dung các con số như tỷ lệ độ dài của các đoạn thẳng với đoạn được coi là 1 - đoạn đơn vị. Để thông báo một con số nào đó người ta thông báo ra phép dựng hình để tạo ra nó. Và thế là hình học Ơ-cơ-lit ra đời. Theo Ơ-cơ-lit thì số được đồng nhất với đoạn thẳng có thể dựng ra được và chỉ những đoạn thẳng nào dựng ra được mới được coi là con số. Như vậy đường chéo của hình vuông có cạnh bằng 1 là số p [căn bậc hai của ]. Nếu giải thích p 3 là số mà bình phương lên thì bằng 3 là vô nghĩa, bởi có biết nó là số đâu mà bình phương lên!! Câu giải thích số p 3 được là đoạn thẳng sinh ra từ hai điểm cắt nhau của hai vòng tròn bán kính bằng nhau, vòng tròn này đi qua tâm của vòng vòng kia thì đúng. A p 3 1 B Và tất nhiên từ định lý Pitago chúng ta có thể chứng minh được bình phương của nó chính là bằng 3: Câu hỏi Có thể dựng ra được đoạn thẳng có độ dài bằng với là diện tích của hình tròn bán kính đơn vị hay không? là một bài toán rất khó và phải mãi tới thế kỷ 19 mới có câu trả lời. Câu trả lời là Không thể dựng ra được đoạn thẳng có độ dài bằng với diện tích hình tròn bán kính đơn vị. Điều này đồng nghĩa với việc số diện tích hình tròn bán kính đơn vị không phải là một con số theo quan niệm của Ơ-cơ-lít. Bằng phép dựng hình chúng ta có thể chia đoạn thẳng đơn vị ra làm 10 phần. Phần thứ nhất được gọi là 0, phần tiếp theo được gọi là 1,.. cứ như thế cho tới 9. Mỗi phần lại có thể chia ra 10 phần và cách gọi tên của đoạn nhỏ là tên của nó kèm theo tên của đoạn mà nó nằm bên trong. Đó là cách gọi theo hệ thập phân mà ngày nay mà chúng ta vẫn dùng để gọi tên các con số. Nếu chúng ta không chia ra làm 10 phần mà chỉ chia thành phần thì chúng ta có cách gọi nhị phân của các con số. Như thế số thập phân 0:99999 : : : tuần hoàn mãi mãi số 9, là ký hiệu đoạn cuối cùng của quá trình chia 10 phần mãi mãi. Nếu coi quá trình trên là việc ăn 1 cái bánh, cứ mỗi lần ăn hết 9=10 chỉ còn lại 1=10, rồi lại ăn hết 9=10 phần còn lại ấy... thì hầu như ai cũng cho rằng sẽ chẳng bao giờ ăn hết được cái bánh - vì Lúc nào cũng còn!. Tuy nhiên nếu coi đấy là một đoạn 7

đường đi từ A đến B, và khi đã đi đến được B rồi thì tất nhiên phải có lúc đi qua vị trí là điểm 9=10 đoạn đường, rồi tiếp theo là đi qua vị trí là điểm 9=10 đoạn còn lại,... cứ như thế. Sự tự mâu thuẫn này không phải là lý do chúng ta không thể đi hết đoạn đường từ A đến B, mà là do chúng ta ép buộc, lý giải quá trình đi này bằng một thứ ngôn ngữ phi thực tế - ngôn ngữ toán học. Điều này cũng sẽ xảy ra khi chúng ta nghiên cứu về thế giới. Chúng ta sẽ làm quen với khái niệm một vật vừa là hạt lại vừa là sóng... và còn nhiều điều nghe như phi lý nữa. Những người cho rằng quá trình đi từ A tới B sẽ đi đến được B thì sẽ công nhận dãy 0:9I 0:99I 0:999I :::có giới hạn và bằng 1; họ là môn đệ của lý thuyết toán học có giới hạn. Dãy có giới hạn kiểu như dãy 0:9I 0:99I 0:999I :::như ở trên, hoặc là dãy sinh ra khi Iasin đuổi con Rùa. Iasin là người vô địch về chạy ở thế vận hội cổ đại. Tài năng chạy của ông ta bị mang ra ví với tài năng chạy của một con Rùa. Khi Iasin đuổi tới chỗ con Rùa hiện tại thì con Rùa đã lại đi được một tí, cứ như thế Iasin lại phải đuổi tới chỗ con Rùa đang ở, rồi cứ như thế mà tạo ra dãy vô hạn có tính chất có giới hạn. Vậy những người theo trường phái Acsimet thì cho rằng Iasin sẽ đuổi được kịp con Rùa, và những người phản bác Acsimet thì không. Nói về Acsimet thường mọi người hay nhớ về câu chuyện ông ta cở truồng chạy ra phố reo Ơrêca khi tìm được phương pháp đo thể tích của chiếc ngai vàng mà không phải nấu chảy nó ra. Mọi người ít biết về việc Acsimet là người đầu tiên đưa ra quan niệm về cách đo diện tích của hình tròn bán kính đơn vị. Ông ta dựng ra các hình đa giác đều nội và ngoại tiếp hình tròn, và nhờ việc có thể tính được diện tích của các đa giác nội ngoại tiếp mà Acsimet có được dãy các số để đánh giá cận trên và cận dưới cho diện tích hình tròn đơn vị. Trên thực tế Acsimet đã sử dụng phép dựng hình để dựng ra được hai đoạn thẳng 3 10 10 và 3, và chứng minh được 71 70 3 10 71 0. Một cách tương đương, ta có f [x + y]= f [x]+ f [y], x > y > 0. Từ đây, với mọi x, y > 0 và z > x + y, ta đều có f [z + x + y]= f [z]+ f [x + y], và Do đó f [z + x + y]= f [ [z + x]+y ] = f [z + x]+ f [y]= f [z]+ f [x]+ f [y]. f [x + y]= f [x]+ f [y], x, y > 0. Như thế, f là hàm cộng tính từ R + vào R +. Suy ra f [x]=kx [k là hằng số dương nào đó] với mọi x > 0. Thay trở lại phương trình đã cho, ta được k [ x + ky]=k[x + y]+ky, x, y > 0, hay k[k ]y = 0, y > 0. Do đó k =. Vậy có duy nhất một hàm số thỏa mãn yêu cầu là f [x]=x. Bài toán 3 [IMO Shortlist, 011]. Tìm tất cả các cặp hàm số f, g : R R thỏa mãn g [ f [x + y] ] = f [x]+[x + y]g[y], x, y R. Lời giải. Đặt f [0]=b và g[0]=a. Từ phương trình hàm đã cho, dễ thấy g [ f [x] ] = f [x]+ax, x R 67

và Từ đó suy ra g [ f [y] ] = b + yg[y], y R. f [x]=xg[x]+b ax, x R. Bây giờ, thay x bởi x + z vào phương trình hàm đã cho, ta được g [ f [x + y + z] ] = f [x + z]+[x + z + y]g[y] =[x + z]g[x + z]+b a[x + z]+[x + z + y]g[y]. Thay y bởi y + z vào phương trình hàm đã cho, ta cũng có Đối chiếu hai kết quả trên, ta được g [ f [x + y + z] ] = f [x]+[x + y + z]g[y + z] = xg[x]+b ax +[x + y + z]g[y + z]. xg[x]+b ax +[x + y + z]g[y + z]=[x + z]g[x + z]+b a[x + z]+[x + z + y]g[y]. Trong phương trình này, cho x = y và rút gọn thành xg[x + z]=[x + z]g[x] az, x, z R. Trong phương trình này, thay x = 1 và z = x 1, ta được g[x]=xg[1] a[x 1]=kx+ a, x R, trong đó k = g[1] a. Suy ra f [x]=xg[x]+b ax = kx ax + b với mọi x R. Thay trở lại phương trình đã cho, ta được k [ k[x + y] a[x + y]+b ] + a = kx ax + b +[x + y][ky + b], x, y R. So sánh hệ số của x ở hai vế, ta được k = k. Suy ra k = 0 hoặc k = 1. Với k = 0, ta có a = ax + b +[x + y]b với mọi x, y R. So sánh hệ số của y ở hai vế, ta được b = 0. Từ đó, so sánh hệ số của x ở hai vế, ta cũng có a = 0. Vậy trong trường hợp này, ta có f [x] 0 và g[x] 0. Thử lại thỏa mãn. Với k = 1, ta có [x + y] a[x + y]+b + a = x ax + b +[x + y][y + b], x, y R, hay ay + a = xb + yb, x, y R. So sánh hệ số của x ở hai vế, ta được b = 0. Từ đó, so sánh hệ số của y ở hai vế, ta được a = 0. Vậy trong trường hợp này, ta có f [x]=x và g[x]=x. Thử lại thỏa mãn. Tóm lại, có hai cặp hàm số [ f [x], g[x] ] thỏa mãn yêu cầu đề bài là [0, 0] và [x, x]. 68

Bài toán 4. Tìm tất cả các hàm số f : R + R + thỏa mãn [ ] x f = f [ xf[x] ] f [ xf[y] ], x > y > 0. x y Lời giải. Nếu có a > b > 0 sao cho f [a]= f [b] thì ta có [ ] a f = f [ af[a] ] f [ af[b] ] = 0, a b mâu thuẫn. Do đó f đơn ánh. Từ giả thiết, ta suy ra [ ] x f + f [ xf[y] ] [ ] x = f + f [ xf[z] ] = f [ xf[x] ] x y x z với mọi x > max{y, z} > 0. Không mất tính tổng quát, ta chỉ cần xét y z > 0. Trong phương trình trên, chọn x = z + 1 f [y] thì ta có xf[y]= x z x. Suy ra [ ] x f = f [ xf[z] ]. x y Do f đơn ánh nên ta có x y x 1 = xf[z], hay x = y + f [z]. Từ đó suy ra z + 1 f [y] = y + 1, z y > 0. f [z] Như thế, ta có f [x]= x+c 1 [c là hằng số thực nào đó] với mọi x > 0. Vì f [x] > 0 với mọi x > 0 nên c 0. Thay trở lại phương trình hàm ban đầu, ta được Cho x = y + 1, ta được x y x + c[x y] = 1 y + 1 + c = x + c x + c[x + c] y + 1 + c y + 1 + c[y + 1 + c] Trong phương trình trên, cho y 0 +, ta được 1 1 + c = 1 + c 1 + c + c c 1 + c. y + c, x > y > 0. x + c[y + c] y + c, y > 0. y + 1 + c[y + c] Giải phương trình này, ta được c = 0. Từ đó suy ra f [x]= 1 x với mọi x > 0. Thử lại ta thấy thỏa mãn. Vậy có duy nhất một hàm số thỏa mãn yêu cầu là f [x]= 1 x. Bài toán 5 [IMC, 1999]. Tìm tất cả các hàm số f : R + R + thỏa mãn f [x + y]= f [x] f [ yf[x] ], x, y > 0. 69

Lời giải. Thay y bởi y + z vào phương trình đã cho, ta được f [x + y + z]= f [x] f [ [y + z] f [x] ], x, y, z > 0. Thay x bởi x + z vào phương trình đã cho, ta được Từ hai kết quả trên, ta suy ra f [x + y + z]= f [x + z] f [ yf[x + z] ], x, y, z > 0. f [x + z] f [ yf[x + z] ] = f [x] f [ [y + z] f [x] ], x, y, z > 0. Giả sử tồn tại x 0, z 0 > 0 sao cho f [x 0 + z 0 ] > f [x 0 ]. Trong phương trình trên, ta thay x = x 0, z z = z 0 và y = 0 f [x 0 ] f [x 0 +z 0 ] f [x 0 ] thì có yf[x 0 + z 0 ]=[y + z 0 ] f [x 0 ]. Suy ra f [x 0 + z 0 ]= f [x 0 ], mâu thuẫn. Do đó f [x + z] f [x] với mọi x, z > 0, tức f không tăng. Xét các trường hợp sau: Trường hợp 1: f giảm ngặt. Thay y bởi f [ x + y f [x] vào phương trình đã cho, ta được y ] = f [x] f [y], x, y > 0. f [x] Đảo vị trí của x và y trong phương trình trên với chú ý f giảm ngặt, ta được x + y f [x] = y + x, x, y > 0. f [y] Thay y = 1 vào phương trình trên, ta được x + 1 f [x] = 1 + x 1 f [1], hay f [x]= kx+1 với mọi x > 0, trong đó k = 1 f [1] 1. Do f giảm ngặt từ R+ vào R + nên dễ thấy k > 0. Thử lại, ta thấy hàm số f [x]= kx+1 1 thỏa mãn các yêu cầu của bài toán. Trường hợp : Tồn tại 0 < a < b sao cho f [a]= f [b]. Lần lượt thay x = a và x = b vào phương trình đã cho, ta được f [y + a]= f [a] f [ yf[a] ] = f [b] [ yf[b] ] = f [y + b], y > 0. Từ đó suy ra f [y] = f [y + b a] với mọi y > a. Do f không giảm nên từ đây ta suy ra f [x]=c [C là hằng số dương nào{ đó] với} mọi x > a. Bây giờ, trong phương trình hàm đã a cho, ta cố định x và cho y > max a, f [x] thì có C = f [x + y]= f [x] f [ yf[x] ] = Cf[x], suy ra f [x]=1 với mọi x > 0. Hàm này thỏa mãn các yêu cầu của bài toán. Tóm lại, các hàm số thỏa mãn yêu cầu có dạng f [x]= kx+1 1 với k 0 là hằng số nào đó. Bài toán 6 [Germany TST, 007]. Tìm tất cả các hàm số f : R + R + thỏa mãn [ ] f [x] x f =, x, y > 0. yf[x]+1 xf[y]+1 70

Lời giải. Từ giả thiết, dễ thấy f là một đơn ánh. Thay x bởi khai triển hai vế, ta được hay [ ] x f = xy + xf[z]+1 f [ x ] xf[z]+1 xy xf[z]+1 + 1 = f [x] zf[x]+1 f [x] f [y] zf[x]+1 + 1, f [x] zf[x]+1 vào phương trình đã cho rồi f [x], x, y, z > 0. f [x] f [y]+zf[x]+1 Thay y bởi f [y] vào phương trình trên, ta được [ ] x f x [ f [y]+ f [z] ] f [x] = + 1 f [x] [ f [ f [y] ] + z ], x, y, z > 0. [1] + 1 [ Đảo vị trí của y và z trong phương trình trên với chú ý giá trị của biểu thức f vẫn không đổi, ta được f [ f [y] ] + z = f [ f [z] ] + y, y, z > 0. x x[ f [y]+ f [z]]+1 Từ đó suy ra f [ f [x] ] = x + c [c là một hằng số thực nào đó] với mọi x > 0. Từ đó, phương trình [1] có thể được viết lại thành [ ] x f x [ f [y]+ f [z] ] f [x] = + 1 f [x][y + z + c]+1 = f [x] f [x] f [ f [y + z] ] + 1. Thay x bởi f [x] và y bởi f [y + z] vào phương trình đã cho, ta được [ ] x + c f [x] f = [x + c] f [y + z]+1 f [x] f [ f [y + z] ], x, y, z > 0. + 1 Kết hợp với kết quả trên, ta được [ ] [ x f x [ f [y]+ f [z] ] = f + 1 x + c [x + c] f [y + z]+1 ], x, y, z > 0. Do f đơn ánh nên ta có x x [ f [y]+ f [z] ] + 1 = x + c, x, y, z > 0, [x + c] f [y + z]+1 hay x [ [x + c] f [y + z]+1 ] =[x + c] { x [ f [y]+ f [z] ] + 1 }, x, y, z > 0. Xem hai vế của phương trình trên là các đa thức ẩn x. Hai đa thức này có giá trị bằng nhau với mọi x > 0 nên đồng nhất với nhau, từ đó bằng cách so sánh hệ số của x ở hai vế, ta suy ra f [y + z]= f [y]+ f [z], y, z > 0. Kết quả này chứng tỏ f là hàm cộng tính từ R + vào R +. Suy ra f [x]=kx [k là hằng số dương nào đó]. Thay trở lại phương trình đã cho, ta được k x kxy+ 1 = x, x, y > 0. kxy+ 1 Do đó k = 1. Vậy có duy nhất một hàm số thỏa mãn yêu cầu là f [x]=x. 71 ]

Bài toán 7. Tìm tất cả các hàm số f : R + R + thỏa mãn f [ xf[x]+ f [y] ] = y + f [x], x, y > 0. Lời giải. Từ giả thiết, dễ thấy f đơn ánh. Đặt f [1]=a, ta có f [ f [y]+a ] = y + a, y > 0. Suy ra hàm f có thể nhận mọi giá trị trên [a, + ]. Thay y bởi f [y] vào phương trình hàm đã cho, ta được f [ xf[x]+ f [ f [y] ]] = f [y]+ f [x], x, y > 0. Đảo vị trí của x và y trong phương trình trên với chú ý f đơn ánh, ta được xf[x]+ f [ f [y] ] = yf[y]+ f [ f [x] ], x, y > 0. Từ đó suy ra xf[x] = f [ f [x] ] + c [c là một hằng số thực nào đó] với mọi x > 0. Từ đây, kết hợp với [1] và giả thiết, ta có f [ y + f [x]+a ] = f [ f [ xf[x]+ f [y] ] + a ] = xf[x]+ f [y]+a Do f [x] có thể nhận mọi giá trị trên [a, + ] nên ta có = f [ f [x] ] + f [y]+a + c. f [x + y + a]= f [x]+ f [y]+a + c, x > a 4, y > 0. Từ đây, với mọi x, y > 0 và với mọi z > a 4, ta có và Kết hợp hai kết quả trên lại, ta được f [z + x + y + a]= f [ [z + x + a]+y + a ] = f [z + x + a]+ f [y]+a + c = f [z]+ f [x]+ f [y]+[a + c] f [z + x + y + z]= f [ z +[x + y + a]+a ] = f [z]+ f [x + y + a]+a + c. f [x + y + a]= f [x]+ f [y]+a + c, x, y > 0. Trong phương trình trên, ta lần lượt thay x, y bởi x+y, x+y thì có [ ] x + y f [x + y + a]=f + a + c, x, y > 0. Từ đó suy ra [ ] x + y f = f [x]+ f [y], x, y > 0. 7

Với mọi x, y, z > 0, ta có [ ] x + y + z 4 f = [ f [z + x]+ f [y] ] = f [z]+ f [x]+f[y]. Đảo vị trí của x và y trong phương trình trên, ta được f [x]+ f [y]= f [y]+ f [x], x, y > 0. Từ đó suy ra f [x]=f[x]+m [m là hằng số thực nào đó] với mọi x > 0. Đặt g[x]= f [x]+m thì ta có g[x]=g[x] và [ ] [ ] x + y x + y g[x]+g[y]=f + m = g = g[x + y], x, y > 0. Từ phương trình trên, bằng quy nạp ta chứng minh được g[nx]=ng[x] với mọi n Z + và với mọi x > 0. Do g[nx]= f [nx]+m > m nên ta có g[x] > m n, x > 0, n Z+. Cho n +, ta được g[x] 0 với mọi x > 0. Từ đó, ta suy ra g là hàm cộng tính từ R + vào R 0. Theo tính chất của hàm cộng tính, ta có g[x]=kx [k là hằng số không âm nào đó] với mọi x > 0. Suy ra f [x]=kx m với mọi x > 0. Thay kết quả này trở lại phương trình đã cho, ta được k[kx mx + ky m] m = y +[kx m], x, y > 0. So sánh hệ số của y ở hai vế, ta được k = 1, hay k = 1 [do k 0]. Từ đó, ta có x mx m =[x m], x > 0. So sánh hệ số của x ở hai vế, ta được m = 0. Như vậy, ta có f [x]=x với mọi x > 0. Thử lại, ta thấy thỏa mãn. Vậy có duy nhất một hàm số thỏa mãn yêu cầu là f [x]=x. Bài toán 8. Tìm tất cả các hàm số f : Z + Z + thỏa mãn f [a + b]= f [a]+ f [b]+ f [c]+ f [d] với mọi a, b, c, d nguyên dương sao cho c + d = ab. Ý tưởng chính cho lời giải bài toán trên là tìm các số nguyên dương a, b, c, d, e, g [trong đó {c, d} = {e, f }] thỏa mãn ab = c + d = e + g. Khi đó, ta sẽ có f [c]+ f [d]= f [e]+ f [g]. Ngoài ra, nếu chọn c, d, e, g là các biểu thức có dạng tuyến tính bậc nhất theo n thì ta có thể hoàn tất lời giải bằng phương pháp quy nạp theo n. Ý tưởng biểu diễn một số thành tổng hai bình phương theo hai cách khác nhau gợi ta nghĩ đến đồng nhất thức Lagrange như sau: Với mọi số nguyên m, n, p, q, ta có [m + n ][p + q ]=[mp+ nq] +[mq np] =[mp nq] +[mq + np]. Chọn m = p = và q = 4, ta được [4n + 4] +[n 8] =[4n 4] +[n + 8] = 0[n + 4]. Dựa trên đẳng thức này và một số tính chất riêng của hàm f, ta thu được lời giải như sau. 73

Lời giải. Đặt f [1]=k. Thay a = b = c = d = n vào phương trình đã cho, ta được f [n]=4 f [n], n Z +. [1] Thay a = 10, b = n + 4, c = 4n + 4 và d = n 8 [n > 4], ta được f [n + 14]= f [10]+ f [n + 4]+ f [4n + 4]+ f [n 8] = f [10]+ f [n + 4]+16 f [n + 1]+4 f [n 4]. Mặt khác, thay a = 10, b = n + 4, d = 4n 4 và d = n + 8, ta cũng có Kết hợp hai kết quả trên, ta được f [n + 14]= f [10]+ f [n + 4]+ f [4n 4]+ f [n + 8] = f [10]+ f [n + 4]+16 f [n 1]+4 f [n + 4]. f [n + 4]+4 f [n 1]= f [n 4]+4 f [n + 1], n > 4. [] Bây giờ, ta sẽ tính giá trị của f [], f [3], f [4],..., f [8]. Từ [1], dễ thấy f []=4k, f [4]=16k và f [8]=64k. Do 5 1 = 3 + 1 nên ta có 4 f [3]= f [6]= f [5 + 1]= f [5]+ f [1]+ f [3]+ f [1]= f [5]+ f [3]+a. Mặt khác, ta cũng có 4 1 = + nên f [5]= f [4 + 1]= f [4]+ f [1]+ f []+ f []=5k. Kết hợp với kết quả ở trên, ta được f [3]=9k. Suy ra f [6]=4 f [3]=36k. Ta cũng có 7 + 1 = 5 + 5 = 5 1 nên f [6]= f [5]+ f [1]+ f [5]+ f [5]= f [5]+ f [1]+ f [7]+ f [1], suy ra f [7]= f [5] f [1]=49k. Như vậy, ta chứng minh được f [n]=kn với n {1,,...,8}. Kết hợp với [], ta dễ dàng quy nạp được f [n]=kn với mọi n nguyên dương. Thử lại, ta thấy hàm này thỏa mãn các yêu cầu của bài toán. Vậy có duy nhất một hàm số thỏa mãn yêu cầu là f [n]=kn [k là hằng số nguyên dương nào đó]. Bài toán 9. Tìm tất cả các cặp hàm số f, g : R R thỏa mãn Lời giải. Từ giả thiết, ta cũng có f [x 3 + y]+ f [x + y]=g[x + y], x, y R. [1] f [z 3 + t]+ f [z +t]=g[z + t], z, t R. [] Ta sẽ chứng minh rằng, với mọi a, b R, hệ phương trình sau luôn có nghiệm [x, y, z, t] với x, y, z, t R : x + y = a, z + t = b, x 3 + y = z +t, x + y = z 3 + t. 74

Từ các phương trình thứ nhất và thứ hai, ta lần lượt có x = a y và z = b t. Thay vào phương trình thứ ba, ta được [a y] 3 + y = b t, suy ra t = b y [a y] 3. Thay x = a y, z = b t, t = b y [a y] 3 vào phương trình thứ tư của hệ, ta được a y =[b t] 3 + t = { b [ b y [a y] 3]} 3 + [ b y [a y] 3 ]. Đây là phương trình bậc 9 ẩn y nên luôn có ít nhất một nghiệm thực y 0. Từ đó suy ra hệ luôn có ít nhất một nghiệm thực [x 0, y 0, z 0, t 0 ] với x 0 = a y 0, z 0 = b t 0 và t 0 = b y 0 [a y 0 ] 3. Khẳng định được chứng minh. Từ khẳng định vừa chứng minh và các phương trình [1], [], ta dễ dàng suy ra g[a]=g[b] với mọi a, b R. Do đó g[x] C. Thay trở lại [1], ta được Thay y = x x 3 vào phương trình trên, ta được f [x 3 + y]+ f [x + y]=c, x, y R. f [x x 3 ]= C, x R. Do x x 3 có thể nhận mọi giá trị trên R nên từ đây, ta có f [x]= C với mọi x R. Thử lại, ta thấy f [x]= C và g[x]=c thỏa mãn yêu cầu. Vậy có duy nhất một cặp hàm số f, g thỏa mãn yêu cầu là f [x]= C và g[x]=c [C là một hằng số thực nào đó]. Bài toán 10. Tìm tất cả các hàm số f : R R thỏa mãn đồng thời các điều kiện sau với mọi bộ số thực [a, b, c] : a] Nếu a + b + c 0 thì f [a 3 ]+ f [b 3 ]+ f [c 3 ] 3 f [abc]. b] Nếu a + b + c 0 thì f [a 3 ]+ f [b 3 ]+ f [c 3 ] 3 f [abc]. Lời giải. Để ý rằng, nếu hàm số f thỏa mãn yêu cầu thì hàm số g với g[x]= f [x] C [C là hằng số nào đó] cũng thỏa mãn yêu cầu. Do đó, không mất tính tổng quát, ta chỉ cần xét trường hợp f [0]=0. Từ giả thiết, ta suy ra f [a 3 ]+ f [b 3 ]+ f [c 3 ]=3 f [abc] [1] với mọi a, b, c R mà a + b + c = 0. Thay c = 0 vào [1], ta được f [ a 3 ]= f [b 3 ]= f [a 3 ], suy ra f là hàm lẻ. Từ đây, thay c = [a + b] vào [1], ta được f [ [a + b] 3] = f [a 3 ]+ f [b 3 ]+3 f [ ab[a + b] ], a, b > 0. [] Thay b bởi b + c vào phương trình trên và khai triển vế phải, ta được f [ [a + b + c] 3] = f [a 3 ]+ f [ [b + c] 3] + 3 f [ a[b + c][a + b + c] ] = f [a 3 ]+ f [b 3 ]+ f [c 3 ]+3 f [ bc[b + c] ] + 3 f [ a[b + c][a + b + c] ]. 75

Đảo vị trí của a và c trong dãy đẳng thức trên, ta được f [ bc[b + c] ] + f [ a[b + c][a + b + c] ] = f [ ab[a + b] ] + f [ c[a + b][a + b + c] ]. [3] Tiếp theo, ta sẽ chứng minh rằng với mọi 0 < x < y, hệ phương trình sau luôn có nghiệm [a, b, c] với a, b, c > 0: bc[b + c]=x, a[b + c][a + b + c]=y, ab[a + b]=c[a + b][a + b + c]. Hệ phương trình này tương đương với bc[b + c]=x, a[b + c][a + b + c]=y, ab = c[a + b + c]. Từ phương trình thứ hai và phương trình thứ ba, ta có a b[b+c]=cy. Kết hợp với phương trình thứ nhất, ta được a x = c y. Suy ra c = ka với k = xy. Ngoài ra, từ phương trình thứ nhất và phương trình thứ hai, ta cũng có xa[a+b+c]=ybc. Suy ra k[a+b+ka]=b, hay b = ka[1+k] 1 k. Thay c = ka và b = ka[1+k] 1 k vào phương trình thứ nhất của hệ, ta giải ra được a = 3 x[k 1] k 3 [k+1]. Như thế, hệ [4] luôn có nghiệm [a, b, c] với a, b, c > 0. Kết hợp kết quả này với [3] và chú ý ta được ab[a + b]+c[a + b][a + b + c]=bc[b + c]+a[b + c][a + b + c]=x + y, Dế thấy f [x]+ f [x]=f [ ] x+x nên ta có [ ] x + y f [x]+ f [y]=f, x, y > 0, x < y. [ x + y f [x]+ f [y]=f [4] ], x, y > 0. [5] Từ đây, với mọi x, y, z > 0, ta có [ ] x + y + z 4 f = [ f [x + z]+ f [y] ] = f [x]+ f [z]+f[y]. Đảo vị trí của x và y trong đẳng thức trên, ta được f [x]+ f [y] = f [y]+ f [x] với mọi x, y > 0. Suy ra f [x]= f [x]+m [M là hằng số thực nào đó] với mọi x > 0. Do đó f [8x]= f [4x]+M = 4 f [x]+3m = 8 f [x]+7m, x > 0. Mặt khác, từ [], ta cũng có f [8x]= f [x]+3 f [x]. Suy ra 8 f [x]+7m = f [x]+3 [ f [x]+m ], và do đó M = 0. Như vậy, ta có f [x]= f [x] với mọi x > 0. Kết hợp với [5], ta được f [x + y]= f [x]+ f [y], x, y > 0. 76

Mặt khác, từ điều kiện a] của bài toán và f [0]=0, ta dễ dàng suy ra f [a] 0 với mọi a 0. Như vậy, f là hàm cộng tính trên R + và f [x] 0 với mọi x > 0 nên ta có f [x]=lx [l là hằng số không âm nào đó] với mọi x > 0. Mà f lẻ nên f [x]=lx, x R. Thử lại, ta thấy thỏa mãn. Vậy các hàm số thỏa mãn yêu cầu có dạng f [x]=lx +C [l, C là các hằng số thực, l 0]. Bài toán 11. Tìm tất cả các hàm số f : R R thỏa mãn f [ x + f [y] ] = f [y + 3]+xf[y]+ f [x] 3, x, y R. Lời giải. Dễ thấy f [x] 0 không thỏa mãn phương trình đã cho nên tồn tại y 0 R sao cho f [y 0 ] 0. Thay y = y 0 vào phương trình đã cho, ta được f [ x + f [y 0 ] ] f [x]=xf[y 0 ]+ f [y 0 + 3] 3, x R. Vế phải của phương trình trên là một hàm bậc nhất ẩn x nên nó có thể nhận mọi giá trị trên R. Từ đó suy ra hiệu f [u] f [v] có thể nhận mọi giá trị trên R. Bây giờ, thay x bởi x f [y] vào phương trình đã cho, ta được f [ x f [y] ] = f [x] f [y + 3] [ x f [y] ] f [y]+3, x, y R. Trong phương trình trên, ta thay x bởi x + f [z] thì được f [ x + f [z] f [y] ] = f [ x + f [z] ] f [y + 3] [ x + f [z] f [y] ] f [y]+3 = f [x]+ f [z + 3]+xf[z] f [y + 3] [ x + f [z] f [y] ] f [y] = f [x]+x [ f [z] f [y] ] + f [z + 3] f [y + 3]+ f [y] [ f [y] f [z] ]. Đảo vị trí của y và z trong phương trình trên rồi cộng phương trình thu được và phương trình trên lại theo vế, ta được f [ x + f [z] f [y] ] + f [ x + f [y] f [z] ] = f [x]+ [ f [y] f [z] ]. Do hiệu f [y] f [z] có thể nhận mọi giá trị trên R nên từ đây ta có f [x + y]+ f [x y]= f [x]+y, x, y R. Đặt f [0]=a và g[x]= f [x] x a thì ta có g[0]=0 và g[x + y]+g[x y]=g[x], x, y R. Thay x = y vào phương trình này, ta được g[x]=g[x] với mọi x R nên ta có g[x + y]+g[x y]=g[x], x, y R. Lần lượt thay x, y bởi x+y, x y vào phương trình trên, ta suy ra g là hàm cộng tính. Tiếp theo, thay x = 0 vào phương trình đã cho, ta được f [ f [y] ] = f [y + 3]+a 3, y R. 77

Thay f [y]=g[y]+y + a vào và rút gọn với chú ý g cộng tính, ta được g [ g[y] ] + g [y]+[y + a]g[y]+[a 3][y + a + ]+g[a] g[3]=0. Thay y = 0 vào hương trình trên, ta được Từ đó suy ra [a 3][a + ]+g[a] g[3]=0. g [ g[y] ] + g [y]+[y + a]g[y]+[a 3]y = 0, y R. Thay y bởi ny [n Z] vào phương trình trên, ta được ng [ g[y] ] + n g [y]+n[n y + a]g[y]+n [a 3]y = 0, y R, n Z. Ta xem vế trái là một đa thức ẩn n. Đa thức này có giá trị bằng 0 tại vô hạn giá trị của n nên nó phải đồng nhất bằng 0. Từ đó suy ra hệ số của n 3 phải bằng 0, hay ta có y g[y] =0 với mọi y R. Kết hợp với g[0] =0, ta suy ra g[y] =0 với mọi y R. Từ đây với chú ý [a 3][a + ]+g[a] g[3] =0, ta tính được a = 3 hoặc a =. Suy ra f [x] =x + 3 với mọi x R hoặc f [x]=x với mọi x R. Thử lại, ta thấy chỉ có hàm f [x]=x + 3 thỏa mãn yêu cầu. Vậy có duy nhất một hàm số thỏa mãn yêu cầu là f [x]=x + 3. Trên đây, chúng tôi đã giới thiệu cùng bạn đọc một số bài toán hay giải được bằng phương pháp thêm biến. Rất mong sẽ nhận được trao đổi đóng góp xây dựng từ bạn đọc gần xa. Dưới đây là một số bài tập khác cũng có thể giải bằng phương pháp này: Bài tập 1. Tìm tất cả các hàm số f : R R thỏa mãn f [x + y]= f [x] f [y] f [xy], x, y R. Bài tập [IMO Shortlist, 005]. Tìm tất cả các hàm số f : R + R + thỏa mãn f [x] f [y]= f [ x + yf[x] ], x, y > 0. Bài tập 3. Tìm tất cả các hàm số f : R + R + thỏa mãn f [ x + f [y] ] = f [x]+ f [x + y] x, x, y > 0. Bài tập 4. Tìm tất cả các hàm số f liên tục, f : R R thỏa mãn f [x + y]+ f [xy]= f [x]+ f [y]+ f [xy + 1], x, y R. Bài tập 5. Tìm tất cả các hàm số f : Z + Z + thỏa mãn f [ f [m]+ f [n] ] = m + n, m, n Z +. Bài tập 6. Cho hàm số f : R R thỏa mãn f [0]=0 và Chứng minh rằng f là hàm cộng tính. f [x]+ f [y]= f [x + y xy]+ f [xy], x, y R. 78

SÁNG TẠO -LÀM CHẶT Ngô Văn Thái, Thái Bình Một tính chất quan trọng của bất đẳng thức toán học mà học sinh được biết ngay từ khi còn đang học tiểu học, đó là tính chất bắc cầu: Nếu A B; B C thì A C: Tính chất này được sử dụng làm phép suy luận logic ở hầu khắp mọi bài toán chứng minh bất đẳng thức. Thế nhưng khi đứng trước một bài toán chứng minh bất đẳng thức A C; không phải bài toán nào người ra đề cũng để người giải nhìn ra ngay A B; B C: Nếu bài toán đề ra mà người giải nhìn thấy ngay A B; B C thì bài toán đó quá tầm thường. Còn với những bài toán bất đẳng thức hay và khó, người giải phải sử dụng các kiến thức cơ bản phù hợp, cách giải phải tường minh, linh hoạt ngắn gọn để được A B và B C; thì người giải đã làm tốt được hai việc. Việc thứ nhất là đã hoàn thành chứng minh bài toán đề ra, việc thứ hai là đã sáng tạo ra được một bài toán bất đẳng thức chặt hơn, khó hơn [có thể hay hơn] bài toán đã cho. Để thấy được vẻ đẹp muôn màu của sự sáng tạo bất đẳng thức toán học, tôi xin giới thiệu bài viết: Sáng tạo bất đẳng thức bằng cách làm chặt một số bất đẳng thức nổi tiếng dựa vào ý tưởng từ Tính chất bắc cầu của bất đẳng thức. Nội dung bài viết gồm năm phần sau đây: Làm chặt bất đẳng thức AM-GM. Làm chặt bất đẳng thức Cauchy-Schwarz. Làm chặt bài toán bất đẳng thức đăng trên báo Toán học và Tuổi trẻ. Làm chặt một bài toán bất đẳng thức thi Toán học Quốc Tế. Giới thiệu những bài toán mới được làm chặt từ những bài toán đã biết. 1. Làm chặt bất đẳng thức AM-GM Bất đẳng thức AM-GM là một bất đẳng thức cơ bản kinh điển quan trọng nhất của toán học sơ cấp, vì nó đã có khá nhiều cách chứng minh được đưa ra, hàng chục mở rộng, hàng chục kết quả chặt hơn đăng trên các diễn đàn toán học. Phần này tôi xin giới thiệu một kết quả chặt hơn bất đẳng thức AM-GM khác được suy ra từ chính cách chứng minh mới bất đẳng thức AM-GM. Bất đẳng thức AM-GM. Với mọi số thực không âm a 1 ;a ;:::;a n.n / ta có a 1 C a CCa n n np a 1 a a n : [1] 79

Lời giải. Nếu a 1 a a n D 0 thì [1] đúng. Xét a 1 a a n > 0; dễ thấy với mọi X 0 thì.1 X/ ; n 1 C X C X CCX n 1 cùng dấu. Ta có thể giả sử hay là Bây giờ đặt X D n q.1 X/ n 1 C X C X CCX n 1 0; a n n 1 p a 1 a a n 1 ; thì.n 1/ C n.1 X/.1 X n / 0;.n 1/ C X n nx: r p a n n n 1 a 1 a a n n 1 a n n 1 p a 1 a a n 1 ;.n 1/ n 1p a 1 a a n 1 C a n n np a 1 a a n ; [] Do đó.n / n p a 1 a a n C a n 1.n 1/ n 1p a 1 a a n 1 ; p a 1 a C a 3 3 3p a 1 a a 3 ; a 1 C a p a 1 a : Cộng vế với vế của n 1 ở trên và rút gọn sẽ được a 1 C a CCa n n np a 1 a a n : Đẳng thức xảy ra khi và chỉ khi a 1 D a DDa n : Ta có điều phải chứng minh. Như vậy từ bất đẳng thức [] chúng ta đã có thêm một cách chứng minh mới bất đẳng thức AM-GM khá gọn, ngoài ra khi sử dụng bất đẳng thức [] lại dễ dàng rút ra được kết quả chặt hơn bất đẳng thức AM-GM sau đây. Với các số tự nhiên n k p và n số thực không âm, khi đó a 1 C a CCa n p pp a 1 a a p C a pc1 CCa n k kp a 1 a a k C a kc1 CCa n n np a 1 a a n :. Làm chặt bất đẳng thức Cauchy-Schwarz Trong toán học bất đẳng thức AM-GM và Cauchy-Schwarz được dùng làm định lý cơ bản của lý thuyết bất đẳng thức. Với tư cách là hai hòn đá tảng để nhiều kết luận quan trọng khác của toán học dựa vào, cặp bất đẳng thức AM-GM, Cauchy-Schwarz được sử dụng khá phổ biến ở 80

phần lớn các bài toán chứng minh bất đẳng thức. Ngoài ra một số hệ quả của cặp bất đẳng thức này có thể vận dụng để giải hàng loạt các bài toán thú vị về cực đại và cực tiểu. Bất đẳng thức Cauchy-Schwarz. Cho các số thực a 1 ;a ;:::;a n và b 1 ;b ;:::;b n ;.n / khi đó a 1 C a CCa n b 1 C b n CCb.a1 b 1 C a b CCa n b n / : [3] Lời giải. Hiện nay bất đẳng thức Cauchy-Schwarz cũng có khá nhiều cách chứng khác nhau, tất cả các cách chứng minh đó đều ngắn gọn đặc sắc, xin giới thiệu một cách chứng minh trong số những cách chứng minh đã có như sau. Nếu a 1 C a CCa n D 0 tức a 1 D a DDa n D 0 thì [3] đúng. Nếu a1 C a CCa n > 0; xét! nx f.x/ D x C Do đó biệt thức a i 0 D! nx a i b i x C! nx a i b i nx nx a i b i D nx.a i x C b i / 0; 8x R:! nx b i! 0: Cho nên a 1 C a CCa n b 1 C b n CCb.a1 b 1 C a b CCa n b n / : Đẳng thức xảy ra khi và chỉ khi a 1 chứng minh. b 1 D a b DD a n b n : 1 Bất đẳng thức Cauchy-Schwarz được Từ bất đẳng thức Cauchy-Schwarz suy ra hai hệ quả để sử dụng trong bài viết này: Hệ quả 1. Với các số thực a i ;b i ;; n thì a i C a ic1 b i C bic1.ai b i C a ic1 b ic1 / : [4] Hệ quả. Với hai dãy hữu hạn số thực a 1 ;a ;:::;a n và b 1 ;b ;:::;b n ;.n / khi đó nx q a i C b i!! nx a i C! nx b i : [5] Lời giải. Xét biểu thức nx q a i C b i!! nx a i D nx q a i C b i C nx q a i C b i! nx a i nx a i! " nx q #" nx q # D ai C bi C a i ai C bi a i : 1 Ở đây chúng ta quy ước nếu mẫu số nào bằng 0 thì tử số của phân số ấy cũng bằng 0: 81

Theo bất đẳng thức Cauchy-Schwarz thì " nx q #" nx q # ai C bi C a i ai C bi a i Suy ra D nx nx q b i! D q a i C b i! " nx nx jb i j!! nx a i C s q a i C b i C a i q a i C b i a i # nx b ˇ iˇˇˇˇˇ! D! nx b i : Vậy ta có điều phải chứng minh. Đẳng thức xảy ra khi và chỉ khi a 1 b 1 D a b Bây giờ ta sử dụng hệ quả trên để làm chặt bất đẳng thức Cauchy-Schwarz.! nx b i : DD a n b n : Dạng 1. Với hai dãy hữu hạn số thực a 1 ;a ;:::;a n và b 1 ;b ;:::;b n ;.n / khi đó nx a i! nx b i! ở đây quy ước a nc1 D a 1 ;b nc1 D b 1 :! nx a i b i C 1 4 nx ja i b ic1 a ic1 b i j! ; [6] Lời giải. Theo [4] và với i 0; i D 1; n; thì a i C a ic1 b i C bic1.ai b i C a ic1 b ic1 / ; suy ra tương đương với a i C a ic1 b i C bic1 D.ai b i C a ic1 b ic1 / C. i/ ; Theo bất đẳng thức Cauchy-Schwarz thì 4 nx Do đó 4 nx a i a i! nx! nx b i b i!! Mặt khác theo [5] ta có D " nx " nx i D ja i b ic1 a ic1 b i j : a i C a ic1 #" nx b i C b ic1 # q a i C aic1 # b i C bic1 D " nx " nx q a i C aic1 # b i C bic1 : q.a i b i C a ic1 b ic1 / C. i/ # : " nx q # " nx.a i b i C a ic1 b ic1 / C. i/.a i b i C a ic1 b ic1 /# C nx i! : 8

Cho nên hay suy ra 4 4 nx nx nx a i a i a i! nx! nx! nx b i b i b i!!! " nx.a i b i C a ic1 b ic1 /# C 4 Đẳng thức xảy ra khi và chỉ khi a 1 b 1 D a b Từ [6] lại có hệ quả đẹp sau! nx a i b i C! nx a i b i C 1 4 DD a n b n : Với các số thực dương a 1 ;a ;:::;a n ;.n /; thì nx với quy ước a nc1 D a 1 : a i! nx! 1 n C 1 4 a i nx ˇ nx i! ; nx ja i b ic1 a ic1 b i j! ; nx ja i b ic1 a ic1 b i j! : r ai a ic1 r aic1 a i ˇ ˇ! ; Dạng. Với hai dãy hữu hạn số thực a 1 ;a ;:::;a n và b 1 ;b ;:::;b n ;.n / khi đó nx a i! nx b i! " nx # nx a i C b i a i b i a i C b i!! nx a i b i : Lời giải. Ta chứng minh vế trái! nx nx a i b i! " nx # nx a i C b i a i b i a i C b i! : Bất đẳng thức này tương đương với mỗi bất đẳng thức bên dưới n! P n P nx a ai b i bi i ; a i C bi np a i C bi! nx nx a i ai b i ai C bi nx a i P n P n ai bi np a i C bi ; 83

nx a i a i b i ai C bi " nx nx ai 4 a i C bi a 4 i a i C b i #" nx nx a i n P n P ai ai n P bi np P ai C n np P ai C n bi # a i C b i ; bi! nx a i : Hiển nhiên đúng theo bất đẳng thức Cauchy-Schwarz. Mặt khác cũng theo bất đẳng thức Cauchy-Schwarz thì " nx # nx a i C b i a i b i a i C b i! nx q a i b i! D nx ja i b i j! Vế phải cũng được chứng minh. Đẳng thức xảy ra khi và chỉ khi a 1 b 1 D a b ;! nx a i b i : DD a n b n : 3. Làm chặt bài toán bất đẳng thức đăng trên báo Toán học và Tuổi trẻ. Trong cuốn Tuyển tập 5 năm tạp chí Toán học và Tuổi trẻ có đăng bài Mở rộng một bất đẳng thức của tác giả Tạ Hồng Quảng, mở đầu bài báo tác giả có viết: Trên báo Gazeta Matematica của Romania số 9; 1989 trang 335 tác giả Gheoghe Marghescu đưa ra bất đẳng thức sau Bài toán 1. Cho các số thực a 1 ;a ;:::;a n và b 1 ;b ;:::;b n ;.n / chứng minh rằng nx q! nx! 1 ai C bi n p n: [7] a i C b i Sau đó tác giả Tạ Hồng Quảng đã đưa ra một bài toán bất đẳng thức chặt hơn [7] như sau Bài toán. Cho các số thực a 1 ;a ;:::;a n và b 1 ;b ;:::;b n.n / chứng minh rằng nx q! nx! 1 ai C bi p n : [8] a i C b i Phần nội dung tiếp theo của bài báo là lời giải [8] rất hay và ngắn gọn, đồng thời từ lời giải [8], tác giả cũng đã sáng tạo ra một số bài toán bất đẳng thức mở rộng đẹp, hấp dẫn. Để tiếp nối bài viết của tác giả tôi xin đưa ra một số bài toán bất đẳng thức mới chặt hơn [8] như sau. 84

Bài toán 3. Cho các số thực a 1 ;a ;:::;a n và b 1 ;b ;:::;b n ;.n / chứng minh rằng 0 B @ v nx u t q a i C b i a i C b i 1 C A n p : [9] Lời giải. Dễ thấy với a i ;b i > 0; 8i D 1; n thì q a i C b i a i C b i 1 p ; suy ra Cho nên tương đương 0 B @ v uu q t ai C bi a i C b 4p 1 : i v q nx u t ai C bi a i C b p n 4 i ; v nx u t q a i C b i a i C b i Đẳng thức xảy ra khi và chỉ khi a i D b i ; 8i D 1; n: 1 C A n p : Quả thật theo đánh giá chủ quan của tôi [9] không phải là một bài toán hay, vì bài toán này được tạo thành thuần túy chỉ gồm phép cộng vế với vế của n bất đẳng thức đơn giản cùng chiều, sau đó đem bình phương cả hai vế của bất đẳng thức đó lại mà thôi. Thế nhưng [9] lại chặt hơn [8] và đương nhiên chặt hơn [7], bởi vì theo bất đẳng thức Cauchy-Schwarz ta có nx q! nx ai C bi 0 v! 1 nx u t B a i C b i @ q a i C b i a i C b i 1 C A n p : Bài toán 4. Cho các số thực a 1 ;a ;:::;a n và b 1 ;b ;:::;b n ;.n /: Chứng minh rằng nx q! nx ai C bi 1 a i C b i! 0 p n C B @ v nx u t q ai C bi p 1 a i C b i 1 C A : 85

Lời giải. Với a i ;b i > 0; 8i D 1; n thì q a i C b i a i C b i 1 p ; do đó tồn tại i 0 để q a i C b i a i C b i Theo bất đẳng thức Cauchy-Schwarz thì nx q! nx ai C bi D 0 v! 1 nx u t B a i C b i @ Mặt khác theo [5], vế phải của [11] ta được 4 suy ra tiếp s 3 nx 1 4p C. i/ 5 Từ [10] ta được vì vậy nx 4 nx 1 4p C. i/ [10] q a i C b i a i C b i 1 4p! C s 3 nx 1 4p C. i/ 5 i D q! nx ai C bi 1 s 3 nx C A D 4 1 4p C. i/ 5 : nx n p C k! D n p C nx i! : v q u t ai C bi p 1 ; 8i D 1; n; a i C b i 1 a i C b i! 0 p n C B @ v nx u t nx i! ; q ai C bi p 1 a i C b i 1 C A : [11] Đẳng thức xảy ra khi và chỉ khi a 1 D a DDa n D b 1 D b DDb n : Bài toán được chứng minh. 4. Làm chặt một bài toán bất đẳng thức thi Toán học Quốc Tế Trở lại với bài bất đẳng thức trong kỳ thi IMO 001 tại Mỹ 86

Bài toán. Cho các số thực dương x; y; z: Chứng minh rằng x p x C 8yz C y p y C 8zx C z p z C 8xy 1: Lời giải. Với ba số thực dương a; b; c áp dụng bất đẳng thức AM-GM ta được.a 4 C b 4 C c 4 /.a 4 C b c / D a 8 C 4a 4 b c C 4b 4 c 4 a 8 C 8a b 3 c 3 ; [1] suy ra hay là Tương tự Do đó 1 a 6 C 8b 3 c 3 a.a 4 C b 4 C c 4 / ; s a 6 a 6 C 8b 3 c a 4 3 a 4 C b 4 C c : 4 s b 6 b 6 C 8c 3 a b 4 3 a 4 C b 4 C c ; 4 s c 6 c 6 C 8a 3 b c 4 3 a 4 C b 4 C c : 4 a 3 p a6 C 8b 3 c 3 C b 3 p b6 C 8c 3 a 3 C c 3 p c6 C 8a 3 b 3 a4 C b 4 C c 4 a 4 C b 4 C c 4 D 1: Cuối cùng thay x D a 3 ;yd b 3 ;zd c 3 ta sẽ có điều phải chứng minh. Dấu đẳng thức xảy ra khi và chỉ khi x D y D z: Đến đây độc giả đã thấy bài toán bất đẳng thức thi IMO 001 được chứng minh thông qua [1]. Bây giờ ta làm chặt [1] và lặp lại cách làm tương tự, sẽ được một chuỗi bất đẳng thức chặt hơn bất đẳng thức IMO 001 sau đây. Bài toán. Cho các số thực dương x; y; z: Chứng minh rằng X x p x C 8yz X x qx X x q C.y 3 C z 3 / 3 x C.y C z/ X q x 1: x C y C z C y 4 3 z 3 C y 3 z 4 3 Lời giải. Với ba số dương a; b; c áp dụng bất đẳng thức AM-GM và đánh giá b 3 C c 3 b C c 3 8b 3 c 3 ; 87

ta được a 4 C b 4 C c 4 D a 8 C a 4 b 4 C b 8 C a 4 c 4 C c 8 C a 4 b 4 C b 4 c 4 C a 4 c 4 C b 4 c 4 Suy ra a 8 C a b 6 C a c 6 C a b 4 c C a b c 4 a 8 C a b 6 C a c 6 C 4a b 3 c 3 h D a a 6 C b 3 C c 3 i a ha 6 C b C c i 3 a a 6 C 8b 3 c 3 : 1 a 6 C 8b 3 c 1 3 a 6 C.b C c / 1 3 a 6 C.b 3 C c 3 / 1 a 6 C b 6 C c 6 C b 4 c C b c 4 a.a 4 C b 4 C c 4 / ; tương đương với s s s a 6 a 6 C 8b 3 c a 6 3 a 6 C.b C c / a 6 3 a 6 C.b 3 C c 3 / s a 6 a 6 C b 6 C c 6 C b 4 c C b c a 4 4.a 4 C b 4 C c 4 / Tương tự thì s s s b 6 b 6 C 8c 3 a b 6 3 b 6 C.c C a / b 6 3 b 6 C.c 3 C a 3 / s b 6 b 6 C c 6 C a 6 C c 4 a C c a b 4 4.a 4 C b 4 C c 4 / : s s s c 6 c 6 C 8a 3 b c 6 3 c 6 C.a C b / c 6 3 c 6 C.a 3 C b 3 / s c 6 c 6 C a 6 C b 6 C a 4 b C a b c 4 4.a 4 C b 4 C c 4 / : [13] [14] [15] Cộng tương ứng vế của [13], [14], [15] rồi rút gọn và đặt x D a 3 ;yd b 3 ;zd c 3 ta được X x p x C 8yz X x qx X x q C.y 3 C z 3 / 3 x C.y C z/ X q x 1: x C y C z C y 4 3 z 3 C y 3 z 4 3 Bài toán được chứng minh. 88

5. Giới thiệu những bài toán mới được làm chặt từ những bài toán đã biết Bài toán 5. Cho a 1 ;a ;:::;a n là các số thực dương. Chứng minh rằng r r a 1 C a CCa n np a1 a n a 1 a a n C n CC n a a 3 Bài toán 6. Cho a 1 ;a ;:::;a n là các số thực không âm. Chứng minh rằng r an a1 C a CCa n n a1 C a CCa n 1 n 1 a n : a 1 : n n 1 Bài toán 7. Cho các số thực a 1 ;a ;:::;a n và b 1 ;b ;:::;b n ;.n / khi đó! nx nx! " nx #" # nx a i b i a i C b ai b i i a i C bi! nx a i b i C 1 nx s a s 4 i C bi a a ic1 b ic1 ic1 4 ˇ aic1 C C 3 ic1 b a i bi 5 b ic1 ai C bi ˇ! nx a i b i ; quy ước a nc1 D a 1 ;b nc1 D b 1 : Bài toán 8. Cho ba số thực dương a; b; c thỏa mãn abc D 1: Chứng minh rằng với mọi số nguyên dương k 3 ta luôn có 1 a k.b C c/ C 1 b k.c C a/ C 1 c k.a C b/ 3 : Bài toán 9. Cho ba số thực dương a; b; c và hai số nguyên dương p q: Chứng minh rằng a p.a C 8bc/ C q p b p.b C 8ca/ C q p c p q p.a C b C c/ p q.c C 8ab/ p : Bài toán 10. Với các số thực a 1 ;a ;:::;a n và b 1 ;b ;:::;b n ;.n / ta có nx q! nx! " nx q 1 ai C bi ai C bi C 1 # q 6 nx ai C b 1 i 64 q a i C b i a i C b i ai C bi C 1 0 v nx u t B @ q a i C b i a i C b i 1 C A p n C 6 4 v nx u t q ai C bi p 1 a i C b i 3 7 5 a i Cb i a i Cb i 3 7 5 n p n p n: 89

Bài toán 11. Cho các số thực a 1 ;a ;:::;a n và b 1 ;b ;:::;b n ;.n 1/ khi đó giả sử c 1 ;c ;:::;c n là một hoán vị nào đó của các số a 1 ;a ;:::;a n : Chứng minh rằng nx q! nx! 1 ai C bi p n : c i C b i Bài toán 1. Ta chia số thực a 1 ;a ;:::;a n và b 1 ;b ;:::;b n ;.n 1/ làm n nhóm sao cho trong mỗi nhóm có ít nhất một số. Gọi M 1 ;M ;:::;M n lần lượt là tổng các số trong mỗi nhóm. Chứng minh rằng nx q! nx ai C bi Bài toán 13. Cho a i ;b i > 0; 8i D 1; n: Chứng minh rằng nx q! nx ai C bi 1 a i C b i! p 1 4 1 M i nx r! n p : ai a i C b i Bài toán 14. Cho a i ;b i ;c i > 0; 8i D 1; n: Chứng minh rằng nx q! nx! 1 ai C bi C ci a i C b i C c i Bài toán 15. Cho a i ;b i ;c i > 0; 8i D 1; n: Chứng minh rằng nx q! nx ai C bi C ci 1 a i C b i C c i! 0 p n C B 3 @ v nx u t! C 0 @ s nx n p 3 : b i a i C b i 13 A q ai C bi C ci p 1 a i C b i C c i 3 5 : 1 C A : Bài toán 16. Cho a i ;b i ;c i > 0; 8i D 1; n và.x 1 ;x ;:::;x n / ;.y 1 ;y ;:::;y n / ;.z 1 ;z ;:::;z n / lần lượt tương ứng là một hoán vị tùy ý của.a 1 ;a ;:::;a n / ;.b 1 ;b ;:::;b n / ;.c 1 ;c ;:::;c n / : Chứng minh rằng nx q! nx ai C bi C ci 1 x i C y i C z i Bài toán 17. Cho a i ;b i > 0; 8i D 1; n: Chứng minh rằng " nx #" # nx a m i C b m 1 i.a i C b i / m Bài toán 18. Cho a i ;b i > 0; 8i D 1; n: Chứng minh rằng! n p 3 : n m 1 : nx p m a m i C bi m a i C b i n mp m 1 : 90

Bài toán 19. Cho a i ;b i > 0; 8i D 1; n: Chứng minh rằng " nx #" nx # m 1.a i C b i / mp ai C mp nmc1 b i : m 1 Bài toán 0. Cho a i ;b i > 0; 8i D 1; n: Chứng minh rằng! nx q 1 nx p m ai m C bi m n mp 4 ai C m 1 Bài toán 1. Cho a i ;b i > 0; 8i D 1; n: Chứng minh rằng " nx q nx! m 1 m ai m C bi m mp.n mp ai C / m 1! 3 nx p bi 5 : nx! m # p m bi : Bài toán. Cho các số thực dương a 1 ;a ;:::;a n ;.n 1/: Chứng minh rằng ny a nc i a i C n! n nx a i : Bài toán 3. Cho các số thực dương a 1 ;a ;:::;a n ;.n 1; k /: Chứng minh rằng a kc1 1 a k C akc1 a3 k CC akc1 n a1 k a 1 a C a a 3 CC a n a 1 : Bài toán 4. Cho các số thực dương x 1 ;x ;:::;x n ;.n 1; k / có tổng bằng 1: Với ; ˇ; 0 thỏa mãn C ˇ C > 0: Chứng minh rằng x k 1 x 1 C ˇx C x 3 C x k x C ˇx 3 C x 4 CC x k n x n C ˇx 1 C x Bài toán 5. Cho ba số thực dương a; b; c và số nguyên k 3: Chứng minh rằng a 4 C b c.a C bc/ k C b4 C c a.b C ca/ k C c4 C a b.c C ab/ k 3 k.a C b C c/ k 4 : Bài toán 6. Với sáu số thực dương a; b; c; x; y; z và số nguyên dương k thì a k x C bk y C ck z.a C b C c/k 3 k.x C y C z/ : Bài toán 7. Cho ba số thực dương a; b; c và số nguyên n : Chứng minh rằng r a nc1 b C c C bnc1 c C a C cnc1.a C b C c/n 1 n an C b n C c n : a C b 3 n 3 1 n k. C ˇ C / : Bài toán 8. Cho ba số thực dương a; b; c và p là số nguyên dương. Chứng minh rằng a pp b C c C b pp c C a C c pp a C b.a C b C c/ p 1 p : 91

Bài toán 9. Cho ba số thực dương a; b; c: Chứng minh rằng r r r a 4 b b c C C 4 c c a C C 4 a b C 3p 3: Bài toán 30. Với k là số tự nhiên và ba số thực dương x k ;y k ;z k ; ;ˇthỏa mãn nx x k D kd1 nx y k D kd1 Chứng minh rằng với mọi số nguyên p ta có nx kd1.1 C x k / p y k C ˇz k nx z k D 1: kd1 p n p. C ˇ/ : Bài toán 31. Cho a k ;b k > 0; k D 1; n và số nguyên dương m: Chứng minh rằng! nx q nx! m a m k C 1 bm k a k C b mp n : k m 1 kd1 kd1 Tài liệu [1] Nguyễn Văn Mậu, Bất đẳng thức định lí và áp dụng, NXB Giáo dục, 006: [] Phạm Kim Hùng, Sáng tạo bất đẳng thức, NXB Tri thức, 006: [3] Phạm Văn Thuận, Lê Vĩ, Bất đẳng thức suy luận và khám phá, NXB ĐHQG Hà Nội, 007: [4] Tạp chí Toán học và Tuổi trẻ. [5] Tạp chí Epilon. 9

BẢY TRỤ CỘT THÔNG THÁI CỦA THỐNG KÊ HỌC Nguyễn Văn Tuấn GIỚI THIỆU Chuyên mục điểm sách kỳ này của Epsilon chúng tôi vinh dự nhận được bài viết từ giáo sư Nguyễn Văn Tuấn, với bài điểm về cuốn "Bảy trụ cột thông thái của thống kê học" ["The Seven Pillars of Statistical Wisdom"] của tác giả Stephen M. Stigler. Đây là một quyển sách mang lại một góc nhìn và những lý giải mới về những phương pháp thống kê đã được sử dụng xuyên suốt chiều dài lịch sử của thống kê học. Chúng tôi chủ quan cho rằng tác phẩm sẽ giúp cho người đọc hiểu sâu hơn về các phương pháp thống kê, và qua đó, giúp chúng ta không chỉ dùng thống kê như những công cụ, mà còn hiểu được đó là cả một hệ thống tư duy. Xin mời độc giả đến với bài điểm sách này của giáo sư Nguyễn Văn Tuấn. Một trong những cuốn sách khoa học mà tôi rất thích trong vài tháng gần đây là cuốn "The Seven Pillars of Statistical Wisdom" của tác giả Stephen M. Stigler [1]. Đây là một cuốn sách nhỏ [00 trang] cung cấp cho chúng ta những lí giải cực kì lí thú về khoa học thống kê và lịch sử đằng sau những phương pháp mà chúng ta sử dụng trong suy luận khoa học. Như tựa đề cuốn sách, tác giả Stigler tập trung vào giải thích 7 trụ cột thông thái của thống kê học, và tôi thử tóm lược theo cách hiểu của tôi dưới đây. Nhưng trước khi giải thích, tôi thấy cần phải dành vài chữ giải thích ý nghĩa chữ wisdom, mà tôi thấy hơi khó dịch sang tiếng Việt mình. Ở mức độc đơn giản nhất, wisdom là thông thái, khôn ngoan. Nhưng nếu có kinh nghiệm cọ sát với xã hội nói tiếng Anh thì hình như chữ "thông thái" và "khôn ngoan" có vẻ không tương đương với wisdom. Trong ngữ cảnh của tựa đề cuốn sách này, tôi hiểu wisdom như là những tri thức và trải nghiệm được đúc kết qua những trải nghiệm thực tế, cũng giống như những câu ca dao là những tinh tuý về ứng xử ở đời mà cha ông chúng ta đã đúc kết và truyền lại. Để hiểu các khái niệm trong sách, cần phải phân biệt dữ liệu [data] và thông tin [information]. Dữ liệu là những gì chúng ta thu thập từ nghiên cứu. Để chuyển hoá dữ liệu thành thông tin, chúng ta phải áp dụng phương pháp phân tích thống kê. Nói cách khác, thống kê học là công cụ để chúng ta thu nạp thông tin từ dữ liệu. Dĩ nhiên, từ thông tin, chúng ta có thể biến thành kiến thức [knowledge] qua dùng phương pháp qui nạp khoa học. Sau khi phân biệt được sự khác biệt giữa dữ liệu, thông tin, và kiến thức, chúng ta thử điểm qua 7 trụ cột mà tác giả Stigler đề cập trong cuốn sách. Bảy trụ cột này cũng có thể xem là 7 nghịch lí, và tôi sẽ giải thích thêm dưới đây: 93

Trụ cột 1 - aggregation: Qui luật loại bỏ dữ liệu để thu nạp thông tin Trong phần này, Stigler lí giải và đưa ra một nhận xét làm chúng ta ngạc nhiên: đó là chúng ta thu nạp kiến thức bằng cách loại bỏ thông tin! Chẳng hạn như đối phó với một dãy số liệu về chiều cao, chúng ta chỉ cần tính một số trung bình, và dùng nó như là một thông tin để kiến tạo tri thức. Còn tất cả những con số để tạo nên số trung bình thì bị loại bỏ, không được đề cập đến. Mỗi ngày, chúng ta đọc và nghe biết bao số trung bình, từ thị trường chứng khoán, chính sách kinh tế, đến nghiên cứu y khoa, tất cả đều dùng số trung bình để đi đến những quyết định phức tạp. Lịch sử và sự ra đời của con số trung bình cũng được tác giả diễn giải rất tường tận. Thống kê học, hay ít ra là các khái niệm thống kê học, đã được sử dụng trong thiên văn học từ thế kỉ 18, phải đợi đến giữa thế kỉ 19 thì mới thịnh hành. Lí do là vấn đề đo lường và liên quan đến giá trị trung bình. Tác giả Stigler chỉ ra rằng nếu chúng ta đo lường [chẳng hạn như] Sao Mộc, thì chúng ta biết rõ đó là một thực thể, nó ở một vị trí và chúng ta có thể ước tính sai số. Nhưng nếu chúng ta đo lường tuổi thọ hay mức độ lạm phát kinh tế, thì chúng ta không có được cái "xa xỉ" như đo lường Sao Mộc, bởi vì những biến số như tuổi thọ nó xuất phát từ mẫu mà chúng ta có được và chúng ta không biết được giá trị thật của quần thể. Người có công đầu trong việc phát kiến trị số trung bình là Nhà khoa học người Bỉ Adolphe Quetelet [người sáng tạo ra chỉ số body mass index]. Vào năm 1831, Quetelet "sáng chế" ra cái mà ông gọi là "L homme Moyen" [người trung bình]. Người trung bình là một cá nhân hư cấu, với giá trị trung bình mà chúng ta có thể sử dụng để đại diện một nhóm người. Do đó, Quetelet tính chiều cao và trọng lượng trung bình của một nhóm lính Pháp, rồi xem đó là một người lính tiêu biểu. Nhưng Quetelet hiểu được rằng trị số trung bình sẽ dao động giữa các nhóm lính, và ông bàn về độ chính xác cũng như cách tính. Từ đó, khoa học thống kê có một giá trị mà sau này trở thành phổ biến nhất và được áp dụng trong hầu như bất cứ lĩnh vực xã hội nào. Cái bất ngờ mà tác giả Stigler chỉ ra rằng chúng ta có cái giá trị tiêu biểu bằng cách loại bỏ dữ liệu! Trụ cột - information: Qui luật giảm lượng thông tin Giả dụ như nếu chúng ta ước tính số trung bình quần thể dựa trên 100 đối tượng [và gọi là x1], và số trung bình dựa trên 00 đối tượng [x], câu hỏi đặt ra là giá trị của thông tin trong x cao gấp lần so với giá trị thông tin trong x1? Câu trả lời là không. Trong thực tế, nếu chúng ta tăng lượng dữ liệu gấp lần thì giá trị thông tin chỉ tăng khoảng 1.4 lần. Nếu chúng ta tăng lượng dữ liệu gấp 3 lần thì lượng thông tin chỉ tăng 1.7 lần. Từ đâu mà có các con số đó? Tác giả chỉ ra một sự thật hiển nhiên từ công thức tính sai số chuẩn [standard error]. Sai số chuẩn bằng độ lệch chuẩn chia cho căn số bậc của số cỡ mẫu; hay nói cách khác, độ lệch chuẩn bằng sai số chuẩn nhân cho căn số bậc của lượng dữ liệu. Chẳng có gì mới ở đây, vì De Moivre đã chỉ ra từ 1738, và đó cũng chính là lí thuyết đằng sau Định lí giới hạn trung tâm [Central Limit Theorem]. Nhưng cái hay ở đây là tác giả Stigler đã đưa ra một cách diễn giải rất có ý nghĩa trong bối cảnh Dữ liệu Lớn [Big Data] đang rất thịnh hành như là một xu hướng thống kê "thời thượng". Những 94

người tin vào Dữ liệu Lớn nghĩ rằng bằng cách tăng lượng dữ liệu thì chúng ta sẽ có thông tin chính xác hơn, đáng tin cậy hơn. Đúng nhưng chưa đủ, bởi vì lượng thông tin không phải là hàm số tuyến tính của lượng dữ liệu. Trụ cột 3 - likelihood: Thu nạp thông tin từ tình trạng bất định Trong chương này, tác giả Stigler bàn về lí thuyết khả dĩ [Likelihood]. Trong phần này, tác giả Stigler lí giải rằng chúng ta thu nạp thông tin từ dữ liệu qua các phương pháp như kiểm định thống kê [test of significance] và trị số P mà Ronald Fisher đề xướng từ năm 195, cùng với những phương pháp sau này như khoảng tin cậy 95%. Các phương pháp này cũng đã giúp chúng ta giảm sự bất định trong cuộc sống. Kiểm định thống kê mà Ronald Fisher đề xướng không phải là ý tưởng mới, bởi vì phương cách này đã được John Arbuthnot áp dụng trước đó để tính toán xem hiện tượng sinh con trai nhiều hơn con gái là do ý của Thượng đế hay ngẫu nhiên! Trụ cột 4 - intercomparison: So sánh Bất cứ ai làm nghiên cứu khoa học cũng cần so sánh. Thường là so sánh hai nhóm xem có khác nhau một cách có hệ thống hay khác biệt chỉ là do yếu tố ngẫu nhiên. Phương pháp kiểm định t [do William Gosset đề xướng] là một phương pháp quen thuộc. Một phương pháp so sánh khác cũng hay được áp dụng là phân tích phương sai [ANOVA hay analysis of variance] do Ronald Fisher phát kiến. Trong khi các ngành khác, người ta so sánh với một chuẩn vàng [gold standard], thì khoa học thống kê so sánh thông tin trong cùng một dữ liệu, một nghiên cứu. Những ai đọc sách sử thống kê học đều biết rằng William Gosset từng làm việc cho hãng bia Guiness, và công việc của ông lúc đó là kiểm nghiệm chất lượng bia. Trong nhiệm vụ đó, ông phải làm nghiên cứu nhưng thường dựa trên số mẫu nhỏ, và "cái khó ló cái khôn", ông đã sáng chế ra phương pháp so sánh khác biệt dựa vào cỡ mẫu nhỏ. Trong một dịp nghỉ hè [sabbatical] ông thăm labo của Karl Pearson tại University College London, và viết bài báo nổi tiếng. Khi bài báo được gửi cho tập san thống kê học, ông không được kí tên thật [vì là nhân viên của Guiness], nên phải kí dưới bút danh là "Student". Từ đó, khoa học có phương pháp kiểm định gọi là "Student s test". Trụ cột 5 - regression: Thu nạp thông tin từ qui luật hồi qui về số trung bình Một trong những phương pháp để chúng ta thu nạp thông tin rất hữu hiệu là mô hình hồi qui tuyến tính [linear regression model]. Mô hình này là một triển khai từ phương pháp phân tích tương quan [correlation analysis]. Phân tích tương quan là phương pháp do Nhà nhân chủng học 95

trứ danh Francis Galton đề xướng từ cuối thế kỉ 19. Lúc đó, Galton đang nghiên cứu về ảnh hưởng của di truyền đến trí thông minh, và ông dùng chiều cao là một marker. Ông quan sát rằng những cặp cha mẹ có chiều cao thấp hơn trung bình thường sinh con có chiều cao cao hơn cha mẹ; ngược lại, những cặp vợ chồng có chiều cao cao thường sinh con có chiều cao thấp hơn họ. Đây là hiện tượng hồi qui về số trung bình, hay thuật ngữ tiếng Anh là "regression to the mean". Đây là một trong những chương hay nhất của cuốn sách. Tác giả bắt đầu với câu chuyện về Charles Darwin, một người em họ của Francis Galton. Darwin là người không thích toán, không phải ông kém khả năng về toán, mà ông cho rằng toán không giúp gì cho khoa học! Darwin đề ra ý tưởng gọi là "The Rule of Three" hay "Qui luật tam suất". Ông nói nếu chúng ta biết rằng a/b = c/d và nếu chúng ta biết 3 số, thì chúng ta có thể xác định được số thứ 4. Nhưng trong thực tế, chúng ta cần nhiều tập hợp 4 giá trị để ước tính tham số của mô hình hồi qui tuyến tính. Nhưng ý tưởng về phân loại chủng vật của Darwin sau này lại nhờ các phương pháp phân tích đa biến giúp đỡ rất nhiều. Tất cả các phương pháp phân tích đa biến đều xuất phát từ mô hình hồi qui tuyến tính. Trụ cột 6 - design: Thu nạp thông tin từ số ngẫu nhiên Ngạc nhiên thay, chúng ta thu nạp thông tin cơ chế... ngẫu nhiên hoá. Ý nghĩa của trụ cột này cần một vài lời giải thích. Thiết kế là một phương pháp thu nạp thông tin có hệ thống. Nhưng thiết kế là phải dùng đến cơ chế ngẫu nhiên hoá [randomization]. Chẳng hạn nếu chúng ta muốn đánh giá hiệu quả của một thuốc điều trị bệnh, chúng ta phải chia nhóm bệnh nhân một cách ngẫu nhiên để đảm bảo các nhóm đều có những yếu tố nhiễu giống nhau. Thật ra, có thể xem thiết kế nghiên cứu là một cách tối ưu hoá. Trong chương này, tác giả Stigler dìu dắt chúng ta qua lịch sử của thiết kế nghiên cứu hết sức thú vị. Ý tưởng thiết kế nghiên cứu để thu nạp thông tin xuất phát từ sổ số bên Pháp vào năm 1757. Sổ số lúc đó đóng góp 4% cho ngân sách của Pháp [có lẽ giống như sổ số tràn lan hiện nay ở Việt Nam]. Sau đó khi Ronald Fisher làm việc cho Trạm thí nghiệm Rothamsted, ông mới nghĩ ra phương pháp chia nhóm ngẫu nhiên [randomization]. Trong một bài diễn thuyết trong Hội nghị Thống kê học Ấn Độ năm 1938, Fisher tuyên bố rằng "To consult the statistician after an experiment is finished is often merely to ask him to conduct a post mortem examination. He can perhaps say what the experiment died of" [Tư vấn nhà thống kê học sau khi thí nghiệm đã làm xong có thể ví von như là hỏi nhà thống kê học làm một cuộc giảo nghiệm tử thi. Nhà thống kê học có thể nói cái thí nghiệm chết vì lí do gì.] Ý của Fisher là muốn thí nghiệm có kết quả tốt và muốn thu nạp thông tin đáng tin cậy thì phải tư vấn nhà thống kê học trước khi làm thí nghiệm một lời khuyên vẫn còn ý nghĩa thời sự ngày hôm nay. Nhưng việc này đòi hỏi nhà thống kê phải hiểu vấn đề khoa học và hiểu qui trình suy luận khoa học. Trụ cột 7 - residual: Thu nạp thông tin từ... sai số Khi nói "sai số" ở đây, tôi muốn nói đến dao động dư, tức "residuals" hay "error terms" trong mô hình hồi qui tuyến tính. Trong khoa học, thỉnh thoảng chúng ta phát hiện những cái bình 96

thường từ những dữ liệu bất bình thường. Cái bất bình thường ở đây chính là residuals, là sai số từ mô hình. Chẳng hạn như để phát hiện những gen có liên quan đến bệnh lí, chúng ta sẽ mô hình phân bố của gen dựa vào một qui luật sinh học, và các dữ liệu nằm ngoài hay lệch so với giá trị kì vọng chính là những gen đáng quan tâm. Như vậy, chúng ta phát hiện cái cơ chế sinh học từ những dữ liệu và thông tin bất thường. Điều này có ý nghĩa rất quan trọng cho nhà thống kê học. Đối với những nhà thống kê học được huấn luyện trong các đại học mà không có tương tác với khoa học, phân tích dao động dư là để kiểm định tính hợp lí của mô hình của họ. Nhưng đối với khoa học, điều đó chẳng quan trọng; điều quan trọng là những dữ liệu mà mô hình không giải thích được. Cuốn sách được viết với văn phong khoa học nhưng vẫn hấp dẫn với người ngoài khoa học. Nhưng thỉnh thoảng, tác giả có vẻ giả định người đọc phải hiểu một số khái niệm thống kê học. Chẳng hạn như người đọc phải "động não" để hiểu L[Θ] = L[Θ] X and Cov[L, W ]= E{Cov[L, W S]} + Cov[E{L S},E{W S}] Nhưng may mắn thay, nhưng công thức loại này chỉ xuất một vài lần trong sách, và người đọc không cần hiểu chúng mà vẫn nắm được ý nghĩa đằng sau các mô hình thống kê. Tóm lại, cuốn sách "The Seven Pillars of Statistical Wisdom" là một tác phẩm rất hay và đáng đọc. Tác phẩm này hay là vì tác giả đã đem đến cho chúng ta những cái nhìn rất tươi, với những lí giải rất mới về những phương pháp thống kê cổ điển. Chỉ trong 00 trang sách mà tác giả đã lược qua những điểm chính [7 điểm] trong suốt chiều dài lịch sử của chuyên ngành khoa học thống kê. Cuốn sách còn giúp cho chúng ta, những người làm nghiên cứu thực nghiệm, hiểu sâu hơn về các phương pháp thống kê, và qua đó giúp chúng ta suy nghĩ một cách thống kê, chứ không phải suy nghĩ như là một công cụ. Xin nhấn mạnh: tôi xem thống kê học là một cách suy nghĩ. Thay vì tập trung vào những chi tiết tính toán, chúng ta cần phải hiểu ý nghĩa đằng sau của các phương pháp và mô hình thống kê. Nếu bạn là nhà thống kê học, nhà khoa học thực nghiệm, cuốn sách "The Seven Pillars of Statistical Wisdom" phải có trong tủ sách của các bạn. Ghi thêm: [1] Stephen M. Stigler là giáo sư xuất sắc chuyên ngành sử thống kê thuộc Đại học Chicago. Ông tốt nghiệp tiến sĩ thống kê học từ Đại học California, Berkeley, nhưng sau đó ông chuyển về Đại học Wisconsin, Madison, và năm 1979 thì chuyển sang Đại học Chicago cho đến nay. Ông công bố nhiều công trình nghiên cứu về sử thống kê rất có giá trị. Những công trình này được đúc kết thành cuốn sách "The History of Statistics" [1986] và "Statistics on the Table" [1999]. Ngoài vai trò là sử gia của khoa học thống kê, ông còn biên tập phần lí thuyết cho tập san Journal of the American Statistical Association [1979-198]. Với những thành tích đó, Stigler là người có thẩm quyền để viết về những ý nghĩa đằng sau 7 wisdom thống kê mà tôi đang giới thiệu đến các bạn. 97

MỘT SỐ BÀI TOÁN TRÊN TÂM ĐƯỜNG TRÒN EULER Trần Quang Hùng GIỚI THIỆU Bài viết xoay quanh một số bài toán liên quan tới tâm đường tròn Euler. Các bài toán này hầu như đều là kết quả của tác giả trong quá trình đi tập huấn một số đội tuyển thi HSG trên cả nước. Thời gian gần đây, nhiều bài toán rất thú vị trên tâm đường tròn Euler và đường tròn Euler liên tục được phát hiện, trong đó bản thân tác giả cũng tìm ra được một số bài toán mới về chủ đề này. Mặt khác quãng thời gian tác giả giảng dạy những bài toán này cách thời điểm hiện tại cũng đã lâu, tuy nhiên một số bài toán vẫn có nhiều giá trị. Đặc biệt là các bài toán đó vẫn được sử dụng khá tốt trong việc bồi dưỡng các em học sinh yêu hình học. Do đó tôi xin trân trọng giới thiệu với bạn đọc bài viết tổng hợp này. 1. Một số bài toán ví dụ Đường tròn Euler hay tên quốc tế thường gọi là đường tròn 9 điểm [7], là đường tròn đi qua trung điểm ba cạnh, chân ba đường cao và trung điểm ba đoạn thẳng nối trực tâm và ba đỉnh tam giác. Đây là một kết quả rất nổi tiếng và được khai thác trong rất nhiều bài toán khác nhau. Tâm đường tròn này cũng là một đề tài thú vị của các bài nghiên cứu hình học sơ cấp và cũng là đề tài hay dành cho các cuộc thi học sinh giỏi toán trong nước và quốc tế. Phần này Tôi xin trình bày lại một số bài toán chủ yếu do tôi đề xuất liên quan đến tâm đường tròn thú vị này trong khoảng thời gian từ năm 013-014. Kỳ thi học sinh giỏi toán quốc gia năm 013 có bài toán như sau [1]. Bài toán 1 [VMO 013]. Cho tam giác ABC, đường tròn nội tiếp [I] tiếp xúc CA, AB lần lượt tại E,F. G, H lần lượt là đối xứng của E,F qua I. Đường thẳng GH giao IB,IC lần lượt tại P, Q. Giả sử B,C cố định, A thay đổi sao cho tỷ số AB = k không đổi. Chứng minh AC rằng trung trực PQluôn đi qua một điểm cố định. Lời giải. Gọi IB,IC lần lượt cắt EF tại K, L. Chú ý tam giác AEF cân tại A nên KEC = AEF = 180 A =180 [90 A ]=180 BIC = KIC. Từ đó tứ giác KEIC nội tiếp suy ra IKC = IEC =90. Tương tự ILB 90. Từ đó nếu gọi M là trung điểm BC, J là trung điểm KL đễ có tam tam giác KLM cân nên MJ EF [1]. Do G, H lần lượt là đối xứng của E,F qua I nên đường thẳng GH đối xứng đường thẳng EF qua I. GH, EF lần lượt cắt IB tại P, K suy ra I là trung điểm PK, tương tự I là trung điểm 98

A L F I J E K P G B R N D Q M H C QL. Vậy hai đoạn KL và PQđối xứng nhau qua I. Từ đó nếu gọi R là trung điểm PQthì trung điểm J của KL và R đối xứng nhau qua I hay I là trung điểm RJ. Gọi trung trực PQcắt BC tại N, ta thấy RN vuông góc PQ, PQsong song EF []. Từ [1] và [] suy ra RN song song JM. Gọi IA cắt BC tại D, dễ có ID IA vuông góc EF nên ID cũng song song với RN, JM. Từ đó trong hình thang RJMN có I là trung điểm RJ nên ID là đường trung bình, vậy D là trung điểm MN. Theo tính chất đường phân giác DB = AB = k không đổi nên D cố định. M là trung điểm BC DC AC cố định nên N đối xứng M qua D cố định. Vậy trung trực PQđi qua N cố định. Nhận xét. Việc dựng ra thêm các điểm phụ L, K trong lời giải đóng vai trò quan trọng, nó cho phép ta sử dụng phép vị tự để chuyển các tính chất đường thẳng RN và JM cho nhau. Trong quá trình tìm hiểu bài toán tôi nhận thấy rằng thực chất K, L, N là các chân đường cao từ C, B, I của tam giác IBC. Như vậy đường tròn ngoại tiếp tam giác NKL là đường tròn Euler của tam giác IBC nên nó cũng đi qua M và vì vậy tâm đường tròn Euler của tam giác IBC nằm trên trung trực KL cũng là đường thẳng JM. Vậy nếu từ tâm đường tròn Euler của tam giác IBC mà ta vẽ đường thẳng song song với IA thì nó cũng chính là trung trực KL mà đi qua trung điểm BC. Ta dễ thấy là các đường thẳng qua trung điểm BC, CA, AB mà lần lượt song song với IA, IB, IC thì đồng quy. Do đó ta đề xuất bài toán như sau Bài toán. Cho tam giác ABC tâm nội tiếp I. Gọi N a,n b,n c lần lượt là tâm đường tròn Euler của các tam giác IBC, ICA, IAB thì các đường thẳng qua N a,n b,n c lần lượt song song với IA, IB, IC đồng quy. Qua tìm hiểu và khai thác tôi nhận ra rằng bài toán này đúng không chỉ với tâm nội tiếp I mà thực chất nó đúng với mọi điểm P bất kỳ trong mặt phẳng. Do đó tôi đề xuất bài toán sau Bài toán 3. Cho tam giác ABC và P là một điểm bất kỳ. Gọi N a,n b,n c lần lượt là tâm đường tròn Euler của các tam giác P BC, P CA, P AB. Chứng minh rằng các đường thẳng qua N a,n b,n c lần lượt song song với P A, P B, P C đồng quy. 99

A O c O b P N a N G K B G a M a C O a Lời giải. Gọi O a,o b,o c lần lượt là tâm đường tròn ngoại tiếp các tam giác P BC, P CA, P AB. Ta dễ thấy đường thẳng qua O a song song PA chính là đường cao từ O a của tam giác O a O b O c do đó các đường thẳng qua O a,o b,o c lần lượt song song với P A, P B, P C đồng quy tại trực tâm K của tam giác O a O b O c. Gọi G a,g b,g c,glần lượt là trọng tâm tam giác P BC, P CA, P AB, ABC ta dễ thấy PG a và AG đi qua trung điểm M a của BC, từ đó dễ thấy G a G song song PAnói cách khác các đường thẳng qua G a,g b,g c lần lượt song song với P A, P B, P C đồng quy tại trọng tâm G của tam giác ABC. Đến đây là lại chú ý vì N a là tâm đường tròn Euler của tam giác PBC nên dễ có G a N a + G a O a = 0. Từ đó sử dụng phép chiếu song song phương PA xuống đường thẳng KG, gọi N là hình chiếu song song phương PAcủa N a xuống KG ta dễ suy ra GN + GK = 0 nói cách khác đường thẳng qua N a song song PA đi qua N xác định. Tương tự các đường thẳng qua N b,n c lần lượt song song với PB,PC cũng đi qua N ta có điều phải chứng minh. Nhận xét. Bài toán trong trường hợp tâm nội tiếp là bài toán có ý nghĩa, việc tổng quát bài toán với điểm P bất kỳ rồi sau đó đặc biệt hóa cho P di chuyển trùng một số tâm đặc biệt để tạo ra bài toán mới là việc làm thú vị. Qua bài toán này ta cũng dễ rút ra tâm đường tròn Euler cũng chỉ là một trường hợp đặc biệt. Tương tư như cách chứng minh trên thì bài toán cũng sẽ đúng với trực tâm hoặc tổng quát hơn là một điểm trên đường thẳng Euler chia đoạn nối trực tâm, trọng tâm tỷ số cố định. Ta có các bài toán khác như sau Bài toán 4. Cho tam giác ABC và P là một điểm bất kỳ. Gọi H a,h b,h c lần lượt là trực tâm của các tam giác P BC, P CA, P AB. Chứng minh rằng các đường thẳng qua H a,h b,h c lần lượt song song với P A, P B, P C đồng quy. Bài toán 5. Cho tam giác ABC và P là một điểm bất kỳ. Gọi H a,h b,h c lần lượt là trực tâm của các tam giác P BC, P CA, P AB. Gọi G a,g b,g c lần lượt là trọng tâm các tam giác P BC, P CA, P AB. Gọi L a,l b,l c lần lượt chia các đoạn H a G a,h b G b,h c G c cùng một tỷ số. Chứng minh rằng các đường thẳng qua L a,l b,l c lần lượt song song với P A, P B, P C đồng quy. 100

Một khai thác tương tự được tác giả đề nghị trong cuộc thi giải toán mathley [3], các bạn hãy làm như bài luyện tập Bài toán 6. Cho tam giác ABC và DEF nội tiếp đường tròn [O]. Gọi N a,n b,n c là tâm đường tròn Euler các tam giác DBC, ECA, F AB. Chứng minh rằng các đường thẳng qua N a,n b,n c lần lượt song song AD, BE, CF đồng quy. Sau khi đề xuất bài toán 3, tôi đã mạnh dạn nghĩ tới kết quả thay đường thẳng song song bởi đường thẳng vuông góc và thật tuyệt vời khi bài toán vuông góc đúng, bài toán phát biểu như sau Bài toán 7 [Kiểm tra đội tuyển THPT chuyên KHTN năm 014]. Cho tam giác ABC và P là một điểm bất kỳ. Gọi N a,n b,n c lần lượt là tâm đường tròn Euler của các tam giác P BC, P CA, P AB. Chứng minh rằng các đường thẳng qua N a,n b,n c lần lượt vuông góc với P A, P B, P C đồng quy. Bài toán trên theo đánh giá của tôi là bài toán hay. Tôi đã sử sụng bài toán này trong đợt kiểm tra đội tuyển VMO của trường THPT chuyên KHTN thật đáng tiếc là không có em nào giải được bài toán này. Lời giải đầu tiên tôi có được cho bài toán này chưa đẹp, sau đây tôi trình bày một lời giải thứ hai tham khảo ý tưởng từ học trò Tạ Hà Nguyên, học sinh lớp 1A1 Toán, khóa 48, trường THPT chuyên KHTN. Trong suốt bài toán này ta ký hiệu [XY Z] chỉ đường tròn ngoại tiếp tam giác XY Z. Công cụ sử dụng trong phần này là góc định hướng giữa hai đường thẳng. A F S X P E B Y D Z C Lời giải. Gọi D, E, F, X, Y, Z lần lượt là trung điểm của BC, CA, AB, P A, P B, P C, thì các đường tròn [DY Z], [EZX], [FXY], [DEF] lần lượt là các đường tròn Euler của các tam giác P BC, P CA, P AB, ABC. Gọi đường tròn [DY Z] và đường tròn [DEF] cắt nhau tại S khác D. Ta có biến đổi góc có hướng như sau [SF,SY ]=[SF,SD]+[SD,SY ] =[EF,ED]+[ZD,ZY ] [Do S thuộc các đường tròn [DY Z], [DEF]] =[BD,BF]+[BY,BD] [Do EF BD,ED AC, Y Z BD,ZD BY ] =[BY,BF] 101

=[XF,XY ] [Do XY BF,XF BY ]. Do đó S thuộc [FXY]. Tương tự S thuộc [EZX]. Ta có điều phải chứng minh. Theo bài toán 3 thì các đường thẳng qua N a,n b,n c lần lượt song song P A, P B, P C đồng quy tại Q. Ta chỉ cần chứng minh rằng Q nằm trên đường tròn [N a N b N c ] thì hiển nhiên đường thẳng qua N a,n b,n c lần lượt vuông góc với P A, P B, P C sẽ đồng quy tại điểm đối tâm Q. Vậy ta biến đổi góc [QN b,qn c ]=[PB,PC]=[DZ, DY ]=[SZ,SY ]=[N a N b,n a N c ]. Ta chú ý đẳng thức cuối có là do đường tròn [N a ] và [N b ] có dây cung chung là SZ nên N a N b SZ, tương tự N a N c SY. Từ đó ta có điều phải chứng minh. Nhận xét. Bài toán nếu cho P trùng với một số điểm đặc biệt cũng sẽ dẫn tới nhiều hệ quả quan trọng. Ngoài ra khi khai thác các bài toán xoay quanh tâm đường tròn Euler, chúng ta cũng nhận được nhiều bài toán lạ mắt. Tôi xin giới thiệu một bài toán mà tôi cũng tình cờ tìm ra khi giải một bài toán liên quan tới vấn đề về điểm cố định và tâm đường tròn Euler. Bài toán 8. Cho lục giác ABCDEF nội tiếp đường tròn [O]. Gọi K, L, N lần lượt là tâm đường tròn Euler của các tam giác DEC, BCA, F AE. Gọi X, Y, Z lần lượt là hình chiếu của K, L, N theo thứ tự lên AD, BE, CF. Chứng minh rằng trung trực của AX, EY, CZ đồng quy. Trước hết ta có bổ đề sau Bổ đề. Cho tam giác ABC nội tiếp đường tròn [O]. P là một điểm trên [O]. K là tâm đường tròn Euler của tam giác PBC. a] Chứng minh rằng đường thẳng qua K vuông góc với PA luôn đi qua điểm cố định khi P di chuyển. b] Gọi H là hình chiếu của K lên PA. Chứng minh rằng trung trực của AH luôn đi qua điểm cố định khi P di chuyển. A M H P N L O K B C I 10

Chứng minh. a] Gọi N là tâm đường tròn Euler của tam giác ABC. Gọi L là trung điểm của ON. Gọi I đối xứng A qua L, gọi M là trung điểm PA. Ta đã biết kết quả quen thuộc KN = 1 PA = MA suy ra AN = MK. Do I đối xứng A qua L nên AN = OI. Vậy từ đó OM = KI suy ra KI OM PA. Vậy đường thẳng qua K vuông góc PAđi qua I cố định. Ta có điều phải chứng minh. b] Dễ thấy trung trực của AH đi qua trung điểm L của AI cũng là trung điểm ON cố định. Ta có điều phải chứng minh. A N Z F Y B L T O E D X K C Lời giải bài toán. Bài toán là hệ quả của bổ đề trên ta dễ thấy trung trực các đoạn AX, EY, CZ đồng quy tại trung điểm OT với T là tâm đường tròn Euler của tam giác AEC. Ta có điều phải chứng minh. Nhận xét. Bổ đề là một bài toán đi qua điểm cố định rất quan trọng. Chúng ta hoàn toàn có thể dựa vào bài toán đi qua điểm cố định để đề xuất thành các bài toán chứng minh đồng quy như đã làm trong bài toán trên. Bài toán trên đã từng được tác giả đề nghị trong kỳ thi chọn đội tuyển thi học sinh giỏi quốc gia trường THPT chuyên KHTN năm 013 xem [4]. Sau đó bài toán này cũng xuất hiện trong [5] và không ghi nguồn gốc, nhân dịp bài viết này, tôi xin giới thiệu lại rõ nguồn gốc với bạn đọc như trên. Sau đây là một bài toán tác giả cũng tình cờ tìm ra. Bài toán này được tác giả tạo ra khi đang tìm hiểu về các hệ thức trong hàng điểm điều hòa, bắt nguồn từ bài giảng cho lớp học toán online của EgoGreen. Bài toán này cũng đã xuất hiện trong [6] và không ghi nguồn gốc, tôi xin giới thiệu lại rõ nguồn gốc với bạn đọc như trên. Bài toán 9. Cho tam giác ABC đường cao BE,CF và đường tròn Euler là [O 9 ], D, G thuộc BC sao cho [BC,DG] = 1. ED,FD lần lượt cắt [O 9 ] lần lượt tại M,N khác E,F. GE, GF cắt [O 9 ] lần lượt tại P, Q khác E,F. PM giao NQ tại R. Gọi S là đối xứng của 103

G qua trung điểm PQ. Gọi T là đối xứng của D qua trung điểm MN. Chứng minh rằng R, S, T thẳng hàng. A S E X P F O 9 Q G Y B M D R N T I C 0 Lời giải. Gọi I là trung điểm của BC theo hệ thức Newton ta dễ có IE = IF = IB = IC = ID.IG. Từ đó dễ có IFD = IGF, IED = IGE. Gọi PS cắt [O 9 ] tại X. Suy ra EPX = EGF = IGE IGF = IED IFD = IED IEN = MEN. Từ đó dễ suy ra EM NX suy ra NT [ đi qua X. ] Tương tự gọi SQ cắt [O 9 ] tại Y thì MT đi qua Y. Áp PNY dụng định lý Pascal cho bộ ta thu được R, S, T thẳng hàng. Ta có điều phải chứng QMX minh. Sau đây là ba bài toán hay liên quan tới tâm đường tròn Euler và tâm đường tròn nội tiếp cũng do tôi đề xuất. Các bài toán này đã được hoàn thiện lời giải bởi một học trò xuất sắc của tôi là Nguyễn Ngọc Chi Lan học sinh lớp 1A1 Toán, khóa 48, trường THPT chuyên KHTN. Bài toán 10. Cho tam giác ABC có tâm đường tròn nội tiếp là I. D, E, F là hình chiếu của A lên BC,IC,IB. Gọi K là tâm ngoại tiếp tam giác DEF. Chứng minh rằng KI đi qua tâm đường tròn Euler của tam giác IBC. A H Y E Z K I F J P M B D X N C 104

Lời giải. Gọi M,N lần lượt là giao điểm AE, AF với BC. Dễ dàng chứng minh E,F lần lượt là trung điểm AM, AN. Suy ra K là tâm ngoại tiếp tam giác DEF cũng là tâm đường tròn Euler của tam giác AMN. Cũng chứng minh được I là tâm ngoại tiếp tam giác AMN. Gọi IX,BY,CZ lần lượt là các đường cao của tam giác IBC và H là trực tâm tam giác. Dễ thấy AM HB, AN HC. Gọi P là giao điểm AH, BC. Phép vị tự tâm P biến tam giác AMN thành tam giác HBC. Lại có IK là đường thằng Euler của tam giác AMN. Gọi J là tâm đường tròn Euler của tam giác HBC, khi đó IJ là đường thẳng Euler của tam giác HBC.VậyI,J,K thẳng hàng. J là tâm ngoại tiếp [XY Z], suy ra J cũng là tâm Euler của tam giác IBC. Suy ra điều phải chứng minh. Bài toán 11. Cho tam giác ABC có tâm đường tròn nội tiếp là I. D, E, F là hình chiếu của A lên BC,IC,IB. Gọi AI cắt đường tròn ngoại tiếp [O] của tam giác ABC tại G khác A. Chứng minh rằng đường thẳng Euler của tam giác DEF đi qua trung điểm AG. A F H K I O E N B D P Q C M J G L Lời giải. Gọi M,N lần lượt là giao điểm AE, AF với BC. H, K lần lượt là trực tâm và tâm ngoại tiếp tam giác DEF. AH cắt [AMN] tại L. J là đối xứng của A qua K và P là hình chiếu của I lên BC. Ta có các kết quả quen thuộc: L là đối xứng của A qua H, I là tâm ngoại tiếp tam giác AMN và J là đối xứng của I qua BC. Như vậy, dễ thấy HK JL hay JL song song với đường thẳng Euler của tam giác DEF. Do F AH = FCD = 1 ABC = ACF = ADF = FEH suy ra A, E, F, H cùng thuộc một đường tròn. Lại có AE IE, AF IF suy ra A, E, F, I cùng thuộc một đường tròn, suy ra H thuộc đường tròn đường kính AI, suy ra IH AD suy ra IHDP là hình chữ nhật. Ta chứng minh H, P, G thẳng hàng. Gọi Q là giao điểm AG, BC. Lại có G là tâm ngoại tiếp tam giác IBC nên dễ có QG.QA = QI[QG + GI] suy ra QG.IA = QI.GI suy ra QG = IQ PQ suy ra = PQ = QG, suy ra H, P, G thẳng hàng. IG IA PD HI IG Suy ra L, J, G thẳng hàng. Suy ra HK LG suy ra HK đi qua trung điểm AG. Bài toán 1. Cho tam giác ABC nội tiếp đường tròn [O]. P di chuyển trên [O]. Đường thẳng qua P song song BC cắt CA tại E. Gọi K là tâm đường tròn ngoại tiếp tam giác PCE và L là tâm đường tròn Euler của tam giác PBC. Chứng minh rằng đường thẳng qua L song song PK luôn đi qua một điểm cố định khi P di chuyển. 105

A E P B H N O M K L C Q J G Lời giải. Gọi H, N lần lượt là trực tâm và tâm đường tròn Euler của tam giác ABC. Gọi M là trung điểm BC, vàg là đối xứng của P qua L. Kết quả quen thuộc, ta có G là đối xứng của O qua BC. Gọi Q là giao điểm AH với [O]. Do PE BC suy ra CEP = ACB, suy ra CPK =90 CEP =90 ACB = CAQ = CPQ suy ra P, K, Q thẳng hàng. Đường thẳng qua L song song PK cắt GQ tại J. Suy ra J là trung điểm LQ. Tứ giác OHQG là hình thang cân, NJ là đường trung bình của hình thang. Suy ra J đối xứng N qua BC. Suy ra đường thẳng qua L song song PK luôn đi qua điểm cố định là điểm đối xứng với tâm Euler của tam giác ABC qua BC. 106

. Một số bài toán luyện tập Các bài toán này [trừ bài toán cuối cùng] đều được tác giả sáng tác và đã gửi lên diễn đàn AoPS trong một số năm gần đây. Bài toán 13. Cho tam giác ABC nội tiếp [O] và có điểm Lemoine L. LA, LB, LC cắt lại [O] tại A, B, C. 1] Chứng minh rằng ABC và A B C có chung hai điểm Brocard Ω 1 và Ω. ] Gọi P là trung điểm của đoạn thẳng Ω 1 Ω. N và N là tâm đường tròn Euler của các tam giác ABC và A B C. Chứng minh rằng PN = PN. Bài toán 14. Cho tam giác ABC có trực tâm H, điểm Lemoine L và tâm đường tròn Euler N. Gọi A B C là tam giác anti-ceva của N. Chứng minh rằng tâm ngoại tiếp tam giác A B C nằm trên HL. B' C' A H L N O' B C A' Bài toán 15. Cho tam giác ABC có tâm ngoại tiếp O. và tâm đường tròn Euler là N. Tam giác Napoleon là DEF. 1] Chứng minh rằng đường thẳng Euler của các tam giác OEF, OFD, ODE đồng quy tại P. 107

] Chứng minh rằng AD, BE, CF đồng quy tại Q. 3] Chứng minh rằng đường thẳng PQchia đôi đoạn thẳng ON. Bài toán 16 [Tham khảo [9]]. Cho tam giác ABC có N, N a, N b,vàn c lần lượt là tâm đường tròn Euler của các tam giác ABC, NBC, NCA,vàNAB. Chứng minh rằng AN a, BN b,và CN c đồng quy. Tài liệu [1] T. Q. Hùng, Mở rộng bài toán hình học trong kỳ thi học sinh giỏi quốc gia năm 013, tạp chí toán học và tuổi trẻ số 49 tháng 3 năm 013. [] T. Q. Hùng, P. V. Thuận, Cuộc thi giải toán mathley, //www.hexagon.edu.vn/mathley.html [3] T. Q. Hùng, Tuyển tập các bài toán hình học chọn đội tuyển KHTN, năm 013. [4] T. Q. Hùng, Tỷ số kép, phép chiếu xuyên tâm, hàng điểm điều hòa, chùm điều hòa, //analgeomatica.blogspot.com/014/01/bai-giang-ty-sokep-phep-chieu-xuyen.html [5] DonaldLove [Nick name người Việt Nam], nice concurrent, //artofproblemsolving.com/community/c6h577336 [6] lambosama [Nick name người Việt Nam], Very hard concurrent problem, //artofproblemsolving.com/community/c6h560895 [7] E. W. Weisstein, Nine-Point Circle, from MathWorld A Wolfram Web Resource, //mathworld.wolfram.com/nine-pointcircle.html [8] T. Q. Hùng, Diễn đàn các bài toán hình học nằm trong AoPS, //artofproblemsolving.com/community/c374081 [9] C. Kimberling, Encyclopedia of Triangle Centers, //faculty.evansville.edu/ck6/encyclopedia/etc.html 108

ĐIỂM HUMPTY - DUMPTY TRONG TAM GIÁC VÀ ỨNG DỤNG Nguyễn Trường Sơn [GV THPT Chuyên Lương Văn Tụy, Ninh Bình] LỜI BAN BIÊN TẬP Gần đây, nhiều mô hình được khai thác để "tăng tốc" việc tiếp cận các bài toán hình học khó. Ngoài các mô hình kinh điển như tứ giác toàn phần, tứ giác điều hòa,... ta còn có các điểm đặc biệt với nhiều tính chất trong tam giác, tứ giác. Trong bài viết này, tác giả sẽ giới thiệu về hai điểm Humpty, Dumpty trong tam giác cùng với các ứng dụng của chúng để giải toán. 1. Điểm Humpty - Dumpty và một số tính chất liên quan. 1.1. Định nghĩa. 1. Điểm Humpty. Cho tam giác ABC. Điểm P A nằm trong tam giác ABC thỏa mãn P A CB = P A AC, P A BC = P A AB được gọi là điểm A Humpty. Định nghĩa tương tự với các điểm B Humpty và C Humpty. A P A O B C 109

. Điểm Dumpty. Cho tam giác ABC. Điểm Q A nằm trong tam giác ABC thỏa mãn Q A CA = Q A AB, Q A BA = Q A AC được gọi là điểm A Dumpty. Định nghĩa tương tự với các điểm B Dumpty và C Dumpty. A J I Q A O B C 1.. Tính chất. Tính chất 1. Cho tam giác ABC có trực tâm H. Giao điểm thứ hai của đường tròn đường kính AH và đường tròn ngoại tiếp tam giác BHC nằm trên đường trung tuyến AM. Giao điểm đó chính là điểm A Humpty. Chứng minh. Gọi A là điểm thỏa mãn ABA C là hình bình hành. Vì BAC = BA C = π BHC. Suy ra A thuộc đường tròn [BHC]. Vì ĤBA = ĤBC + Â BC = π ÂCB + ÂCB = π, vì vậy HA là đường kính của [BHC]. Khi đó: ÂP A H = π = π ĤP A A, suy ra P A nằm trên đường trung tuyến AM. A F H P A O E B D M C A 110

Nhận xét 1. Đường trung tuyến AM cắt đường tròn [BHC] tại điểm A khác điểm P A thì ABA C là hình bình hành. Tính chất. Gọi [I] là đường tròn qua A, B tiếp xúc với BC,[J] là đường tròn qua A, C và tiếp xúc với BC. Khi đó giao điểm khác A của hai đường tròn [I], [J] chính là điểm A Humpty. Chứng minh. Gọi P là giao điểm thứ hai của hai đường tròn [I], [J]. Vì ÂP B = π ĈBA và ÂP C = π ÂCB nên BPC =π ÂP B ÂP C = ÂCB + ĈBA = π BAC Suy ra P nằm trên đường tròn [BHC]. Vì vậy theo tính chất 1,P nằm trên đường trung tuyến của tam giác ABC. Suy ra P P A. Tính chất 3. Cho tam giác ABC nội tiếp [O].AD là đường đối trung của tam giác ABC, D thuộc [O]. Gọi P là trung điểm AD. Khi đó P, P A liên hợp đẳng giác. Chứng minh. Do AD là đường đối trung của tam giác ABC, D thuộc đường tròn [O] nên tứ giác ABDC là tứ giác điều hòa. Do đó BC là đường đối trung của tam giác BAD. Mặt khác, BP là đường trung tuyến của tam giác BDA. Do đó BC,BP đẳng giác. Suy ra PBA = ĈBD = DAC = P A AB = P A BC. Vậy BP,BP A là hai đường đẳng giác. Suy ra P, P A là hai điểm đẳng giác liên hợp. A H P P A O B M C D K 111

Tính chất 4. Cho tam giác ABC nội tiếp đường tròn [O].AD là đường đối trung của tam giác ABC, D thuộc đường tròn [O]. Khi đó D, P A đối xứng với nhau qua BC. Chứng minh. Bằng biến đổi góc, suy ra hai tam giác BP A C và BDC bằng nhau nên dễ dàng suy ra D, P A đối xứng nhau qua BC. Từ đây, ta còn suy ra kết quả: P A nằm trên đường tròn A Apollonius của tam giác ABC. Tính chất 5. Gọi [I,R 1 ] là đường tròn qua A, B tiếp xúc với BC,[J, R ] là đường tròn qua A, C và tiếp xúc với BC. Gọi R, R 3 lần lượt là bán kính đường tròn ngoại tiếp tam giác ABC, P A BC. Khi đó R.R 3 = R 1.R. Chứng minh. Theo tính chất 4, suy ra R = R 3. Gọi K là tâm đường tròn ngoại tiếp tam giác BP A C. Dễ thấy, IK,JK tương ứng là trung trực của BP A,CP A, ta có BJK = 1 BJP A = P A BC = 1 P A KC = ĈKI Tương tự, BKJ = ĈIK, do đó BJK CKI. Suy ra JB KC = KB IC R 3 = R 1 R R R 3 = R 1 R A I J P A B C K 11

Nhận xét. Từ chứng minh trên dẫn đến P A BC BJK P AB P A C = R 1. Tương tự, R 3 AB AC = R 1. Do đó phân giác trong của hai góc BAC, BP A C cắt nhau tại một điểm thuộc BC. R 3 Tính chất 6. Giả sử AD là đường đối trung của tam giác ABC, D BC. Đường thẳng qua D và vuông góc với BC cắt đường trung tuyến AM tại S. Đường thẳng qua S và song song với BC cắt AB, AC theo thứ tự tại F, E. Khi đó P A BE CF. Chứng minh. Ta sẽ chứng minh bài toán sau: Cho tam giác ABC, P A là điểm A Humpty và M là trung điểm của đoạn BC. Gọi E = BP A AC, F = CP A AB, S = EF AM. Gọi D là hình chiếu của S lên BC. Khi đó AD là đường đối trung của tam giác ABC. Thật vậy, ta có: MB = MA MP A = MC MB MP A = MA MB, MC MP A = MA MC. Suy ra các cặp tam giác [MBP A, MAB], [MCP A, MAC] đồng dạng. Do đó: BP A AB = MB MA, CP A AC = MC MA nên BP A AB = MC [1]. MA Lấy điểm L trên AD sao cho DS P A L. Từ đó, chú ý rằng DA, DP A,DS,DM là chùm điều hòa, suy ra DM đi qua trung điểm của LP A. A F O P A S K H E B M D C L Từ đó, chú ý rằng LP A DM, suy ra L, P A đối xứng nhau qua BC. []. Từ [1], [] suy ra LB AB = LC CA. Hơn nữa, BAC + BLC = BAC + BP A C = π Do đó, tứ giác ABLC nội tiếp, và nó cũng điều hòa. Do đó AD là đường đối trung của tam giác ABC. Tính chất 7. Trực tâm K của tam giác DEF, P A và tâm đường tròn ngoại tiếp O tam giác ABC thẳng hàng. 113

Chứng minh. Từ chứng minh ở tính chất 6, ta có DP A C = DLC = FBC, F AE + FP A E = π. Do đó các tứ giác FP A EA, BF P A D nội tiếp. Vì FP A EA nội tiếp và BC tiếp xúc với đường tròn [AP A B] nên P A EF = P A AF = P A BC. Suy ra BC EF. Vì các tứ giác FP A EA, BF P A D nội tiếp và O là tâm đường tròn ngoại tiếp tam giác ABC nên FDB = BP A F = BAC = 1 = BOC π ÔBC. Suy ra BO FD. Từ đó, chú ý rằng, EK FD,suy ra BO EK. Tương tự, CO FK. Từ đó, ta suy ra được OK, BE, CF đồng quy tại P A. Điều đó có nghĩa là K, P A,O thẳng hàng. Ở trên là khá nhiều tính chất [từ dễ đến khó] của điểm Humpty. Sau đây, chúng tôi sẽ liệt kê một số tính chất của điểm Dumpty, bạn đọc thử tự đưa ra chứng minh, xem như bài tập. Tính chất 8. Cho tam giác ABC có Q A là điểm A Dumpty. Khi đó: 1. Q A nằm trên đường A đối trung của tam giác ABC.. Q A là tâm vị tự quay biến AQ A C thành CQ A B và biến AC thành BA. 3. Bốn điểm B,Q A,O,Cđồng viên trong đó O là tâm đường tròn ngoại tiếp tam giác ABC. 4. OQ A AQ A. 5. Giao điểm của đường tròn đi qua A, B và tiếp xúc với AC và đường tròn đi qua A, C tiếp xúc với AB chính là điểm A Dumpty.. Bài tập áp dụng. Bài toán 1. [ELMO shortlist 013] Cho tam giác ABC. Gọi D là điểm di động trên đường thẳng BC. Đường tròn ngoại tiếp tam giác ABD cắt AC tại điểm thứ hai E, đường tròn ngoại tiếp tam giác ADC cắt AB tại điểm thứ hai F. Khi đó đường tròn ngoại tiếp tam giác AEF luôn đi qua một điểm cố định khác A, khi D di động. Lời giải. Cách 1. Gọi K là giao điểm của BE và CF. Áp dụng định lí Miquel cho tứ giác toàn phần AEKF BC ta có D thuộc đường tròn ngoại tiếp tam giác BFK. Do đó: [KF,KB] [DF, DB] [AF, AC] [modπ]. Vậy tứ giác AEKF nội tiếp đường tròn, suy ra [KF,KE] [AF, AE] [AB, AC] [P A B,P A C] [mod π]. 114

Vậy tứ giác BKP A C nội tiếp đường tròn, suy ra [KP A,KE] [CP A,CB] [AP A, AC] [AP A, AE] [mod π]. Vậy tứ giác AKP A E nội tiếp đường tròn, hay đường tròn ngoại tiếp tam giác AEF luôn đi qua điểm A Humpty. A E F K P A B D C Cách. Ta phát biểu không chứng minh bổ đề sau: Cho tam giác ABC có M là trung điểm BC.CQ,BP là các đường cao của tam giác ABC. Khi đó MP,MQ và đường thẳng qua A song song với BC là các tiếp tuyến của đường tròn ngoại tiếp tam giác AP Q. Trở lại bài toán, gọi H là trực tâm của tam giác ABC.P, Q theo thứ tự là hình chiếu của H lên AC, AB. Phép nghịch đảo IA AP.AC biến điểm B thành Q, C thành P, đường thẳng BC thành đường tròn ngoại tiếp tam giác AP Q, biến P A thành trung điểm M của BC, biến D thành D nằm trên đường tròn [AP Q]. Đường tròn [ABD] biến thành đường thẳng D Q, biến đường tròn [ACD] thành đường thẳng D P.VìvậyE = QD AC, F = PD AB. Suy ra đường tròn [AEF ] biến thành đường thẳng E F. Ta sẽ chỉ ra đường thẳng E F đi qua trung điểm M của BC. Dễ thấy E F là đường đối cực của điểm AD PQ. Áp dụng bổ đề trên ta có, PQ là đường đối cực của điểm M. Theo định lí La-hire, đường đối cực của điểm AD PQ sẽ đi qua điểm M, hay M thuộc đường thẳng E F. Điều phải chứng minh. 115

A F P E Q H D P A E B D M C F Bài toán. [Tạp chí toán học tuổi trẻ, số 490, năm 018]. Cho tam giác ABC có đường tròn [K] đi qua A, C tiếp xúc với AB; đường tròn [L] đi qua A, B tiếp xúc với AC.[K] cắt [L] tại D khác A.AK, AL lần lượt cắt BD,DC tại E,F. Gọi M,N theo thứ tự là trung điểm của BE,CF.P là điểm A Humpty của tam giác ABC. Chứng minh rằng bốn điểm A, P, M, N thẳng hàng. Lời giải. Cách 1. Từ giả thiết của đề bài ta thấy AB tiếp xúc với đường tròn [K] tại A, AC tiếp xúc với đường tròn [L] tại A. Trong tam giác vuông F AC có: NA = NC = CF NAC = NCA, [1] Mặt khác, do AB tiếp xúc với đường tròn [K] tại A, nên NCA = DCA = BAD [cùng chắn cung AD của đường tròn [K]] [] Từ [1] và [] suy ra: BAD = NAC, hay AD và AN là hai đường đẳng giác đối với BAC [3]. Tương tự, ta cũng chứng minh được BAM = ĈAD BAD = ĈAM hay AD và AM là hai đường đẳng giác đối với góc BAC. Từ [3], [4] suy ra ba điểm A, M, N thẳng hàng. 116

A E K L F M P O D N B C d[d, AB] Do ABD CAD[g.g] suy ra d[d, AC] = AB nên suy ra AD là đường đối trung của AC tam giác ABC. Do đó AM là đường trung tuyến của tam giác ABC. Do vậy A, P, M thẳng hàng. Suy ra điều phải chứng minh. Cách. J A E K U L F B D M P N Q O C R Giả sử đường tròn [L] cắt BC tại điểm thứ hai U.UD cắt AC tại điểm Q thì QUC = DAB = DCA. Do đó QCD QUC, nên QC = QD QU = QA. 117

Gọi J là điểm đối xứng của B qua A và R là điểm đối xứng của A qua U. Do hai tam giác CUA và CAB đồng dạng, nên hai tam giác CRA và CJB đồng dạng. Từ đó chú ý tam giác JBE cân tại E nên BJC = ÂRC = ÂUQ = ÂBD = BJE. Suy ra C, E, J thẳng hàng, mà A là trung điểm BJ do đó AM CE. Tương tự AN BF. Lại có ĈFA =90 ÂCF =90 0 BAD = DAE; tương tự thì ÂED = DAF. Từ đó hai tam giác AF D và EAD đồng dạng [g.g]. Kết hợp với tam giác BAD và ACD đồng dạng [g.g] ta suy ra DF DE = DA = DB DC. Từ đây ta thấy DB DE = DF nên BF CE suy ra A, M, N thẳng hàng. DC d[d, AB] Do ABD CAD[g.g] suy ra d[d, AC] = AB nên suy ra AD là đường đối trung của AC tam giác ABC. Do đó AM là đường trung tuyến của tam giác ABC. Do vậy A, P, M thẳng hàng. Suy ra điều phải chứng minh. Bài toán 3. [IMO Shorlist 008, G4]. Cho tam giác nhọn ABC và BE,CF là hai đường cao của tam giác. Hai đường tròn cùng đi qua hai điểm A và E và tiếp xúc với đường thẳng BC tại P và Q sao cho B nằm giữa C và Q. Chứng minh rằng hai đường thẳng PE và QF sẽ cắt nhau và giao điểm của hai đường thẳng này nằm trên đường tròn ngoại tiếp tam giác AEF. Lời giải. Cách 1. A S F H E Q B D P C Theo tính chất của điểm Humpty ta có B là trung điểm của PQ. Vẽ đường cao AD và gọi H là trực tâm của tam giác ABC. Các tứ giác CDF A, CDHE nội tiếp nên BA BF = BC BD = BE BH. 118

Từ đây suy ra BP = BE BH BP BH = BE hay tam giác BPH và BEP đồng dạng. Suy BP ra BPE = BHP [1]. Điểm P nằm giữa D và C, lại có BP = BC BD nên DP DQ =[BP BD] [BD + BQ] =BP BD = BC BD BD = BD [BC BD] =BD DC Vì hai tam giác vuông BDH và ADC đồng dạng nên BD DC = AD DH. Kết hợp với đẳng thức trên ta được DP DQ = AD DH DA DQ = DP DH. Từ đó hai tam giác vuông DAQ và DPH đồng dạng nên ĤPD = QAD. Có thể viết lại đẳng thức trên là BPH = BAD + BAQ. Vì BQ là tiếp tuyến của [AF Q] nên BQF = BAQ = BPH BAD. []. Từ [1] và [] suy ra BPE + BQF = BHP + BPH BAD =[180 0 PBH] BAD =[90 0 + BCA] [90 0 ÂBC] = BCA + ÂBC =180 0 ĈAB Do đó BPE + BQF < 180 0, vì vậy hai tia PE và QF phải cắt nhau. Gọi S là giao điểm của PE và QF. Khi đó: PSQ =180 0 [ BPE + BQF] =ĈAB = ÊAF. Nếu S nằm giữa P, E thì PSQ =180 0 ÊSF; còn nếu E nằm giữa P, S thì widehatp SQ = ÊSF. Từ đó ta nhận được S nằm trên đường tròn ngoại tiếp tam giác AEF. Cách. A V F T K H U S E Q B D P C 119

Gọi U, V, S là giao điểm thứ hai của P A, P H, P E với đường tròn đường kính AH. Tam giác AHP có các đường cao AV, BC, HU chúng đồng quy tại Q. Áp dụng định lí Pascal cho sáu điểm AESF HV ta có ba điểm C = AE FH,P = ES HV,SF VA thẳng hàng, suy ra Q FS. Giả sử đoạn thẳng AD theo thứ tự cắt đường tròn đường kính [BC], [PQ ] tại K, L. Vì H là trực tâm tam giác ABC, và BKC vuông nên ABD ADC DA DH = DB DC = DK. Vì H là trực tâm tam giác AQ P ta có DL = DA DH. VìvậyDK = DL K L. Kết hợp BKC vuông và ABD CBF, suy ra BD BC = BA BF = BK. Mặt khác, BA BF = BP = BQ, suy ra BK = BP = BQ. Điều này suy ra B là tâm đường tròn đường kính PQ. Do đó Q Q. Do đó giao điểm của PE và QF là S nằm trên đường tròn [AEF ]. Nhận xét 3. 1. Bài toán này ta nhận thấy, F chính là điểm A-Humpty của tam giác AP Q. Đồng thời, H vừa là trực tâm tam giác ABC vừa là trực tâm tam giác AP Q.. Nếu ta áp dụng định lí Pascal với sáu điểm AF T EHV ta cũng thấy ET đi qua Q. Như vậy: PF và QE cũng cắt nhau tại một điểm trên đường tròn [AEF ]. 3. Bài tập tương tự như sau: Cho tam giác ABC nhọn, có các đường cao BE,CF. Gọi P trên cạnh BC,Q thuộc tia đối của tia BC sao cho BQ = BP = BF BA. Gọi S = QF PE. Chứng minh bốn điểm E, S, A, F đồng viên. Một bài tập có mô hình tương tự, xin được chia sẻ tới bạn đọc. Bài toán 4. Cho tam giác ABC không cân và không vuông tại A. Các đường cao BE,CF. Các đường tròn [O 1 ], [O ] cùng đi qua A và theo thứ tự tiếp xúc với BC. D là giao điểm thứ hai của [O 1 ] và [O ].M,Ntheo thứ tự là giao điểm thứ hai của BA, BD và [O ].P, Q theo thứ tự là giao điểm thứ hai của CA, CD với [O 1 ].Rlà giao điểm của AD với EF. S là giao điểm của MP và NQ. Chứng minh rằng RS BC. Lời giải. Ta cần có một bổ đề: Cho tam giác ABC. Các đường cao BE,CF. Các đường tròn [O 1 ], [O ] cùng đi qua A và theo thứ tự tiếp xúc với BC. D là giao điểm thứ hai của [O 1 ] và [O ].Rlà giao điểm của AD với EF. X là giao điểm của BD và AC. Y là giao điểm của CD và AB. Khi đó: a. R thuộc XY. b. XY BC và RX = RY. 10

Chú ý rằng D là điểm A Humpty của tam giác ABC nên bổ đề này không khó, xin dành chứng minh cho bạn đọc. A O O 1 Y F H R D E X W B I C Trở lại bài toán, M N S Q A P O O 1 F Y R E D X B C Gọi X là giao điểm của BD và AC, Y là giao điểm của CD và AB. 11

Dễ thấy: [PB,PQ] [DB, DC] [DB, BC]+[BC,DC] [AD, AB]+[AC, AD] [AC, AB] [AP, AB] [QP, QB] [QB, QP ] [mod π] Do đó, tam giác BPQ cân tại B. Kết hợp với BC tiếp xúc với [O 1 ], suy ra PQ BC. Tương tự, tam giác CNM cân tại C và NM BC. Dễ thấy [PB,CN] [P B, AP ]+[AC, NC] [BD, AD]+[AD, ND] [BD,ND] 0 [mod π] Do đó: BP CN. Tương tự BQ CM. Vậy PQ BC NM,BP CN,BQ CM. Do đó các tam giác CMN,BPQ hoặc là ảnh của nhau qua một phép vị tự hoặc là ảnh của nhau qua một phép tịnh tiến. Vậy nên XP XC = BP NC = PB NC = PQ NM = QP NM = SP SM Do đó SX CM. Tương tự, chú ý rằng BP CN, ta có SY BP CN. Theo bổ đề trên, chú ý rằng MN BC, ta có XY BC MN. Vậy các tam giác SXY,CMN đồng dạng cùng hướng. Từ đó, chú ý rằng tam giác CMN cân tại C, suy ra tam giác SXY cân tại S. Lại theo bổ đề trên thì RX = RY nên suy ra RS BC. Bài toán 5. [ELMO 014.] Cho tam giác ABC với O, H lần lượt là tâm ngoại tiếp tam giác, trực tâm tam giác ABC. Gọi ω 1,ω là đường tròn ngoại tiếp tam giác BOC,BHC. Giả sử đường tròn đường kính AO cắt ω 1 tại điểm thứ hai M, đường thẳng AM cắt lại đường tròn ω 1 tại điểm N. Tương tự, giả sử đường tròn đường kính AH cắt đường tròn ω tại điểm thứ hai N và đường thẳng AN cắt lại đường tròn ω tại Y. Chứng minh rằng MN XY. Lời giải. Do OM AX nên OX là đường kính của đường tròn ω 1. Suy ra XC OC, XB OB hay XC,XB là các tiếp tuyến của đường tròn [O]. Suy ra AX là đường đối trung của tam giác ABC. Ta có BMX = BCX = BAC MBA = MAC. Tương tự ta cũng suy ra MCA = MAB. Ta có N là giao điểm thứ hai của hai đường tròn [AH], [BHC], nên theo tính chất 1 thì N là điểm A Humpty. Suy ra, N nằm trên đường trung tuyến dựng từ A của tam giác ABC. Do ABY C là hình bình hành và AX, AY đẳng giác nên MAC = NAB = ÂY C MBA = ÂY C. Suy ra AMB ACY AM AC = AB AM AY = AC AB [1]. AY Lại có: MBA = MAC = NAB = NBC. Suy ra ANB ACX AN AC = AB AN AX = AC AB []. AX Từ [1], [] suy ra AM AY = AN AX AM AX = AN MN XY. AY 1

A H M N O B C Y X Bài toán 6. [USA TST 005.] Cho tam giác ABC nội tiếp đường tròn [O]. Điểm P nằm trong tam giác ABC thỏa mãn PCB = P AC, PBC = P AB. Trung trực của đoạn AP cắt BC tại Q. Chứng minh rằng ÂQP =ÔQB. Lời giải. Không mất tính tổng quát, giả sử AB < AC. Gọi M là trung điểm của đoạn BC.D,K lần lượt là giao điểm của đường phân giác trong, phân giác ngoài ứng với góc A của tam giác ABC.X là giao điểm của hai tiếp tuyến tại B,C của đường tròn [O].Y là giao điểm thứ hai của AK với [O]. Suy ra Y,O,M,X thẳng hàng. Theo giả thiết, suy ra P chính là một điểm A Humpty của tam giác ABC. Theo tính chất về điểm A Humpty thì P nằm trên đường tròn A Apollonius, đường tròn đường kính KD. Suy ra Q chính là trung điểm của KD. Khi đó, ÂQP =ÂKP. Gọi F là giao điểm của OQ với AX, T là giao điểm thứ hai của AX với đường tròn [O]. Theo tính chất 4, ta suy ra T cũng thuộc đường tròn A Apollonius. Vậy OQ AX. Suy ra QF MX là tứ giác nội tiếp. Suy ra FQM = FXM.Do BX là tiếp tuyến của đường tròn [O] nên ÂCB = ÂBX + BXA ÂCP + BCP = ÂBX + BXA ÂCP = BXA 13

Y A K Q B F P D O M C T X Xét hai tam giác: AP C, ABX có P AC = BAX, ÂCP = ÂXB. Suy ra AP C ABX AP AX = AB AC. Xét hai tam giác: ABK, AY C có Ŷ AC = BAK, ÂBK = ÂY C. Suy ra ABK AY C AK AY = AB AC. Từ đó, suy ra AP AX = AK AY. Kết hợp với điều kiện P AK = Ŷ AX, ta suy ra AP K AY X ÂKP = ÂXY. Mặt khác, ÔQB = ÔQM = FQM = FXM = ÂKP, kết hợp với ÂQP =ÂKP, suy ra điều phải chứng minh. Bài toán 7. [Brazil National Olympiad 015.] Cho tam giác ABC không đều có H, G lần lượt là trực tâm, trọng tâm của tam giác. Gọi X, Y, Z là các điểm theo thứ tự nằm trên các đường thẳng BC, CA, AB thỏa mãn ÂXB = BY C = ĈZA. Hai đường tròn [BXZ], [CXY ] cắt nhau tại điểm thứ hai P. Chứng minh rằng P nằm trên đường tròn đường kính HG. Lời giải. Gọi M là điểm B Humpty, N là điểm C Humpty. Dễ thấy: M BG,N CG. và M,N cùng thuộc đường tròn đường kính HG. Do BXA + BZC = π nên giao điểm T của đường thẳng AX và CZ nằm trên đường tròn [BXZ]. Lại có [T A, T C] [BA, BC] [MA, MC] [mod π] 14

nên bốn điểm A, T, M, C đồng viên. Suy ra: [TX,TM] [CA, CM] [BC,BM] [BX,BM] [mod π]. Do đó: B,M,T,Z đồng viên. Vậy nên M thuộc vào đường tròn [BXZ]. Chứng minh tương tự N thuộc vào đường tròn [CXY ]. Áp dụng định lí Miquel vào tam giác BGC ta có: P thuộc vào đường tròn [GMN]. Vậy P nằm trên đường tròn đường kính HG. Điều phải chứng minh. A Z T NP H M G O Y B X C Bài toán 8. [Sharygin Geometry Olympiad 015.] Cho tam giác ABC nhọn, không cân. Gọi A 1,B 1,C 1 lần lượt là trung điểm của BC, CA, AB. Gọi B,C lần lượt là trung điểm của BA 1,CA 1. Gọi B 3,C 3 lần lượt là điểm đối xứng của C 1 qua B và của B 1 qua C. Chứng minh rằng giao điểm các đường tròn ngoại tiếp các tam giác BB B 3,CC C 3 nằm trên đường tròn ngoại tiếp tam giác ABC. Lời giải. Gọi P là điểm nằm trên đường tròn ngoại tiếp tam giác ABC sao cho AP là đường đối trung của tam giác. Gọi P là điểm đối xứng của P qua BC. Theo tính chất trên ta suy ra P là điểm A Humpty của tam giác ABC và P nằm trên đoạn AA 1.Ta có PA 1 B = ÂA 1 B = ÂCP = PBB 3 Suy ra AB là tiếp tuyến của đường tròn ngoại tiếp tam giác BA 1 P. Từ đó, ta có PBA = ĈA 1 P = ĈA 1 P. Lại có P CA 1 = B 1 AP nên PBA P A 1 C. Suy ra PBA PA 1 C. Từ đó, dễ dàng suy ra P là tâm vị tự quay biến CA 1 thành AB. Do B A 1 B C = B 3B B 3 A = 1 nên qua phép vị tự quay tâm P trên sẽ biến B thành B 3. Suy ra: B PB 3 = Â 1 PB = ÂBB hay P thuộc đường tròn [BB B 3 ]. Chứng minh tương tự ta suy ra được P thuộc đường tròn [CC 1 C ]. Điều phải chứng minh. 15

A C 1 P O B 1 B B C A 1 C B 3 P C 3 Bài toán 9. [G6, IMO Shortlist 014] Cho tam giác nhọn ABC. Xét các điểm E,F lần lượt nằm trên các cạnh CA, AB. Gọi M là trung điểm của EF. Trung trực của EF cắt BC tại K. Trung trực của MK cắt AC, AB theo thứ tự tại S,T. Giả sử cặp điểm E,F được gọi là thú vị nếu tứ giác KSAT nội tiếp. Giả sử các cặp E 1,F 1 và E,F là cặp điểm thú vị. Chứng minh rằng E 1E AB = F 1F AC. Lời giải. Với mỗi cặp điểm thú vị [E,F], ta sẽ nói tam giác KEF là tam giác thú vị. X A [ω 1 ] I M E S [ω] F Y C G N K B T L Tam giác EFK là tam giác thú vị. Trước hết ta chứng minh rằng KEF = KFE = Â, điều đó có nghĩa là đường tròn [ω 1 ] ngoai tiếp tam giác AEF tiếp xúc với KE và KF. 16

Kí hiệu [ω] là đường tròn đi qua các điểm A, K, S và T. Đường thẳng AM cắt ST và đường tròn [ω] tại N và điểm L. Vì EF TS và M là trung điểm của EF,N là trung điểm của ST. Do K, M đối xứng với nhau qua ST nên KNS = MNS = LNT. Vì vậy K, L đối xứng với nhau qua trung trực của ST. Vì vậy LK ST. Gọi G là điểm đối xứng với K qua N. Khi đó G nằm trên đường thẳng EF và ta có thể giả sử G nằm trên tia MF. Ta có: KGE = KNS = ŜNM = KLA =180 0 KSA [nếu K L thì góc KLA được hiểu là góc giữa AL và tiếp tuyến của đường tròn [ω] tại L]. Điều này có nghĩa là K, G, E, S đồng viên. Vì KSGT là hình bình hành nên KEF = KSG =180 0 TKS = Â Vì KE = KF nên ta có: KEF = KFE = Â Suy ra AK là đối trung của tam giác AST. A E 1 F Q Z E P B F 1 R K K 1 C Gọi Y là giao điểm của BE và CF. Xét bộ sáu điểm [EEY FFA] có K = EE FF,B = EY F A, C = FY EA thẳng hàng, nên theo định lí Pascal đảo ta có: Y thuộc đường tròn [AEF ]. Đường cao BP,CQ của tam giác ABC cắt nhau tại Z nên BY C = ÊY F =180 0 BAC = BZC Y [BZC] Gọi R là trung điểm của BC, AR giao với đường tròn [BZC] tại điểm thứ hai H. Khi đó H là điểm A Humpty của tam giác ABC. Ta có: FYH = ĤBC = ĤAF H [AEF ] Mà H [AP Q] nên theo phép vị tự quay tâm H, tỉ số HQ HP, góc quay QHP biến Q thành L, biến F thành E và biến F 1 thành E 1, biến F thành E. Do đó E 1E F 1 F = HP HQ = AP AQ = AB AC. 17

Bài toán 10. [Đề xuất 30/4/016, THPT Chuyên Lê Hồng Phong, TPHCM.] Cho tam giác nhọn ABC và điểm D di động trên tia đối của tia CB. Đường tròn [I] nội tiếp tam giác ABD và đường tròn [J] nội tiếp tam giác ACD. Giả sử H, K là giao điểm của hai đường tròn [I], [J]. Chứng minh HK luôn đi qua một điểm cố định khi D di động. Lời giải. Gọi N,M lần lượt là các điểm tiếp xúc của đường tròn nội tiếp [I] với các cạnh AD, BD.Q, P lần lượt là các điểm tiếp xúc của đường tròn nội tiếp [J] với các cạnh AD, BD.S là giao điểm của HK với AD. Theo định lí về điểm Humpty ta suy ra E,F là trung điểm của MP và NQ. Do đó: EM = EP = FN = FQ. Gọi T là hình chiếu của tâm bàng tiếp ứng với đỉnh B của tam giác ABC. Suy ra T cố định. Gọi F là giao điểm của AT với HK. Ta có: CA + CD AD CA + AB BC CP =,CT = AB + AD BD PT = CT CP = = AN Từ đây suy ra ET = AS, ED = SD. Áp dụng định lí Menelaus vào tam giác AT D với ba điểm S, F, E thẳng hàng ta có FA FT ET ED SD =1 FA = FT SA Suy ra F là trung điểm của đoạn AT, là điểm cố định. A U N I L S F K V Q J H B M EC P T D Bài toán 11. Cho tam giác ABC cân tại A và M là điểm ở trong tam giác ABC sao cho ÂBM = BCM. Gọi H, I, K lần lượt là hình chiếu vuông góc của điểm M lên AB, BC, CA. Gọi E là giao điểm của MB và IH,F là giao điểm của MC và IK. Giả sử hai đường tròn ngoại tiếp các tam giác MEH và MFK cắt nhau tại điểm thứ hai N. Chứng minh rằng đường thẳng MN đi qua trung điểm của BC. [Tạp chí toán học tuổi trẻ số 4/477-017] 18

Lời giải. Dễ thấy các tứ giác HBIM,KCIM nội tiếp, suy ra ĤIM = ÂBM = BCM; KIM = ÂCM = ĈBM. Do đó ÊMF = ÊIF = ÊMF + ĤIM + KIM = BMC + BCM + ĈBM =180 nên EMFI nội tiếp, ta có: MFE = MIE = MBH = BCM. Suy ra EF BC. Gọi P, Q lần lượt là giao điểm của MN với EF và BC. Ta có: PNE = MHE = MBC = MEP. Tương tự ta chứng minh được PNF = MFP. Suy ra M là điểm N Humpty của tam giác NEF. Do đó P là trung điểm EF, suy ra Q là trung điểm BC. A H N M K E P F B I Q C Bài toán 1. Cho tam giác nhọn ABC [AB < AC] với trung tuyến AM. Đường thẳng AM cắt đường tròn ngoại tiếp tam giác ABC tại điểm thứ hai D. Đường thẳng AB cắt đường thẳng CD tại E, đường thẳng AC cắt đường thẳng BD tại F. Đường tròn ngoại tiếp tam giác ABF cắt đường tròn ngoại tiếp tam giác ACE tại hai điểm A và P. Gọi [S 1 ] là đường tròn đi qua C và tiếp xúc với AB tại A, [S ] là đường tròn đi qua B tiếp xúc với AC tại A. Gọi O là tâm đường tròn ngoại tiếp tam giác ABC và Q là giao điểm thứ hai của [S 1 ] và [S ]. Chứng minh tam giác OPQ là tam giác vuông. [Đề thi chọn đội tuyển HSG quốc gia TP. Hà Nội, 016-017] Lời giải. Nhận thấy điểm Q chính là điểm A Dumpty của tam giác ABC. Ta có [PE,PF]=[PE,PA]+[P A, P F ]=[CE,CA]+[BA, BF ] =[CD, CA]+[BA, BD] =[CD, CA]+[CA, CD] =0 [modπ]. Do đó E,P,F thẳng hàng. Áp dụng định lý Menelaus cho tam giác ABM với cát tuyến EDC và tam giác ACM với cát tuyến FBD ta có EA EB CB CM DM DA =1 và FA FC BC BM DM DA =1. 19

Suy ra EA EB = FA hay BC EF. Khi đó FC PBD = DEP = BCD, dẫn đến PB là tiếp tuyến của đường tròn [O]. Chứng minh tương tự ta cũng có PC là tiếp tuyến của đường tròn [O]. Gọi Q là hình chiếu của O trên AP thì O, Q, B, C, P cùng nằm trên đường tròn đường kính OP. A Q O B M D C E P F Suy ra ÂBQ = ÂBC Q BC = ÂBC Q PC = ÂBC ÂP C = ÂBC ÂEC = BCD = BAD. Mặt khác, do BC EF nên BAD = BCD = ĈEP = ĈAP = ĈAQ. Do đó ÂBQ = ĈAQ. Suy ra AC là tiếp tuyến của [ABQ ] nên [ABQ ] [S ]. Chứng minh tương tự ta có [ACQ ] [S 1 ], suy ra Q Q. Vậy tam giác OPQ vuông tại Q. Bài toán 13. [Iranian Mathematical Olympiad.] Giả sử rằng P là một điểm nằm trong tứ giác ABCD sao cho BPC = BAC, PCA = P AD, PDA = P AC Chứng minh rằng PBD = PCA BCA Lời giải. Ta dễ dàng nhận thấy P là điểm A Dumpty của tam giác ADC. Kí hiệu [ω 1 ], [ω ] theo thứ tự là đường tròn ngoại tiếp các tam giác AP C, AP D. Q là giao điểm thứ hai của PB với đường tròn [ω 1 ],DBcắt [ω ], AQ theo thứ tự tại X, Y. Giả sử rằng Z là điểm thứ hai của AB với [ω 1 ]. Đặt P AD = PCA = β, PDA = P AC = θ. [ Dễ thấy Y tồn tại, vì nếu ngược lại thì AQ DB, khi đó DXP = ÂQB = DBP. Suy ra 130

B X, khi đó ÂBD = ÂXD = ÂP D =180 0 [β + θ]. Xét tam giác ABD ta nhận được BAC + ÂDB =0 0, điều này chỉ xảy ra khi A, B, C, D thẳng hàng [ vô lí]. ] Ta có: PQA = PCA = P AD = PXD Suy ra tứ giác PXQY nội tiếp. Suy ra BX BY = BP BQ. [ω ] [ω 1 ] Q Y A B X Z P D C Lại có: P B/[ω1 ] = BP BQ = BZ BA nên BX BY = BZ BA. Điều này có nghĩa là AXZY nội tiếp. Do đó, ÂZY = ÂXY =180 ÂXD Mặt khác, ÂXD = ÂP D = 180 [β + θ] nên ÂZY = β + θ. Hơn thế nữa, ÂZC = P AC + PCA = β +θ. Vậy nên ÂZC = ÂZY = β +θ. Mà ẐAC +ẐAY = QAC = BPC = BAC ẐAC = ẐAY = α nên suy ra AZC = AZY AC = AY. Điều này dẫn tới BAC = BAY BCA = BY A. Vì vậy PBD = QBY = BQA BY A = PCA BCA. 3. Bài tập tự luyện. Bài tập 1. [USA TSTST 015] Cho tam giác ABC không đều. Gọi K a,l a,m a theo thứ tự là giao điểm của BC với phân giác trong, phân giác ngoài và đường trung tuyến của tam giác ứng với góc A. Đường tròn ngoại tiếp tam giác AK a L a cắt AM a tại điểm thứ hai X a. Định nghĩa tương tự với X b,x c. Chứng minh rằng tâm đường tròn ngoại tiếp tam giác X a X b X c nằm trên đường thẳng Euler của tam giác ABC. Bài tập. [IMO 010/4, Modified] Cho tam giác ABC với trực tâm H. Giả sử P là hình chiếu của H lên trung tuyến CM của tam giác ABC. Giao điểm thứ hai của AP, BP, CP với đường tròn [ABC] theo thứ tự là K, L, M. Chứng minh rằng MK = ML. 131

Bài tập 3. [USA TST 008] Cho tam giác ABC với trọng tâm G. P là điểm di động trên đoạn thẳng BC. Gọi Q, R lần lượt là các điểm trên AC, AB sao cho PQ AB, P R AC. Chứng minh rằng khi P di động trên BC, đường tròn ngoại tiếp AQR luôn đi qua điểm cố định X thỏa mãn BAG = ĈAX. Bài tập 4. [EGMO 016/4] Hai đường tròn ω 1,ω có bán kính bằng nhau, cắt nhau tại hai điểm X 1,X. Xét đường tròn ω tiếp xúc ngoài với ω 1 tại T 1 và tiếp xúc trong với ω tại T. Chứng minh rằng X 1 T 1,X T cắt nhau tại một điểm nằm trên đường tròn ω. Bài tập 5. [Mathematical Reflections O371] Cho tam giác ABC với AB < AC. Gọi D, E lần lượt là chân đường vuông góc của B,C xuống AC, AB. Gọi M,N,P lần lượt là trung điểm của BC,MD,ME. Gọi S là giao điểm của NP và BC. Đường thẳng qua A song song với BC cắt DE tại T. Chứng minh rằng ST tiếp xúc với đường tròn ngoại tiếp tam giác ADE. Bài tập 6. [EGMO Shortlist 01/G7] Cho tam giác ABC[AB < AC] nhọn với O là tâm ngoại tiếp tam giác. Q là giao điểm của phân giác ngoài góc A với BC. P là điểm bên trong tam giác ABC sao cho BPA AP C. Chứng minh rằng QP A + ÔQB =90 0. Bài tập 7. [Iranian Geometry Olympiad 014] Tiếp tuyến tại A của đường tròn ngoại tiếp tam giác nhọn ABC[AB < AC] cắt BC tại P. Gọi X là điểm trên OP sao cho ÂXP = 90 0. Điểm E,F lần lượt nằm trên cạnh AB, AC và ở cùng phía so với OP sao cho ÊXP = ÂCX, FXO = ÂBX. K, L là giao điểm của EF với đường tròn ngoại tiếp tam giác ABC. Chứng minh rằng OP là tiếp tuyến của đường tròn ngoại tiếp tam giác KLX. Bài tập 8. Cho P là một điểm trên đường đối trung của tam giác ABC. Gọi O 1,O,Olần lượt là tâm đường tròn ngoại tiếp các tam giác AP B, CAP, ABC. Chứng minh rằng AO chia đôi O 1 O. Bài tập 9. Gọi M,N lần lượt là các điểm trên nửa đường tròn đường kính AB, tâm O. X là giao điểm của MN với AB. Gọi K là giao điểm thứ hai của hai đường tròn MBO, NAO. Chứng minh rằng XK KO. Bài tập 10. Cho tam giác ABC nội tiếp đường tròn [O] có đường cao AD. Đường tròn với tâm trên AD và tiếp xúc ngoài với đường tròn [BOC] tại X. Chứng minh rằng AX là đường đối trung của tam giác ABC. Bài tập 11. [Dựa theo đề đề xuất của Luxembourg, IMO Shortlisted 017,G Cho hai điểm R và S phân biệt trên đường tròn Ω và là tiếp tuyến của Ω tại R. Gọi R là điểm đối xứng của R qua S. Lấy điểm I trên cung nhỏ của cung RS của đường tròn Ω. Giả sử đường tròn ngoại tiếp tam giác ISR cắt đường thẳng tại hai điểm phân biệt và A là một điểm cắt gần điểm R. Đường thẳng AI cắt Ω tại điểm thứ hai J và K là giao điểm thứ hai của R I với Ω. Chứng minh rằng S là điểm R Dumpty của tam giác T AK. Bài tập 1. [IMO Shortlisted 017, G3] Cho tam giác ABC nội tiếp đường tròn [O]. Hlà trực tâm tam giác ABC. Đường thẳng OA cắt đường cao của tam giác ABC hạ từ B,C theo 13

thứ tự P, Q.GiLlØim A- Humptycatamgic ABC.Chngminhrng L nmtrnøngtrnøngknh HT vi T ltmøngtrnngoitiptamgic HPQ. Bài tập 13. Cho tam giác ABC có trực tâm H. Gọi M là trung điểm của đoạn BC, N là giao điểm của đoạn AM với đường tròn ngoại tiếp tam giác BHC. D là điểm nằm trong tam giác HBC. Gọi X BD, Y CD sao cho N là trung điểm của đoạn XY. Giả sử AD, BY, CX đồng quy tại L. Chứng minh rằng A là trung điểm của LD và bốn điểm L, Y, X, D đồng viên. Bài tập 14. [Toán học và tuổi trẻ, T1/49]. Cho tam giác ABC nhọn với các đường cao BE,CF. ST là một dây cung của đường tròn ngoại tiếp tam giác AEF. Hai đường tròn qua S, T tiếp xúc với đường thẳng BC lần lượt tại P, Q. Chứng minh rằng PE,QF cắt nhau trên đường tròn ngoại tiếp tam giác AEF. Bài tập 15. Trong mặt phẳng cho hai đường tròn [ω 1 ] và [ω ] cắt nhau tại A và B. Một tiếp tuyến chung của hai đường tròn tiếp xúc với [ω 1 ] ở P và [ω ] ở T. Các tiếp tuyến tại P và T của đường tròn ngoại tiếp tam giác AP T cắt nhau tại S. Gọi H là điểm đối xứng của B qua PT. Chứng minh A, H, S thẳng hàng. Bài tập 16. [USAMO 008.] Cho tam giác ABC. Gọi M,N,P lần lượt là trung điểm của AB, AC, BC. Trung trực của AB, AC cắt trung tuyến AP theo thứ tự tại D, E. Gọi F là giao điểm của BD và CE. Chứng minh rằng bốn điểm A, M, F, N đồng viên. Bài tập 17. [VN TST 01.] Trên mặt phẳng, cho đường tròn [O] và hai điểm cố định B,C trên đường tròn này sao cho BC không là đường kính của [O]. Gọi A là điểm di động trên đường tròn [O] và A không trùng B,C. Gọi D, K, J lần lượt là trung điểm của BC, CA, AB và E,M,N lần lượt là hình chiếu vuông góc của A, B, C trên BC,DJ,DK. Chứng minh rằng các tiếp tuyến tại M,N của đường tròn ngoại tiếp tam giác EMN luôn cắt nhau tại T cố định khi A thay đổi trên [O]. Tài liệu [1] Anant Mudgal,Gunmay Handa, A Special Point On The Median. [] Tạp chí Toán học & tuổi trẻ,,nhà xuất bản giáo dục Việt Nam. [3] Các bài toán trên các diễn đàn //artofproblemsolving.com,// diendantoanhoc.net. [4] Trần Quang Hùng, Mỗi tuần một bài toán hình học, NXB ĐHQG Hà Nội. [5] Nguyễn Văn Linh, 108 bài toán hình học sơ cấp, NXB ĐHQG Hà Nội. [6] Trần Nam Dũng [ Chủ biên], Các phương pháp giải toán qua các kì thi Olympic. 133

MỘT SỐ BỔ ĐỀ HỮU DỤNG TIẾP CẬN LỜI GIẢI TRONG CÁC BÀI TOÁN HÌNH HỌC Lê Viết Ân [TP Hồ Chí Minh] GIỚI THIỆU Trong bài viết này, chúng tôi xin giới thiệu một số bổ đề rất hữu ích có thể áp dụng trong các bài toán hình học, đặc biệt là có thể áp dụng tốt trong các bài hình thi học sinh giỏi. Các bổ đề được phát biểu đơn giản nhưng lại cho chúng ta một góc nhìn đơn giản hơn khi vẽ thêm hình phụ, thậm chí đặc biệt hóa chúng để áp dụng vào các bài toán khác. 1. Bổ đề 1. Định lý Reim Trong một bài viết [1] của tác giả người Pháp Jean-Louis Ayme có đưa ra một kết quả và gọi là định lí Reim như sau: Bổ đề 1. Cho hai đường tròn [O 1 ] và [O ] cắt nhau tại A và B. Một đường thẳng đi qua A cắt [O 1 ] và [O ] thứ tự tại A 1 và A ; và một đường thẳng đi qua B cắt [O 1 ] và [O ] theo thứ tự tại B 1 và B. Khi đó A 1 B 1 A B. Chứng minh. Có nhiều trường hợp xảy ra cho hình vẽ, ở đây tác giả chỉ chứng minh cho một trường hợp như hình 1.1. Các trường hợp khác bạn đọc chứng minh tương tự. Ta có  1 B 1 B =180  1 AB [vì ABB 1 A 1 nội tiếp] =  AB =180  B B [vì ABB A nội tiếp]. Suy ra  1 B 1 B +  B B =180. Do đó A 1 B 1 //A B [theo dấu hiệu hai góc trong cùng phía bù nhau]. Nhận xét. Ngoài ra bổ đề 1 vẫn đúng trong các trường hợp suy biến khi các điểm trùng nhau và khi đó ta xem cạnh do một cặp điểm trùng nhau là tiếp tuyến tại cặp điểm trùng nhau đó hoặc hai đường tròn [O 1 ] và [O ] tiếp xúc với nhau. Các trường hợp được minh họa bằng các hình vẽ sau đây [xem các hình 1.a, 1.b, 1.c, 1.d, 1.e, 1.f, 1.g, 1.h]. Điều ngược lại của bổ đề 1 cũng đúng. Và ta gọi là định lí Reim đảo với nội dung như sau: Cho bốn điểm A, B, A 1,B 1 cùng nằm trên đường tròn [O 1 ]. Trên các đường thẳng AA 1,BB 1 134

Hình 1.1 Hình 1.a 135

Hình 1.b Hình 1.c 136

Hình 1.d Hình 1.e 137

Hình 1.f Hình 1.g 138

Hình 1.h lấy các điểm A,B khác tương ứng sao cho A 1 B 1 A B thì khi đó tồn tại đường tròn [O ] đi qua bốn điểm A, B, A,B. Từ bổ đề 1 [định lí Reim thuận] và định lí Reim đảo, ta có hai hệ quả rất hay dùng để chứng minh đường thẳng tiếp xúc với đường tròn và hai đường tròn tiếp xúc với nhau. Hệ quả 1. Cho tam giác ABC nội tiếp đường tròn [O]. Một đường tròn [O ] đi qua các đỉnh B và C lần lượt cắt các đường thẳng AB và AC tại B và C khác A. Đường thẳng t đi qua đỉnh A. Khi đó t tiếp xúc với [O] khi và chỉ khi t B C. Hệ quả. Cho tam giác ABC. Các điểm B và C lần lượt nằm trên các đường thẳng AB và AC và khác với các đỉnh của tam giác. Khi đó hai đường tròn ngoại tiếp của các tam giác ABC và AB C tiếp xúc với nhau [tại A] khi và chỉ khi BC B C. Bây giờ ta đi xét một số thí dụ sau: Ví dụ 1 [P1, IMO 018]. Cho Γ là đường tròn ngoại tiếp của tam giác nhọn ABC. Các điểm D và E lần lượt được chọn trên các đoạn thẳng AB và AC sao cho AD = AE. Các đường trung trực của BD và CE lần lượt cắt các cung nhỏ AB và AC của Γ tại các điểm F và G. Chứng minh rằng DE và FGsong song với nhau. Lời giải.. Từ nội dung bổ đề 1, rõ ràng có hai hướng vẽ đường phụ để áp dụng bổ đề 1. Hướng thứ nhất, chúng ta xét hai giao điểm của các cặp đường thẳng FD và GE với đường tròn Γ; và hướng thứ hai là xét hai giao điểm của các cặp đường thẳng FE và GD với Γ. Sau đó tìm cách chứng minh hai giao điểm của mỗi hướng với các điểm D và E đồng viên. Tuy nhiên, khi vẽ hình và quan sát thì chú ý rằng từ giả thiết chúng ta có FD = FB [tương ứng GE = GC]. Do đó hướng thứ hai là hướng tốt nhất để có thể sử dụng được FD = FB. Cụ thể chúng ta đi đến lời giải như sau [xem h. 1.]: Gọi H, K lần lượt là giao điểm thứ hai của các đường thẳng FD và GE với đường tròn Γ. 139

Hình 1. Dễ thấy FBD AHD [g.g]. Kết hợp với FB = FD. suy ra AH = AD. Tương tự, ta cũng có AK = AE. Do đó AH = AD = AE = AK, suy ra bốn điểm D, E, H, K cùng thuộc đường tròn [A; AD]. Áp dụng bổ đề 1 cho hai đường tròn [A; AD] và Γ cắt nhau tại H và K với hai cát tuyến HDF và KEG thì DE FG. Ví dụ [P4, IMO 015]. Cho tam giác ABC nội tiếp đường tròn Ω tâm O. Đường tròn Γ tâm A cắt đoạn thẳng BC tại các điểm D và E sao cho B,D,E và C đôi một phân biệt và nằm trên đường thẳng BC theo đúng thứ tự đó. Gọi F và G là các giao điểm của Γ và Ω, sao cho A, F, B, C và G nằm trên Ω theo thứ tự đó. Gọi K là giao điểm thứ hai của đường tròn ngoại tiếp tam giác BDF và đoạn thẳng AB. Gọi L là giao điểm giao điểm thứ hai của đường tròn ngoại tiếp tam giác CGE và đoạn CA. Giả sử các đường thẳng FK và GL phân biệt và cắt nhau tại điểm X. Chứng minh rằng X nằm trên đường thẳng AO. Lời giải.. Giả sử các đường thẳng FD,GE,FK,GL cắt lại Ω tại các điểm P, Q, Y, Z [xem h. 1.3]. Áp dụng bổ đề 1 cho hai đường tròn Γ và Ω cắt nhau tại F và G với hai cát tuyến FDP và GEQ, ta có PQ//DE BC. Do đó BQ = CP. [1] Áp dụng bổ đề 1 cho hai đường tròn [BDF] và Ω cắt nhau tại B và F với hai cát tuyến BDC và FKY, ta có DK CY. [] 140

Hình 1.3 Áp dụng bổ đề 1 cho hai đường tròn [BDF] và Ω cắt nhau tại B và F với hai cát tuyến BKA và FDP, ta có KD AP. [3] Từ [] và [3] suy ra YC AP. Do đó AY = CP. [4] Chứng minh tương tự, ta cũng có AZ = BQ. [5] Từ [1], [4] và [5], suy ra AY = AZ, suy ra AO YZ. Từ đó YZ FG nên FGYZ là hình thang cân. Suy ra giao điểm hai đường chéo FK và GL phải nằm trên đường trục đối xứng của hình thang, cũng là trung trực của hai đáy FGvà YZ. Tức là X AO. Nhận xét. Hầu hết các lời giải cho bài toán này bằng cách biến đổi góc để chứng minh XF = XG. Tuy nhiên bằng cách sử dụng bổ đề 1 một cách khéo léo, chúng ta đã có một lời giải rất nhẹ nhàng cho bài toán mà không cần qua một bước biến đổi góc. Hơn nữa với cách giải tương tự, chúng ta còn có thể mở chứng minh cho bài toán mở rộng bài toán bằng cách thay đường tròn Γ tâm A bởi đường tròn Γ có tâm nằm trên đường thẳng AO. Ví dụ 3 [P3, IMO 013]. Đường tròn bàng tiếp góc A của tam giác ABC tiếp xúc với cạnh BC tại điểm A 1. Điểm B 1 trên CA và điểm C 1 trên AB được định nghĩa một cách tương tự bằng cách xét đường tròn bàng tiếp góc B và góc C tương ứng. Giả sử tâm đường tròn ngoại tiếp tam giác A 1 B C 1 nằm trên đường tròn ngoại tiếp tam giác ABC. Chứng minh rằng ABC là tam giác vuông. 141

Hình 1.4 Lời giải.. Giả sử I a,i b,i c là tâm đường tròn bàng tiếp các góc A, B, C của tam giác ABC. Gọi M a,m b và M c thứ tự là trung điểm của I b I c,i c I a và I a I b. Dễ thấy A, B, C là chân các đường cao của tam giác I a I b I c [xem h. 1.4]. Vì sáu điểm A, B, C, M a,m b,m c cùng nằm trên đường tròn Euler của tam giác I a I b I c. Ta có BAC = BM a C nên BM a C 1 = CM a B 1. Lại có M a BC 1 = M a BA = M a CA = M a CB 1 và M a B = M a C nên M a BC 1 = M a CB 1. Suy ra M a B 1 = M a C 1. Chứng minh tương tự, ta cũng có M b C 1 = M b A 1 và M c A 1 = M c B 1. Các điều này chứng tỏ rằng các đường trung trực của các đoạn thẳng B 1 C 1,C 1 A 1,A 1 B 1 với đường tròn ngoại tiếp tam giác ABC là các điểm trong ba điểm M a,m b,m c. Do đó giả thiết tâm đường tròn ngoại tiếp tam giác A 1 B 1 C 1 nằm trên đường tròn ngoại tiếp tam giác ABC, suy ra có một điểm trong ba điểm M a M b M c là tâm đường tròn ngoại tiếp tam giác A 1 B 1 C 1. Không mất tính tổng quát, ta giả sử M a là tâm đường tròn ngoại tiếp của tam giáca 1 B 1 C 1. Khi đó M a M c A 1 B 1. Mà M a M c AC và I b B CA nên A 1 B 1 I b B. [6] Gọi O là tâm đường tròn ngoại tiếp tam giác I a I b I c. Vì bốn điểm I b,i c,b,c cùng nằm trên một đường tròn đường kính I b I c nên theo OI a BC [một kết quả quen thuộc], mà I a A 1 BC tại A 1 nên ba điểm I a, A, O thẳng hàng. Từ đó bốn điểm C, B 1,A 1 và O cùng nằm trên đường tròn đường kính OC. Lại từ [6] nên theo phần đảo của bổ đề 1, ta có bốn điểm I b,b,c,ocùng nằm trên một đường tròn, suy ra O nằm trên đường tròn đường kính I b I c hay I b OI c =90. Từ đó chú ý rằng OI b CA và OI c AB nên BAC = I b OI c =90 [cặp góc có cạnh tương ứng vuông góc]. Nhận xét. Có thể nói rằng, bổ đề 1 cũng như phần đảo của nó rất hữu hiệu trong bài toán mà giả thiết có nhiều đường tròn tham gia. 14

Ví dụ 4 [P3, IMO 015]. Cho tam giác nhọn ABC với AB > AC. Ký hiệu Γ là đường tròn ngoại tiếp, H là trực tâm và F là chân đường cao hạ từ A của tam giác đó. Gọi M là trung điểm của BC. Gọi Q là điểm trên Γ sao cho ĤQA =90, và gọi K là điểm trên Γ sao cho ĤKQ =90. Giả sử rằng các điểm A, B, C, K và Q đôi một phân biệt và nằm trên Γ theo đúng thứ tự đó. Chứng minh rằng đường tròn ngoại tiếp của các tam giác KHQ và FKM tiếp xúc với nhau. Lời giải.. Kẻ đường kính AA của Γ thì ta có CH BA [vì cùng vuông góc với AB ]. Tương tự BH CA. Do đó BHCA là hình bình hành. Suy ra M là trung điểm HA [xem h. 1.5]. Hình 1.5 Lại có ÂQA =90 = ÂQH nên ba điểm Q, H, A thẳng hàng. Từ đó bốn điểm Q, H, M, A thẳng hàng. Đường thẳng AH cắt lại Γ tại H thì ÂHA =90. Do đó nếu gọi D := QK A H thì từ ĤKD = DH H =90, ta có tứ giác DKHH nội tiếp đường tròn đường kính DH. Vẽ đường tròn [N] đường kính DH thì N là trung điểm của DH. Ta có BC A D[vì cùng vuông góc với AH], kết hợp với BC đi qua trung điểm của HA, suy ra BC là đường trung bình của tam giác HA D nên BC đi qua trung điểm của HD hay N BC. Áp dụng bổ đề 1 cho hai bộ bốn điểm cùng nằm trên đường tròn dạng suy biến [HKQ] và [HKD] với hai cát tuyến HHD và KQD thì ta có QH DD [với ký hiệu DD là tiếp tuyến của [N] tại D], mà theo tính chất tiếp tuyến thì DD ND, suy ra ND QH QA tại H. Từ đó ta có NK = ND = NF.NM [hệ thức lượng cho tam giác vuông HMN]. Điều này chứng tỏ NK tiếp xúc với đường tròn ngoại tiếp FKM. [7] 143

Lại vì NH QH nên NH tiếp xúc với đường tròn đường kính QH [cũng chính là đường tròn ngoại tiếp HKQ], suy ra NKH = NHK = ĤQK nên NK cũng tiếp xúc với đường tròn ngoại tiếp HKQ, kết hợp với [7] ta suy ra đpcm. Nhận xét. Khéo léo áp dụng bổ đề 1 ở dạng suy biến cũng cho lời giải đẹp.. Bổ đề. Bổ đề góc thứ nhất Trong kì thi BrMO 01-013, vòng có một bài toán sau: Bổ đề. Cho tam giác ABC và điểm P nằm trong tam giác sao cho ÂBP = ÂCP. Dựng hình bình hành PBQC. Khi đó QAB = ĈAP. Đây là một kết quả liên quan chặt chẽ đến tứ giác nội tiếp trong một đường tròn, điều này sẽ được thấy rõ khi ta gọi giao điểm của các đường thẳng BP,CP và AC, AB tương ứng thì các giao điểm đó cùng với các đỉnh B,C cùng nằm trên một đường tròn. Sau đây chúng ta đi đến một số chứng minh cho bổ đề : Chứng minh. Cách 1. Nhận xét rằng yếu tố hình bình hành "gợi" cho chúng ta về quan hệ song song và bằng nhau của các cạnh đối diện của hình bình hành. Do đó sẽ "tận dụng" các cặp góc so le, đồng vị trong biến đổi góc, các đoạn thẳng bằng nhau để xem xét hai tam giác bằng nhau. Vì vậy chúng ta sẽ dựng thêm hình bình hành để tạo sự thuận lợi trong phép chứng minh. Cụ thể như sau: Dựng hình bình hành AP BR [xem h..1]. Dễ thấy ACQR là hình bình hành và P AC = Hình.1 144

BRQ [c.c.c]. Do đó PCA = BQR. Chú ý rằng PBA = BAR [so le trong]. Vậy BQR = PCA = PBA = BAR. Suy ra tứ giác AQBR nội tiếp. Do đó P AB = ÂBR = ÂQR = QAC. Cách. Chú ý rằng hình bình hành có một tâm đối xứng. Do đó, xem xét cấu hình của bổ đề 1, chúng ta sẽ tận dụng phép đối xứng tâm với tâm là trung điểm của BC. Cụ thể như sau: Gọi M là trung điểm cạnh BC; dựng hình bình hành ABA C; kẻ từ P các đường song song với các cạnh AB, AC cắt AB, AC, A B thứ tự tại E,F,G [xem h..a]. Hình.a Dễ thấy PBE PCF [g.g] và các hình bình hành AEP F, BEP G và A CFG. Do đó EP EA = EP FP = EB FC = GP GA, kết hợp với PEA = ÂF P = PGA. Suy ra PEA PGA [c.g.c]. Do đó P AE = PA G. Xét phép đối xứng tâm M: qua phép biến hình này thì các điểm A, B, C, P theo thứ tự biến thành A,C,B,Qnên QAC = PA B. Vậy P AB = P AE = PA B = QAC. Cách 3. Cách chứng minh tiếp theo là dựa vào quan hệ đồng dạng của hai hình, việc sử dụng mối quan hệ của hai hình đồng dạng vào giải toán là một kỹ năng nên rèn luyện nhuần nhuyễn 145

sẽ cho chúng ta cái nhìn nhanh chóng trong việc định hướng lời giải và vẽ thêm hình phụ. Cụ thể như sau: Gọi B = BP AC và C = CP AB. Dễ thấy ABC AB C. Do đó chúng ta có thể dựng điểm P sao cho ABC {P } AB C {P } [xem h..b]. Hình.b Dễ thấy B P CP là hình bình hành. Do đó ABC {P ; Q} AB C {P ; P }. Suy ra QAC = P AC = P AB. Cách 4. Từ giả thiết PBA = PCA và chú ý PBQ = PCQ. Suy ra QBA = QCA. Do đó chúng ta sẽ nghỉ đến định lí sin trong tam giác. Thật vậy, áp dụng định lí sin cho các tam giác ABP, ACP [xem h..1 hoặc h..a hoặc h..b], ta có: PB sin P AB = PA sin P AB = PA sin P AC = PC sin P AC = sin P AB sin P AC = PB PC. Tương tự, áp dụng định lí sin cho các tam giác ABQ, ACQ ta cũng thu được Chú ý PB = QC và PC = QB, suy ra sin QAC sin QAB = QC QB. sin P AB sin P AC = sin QAC sin QAB. Đến đây, kết hợp với P AB + P AC = BAC = QAC + QAB. Do đó ta phải có P AB = QAC và P AC = QAB [xem thêm bài viết ứng dụng của tỉ số đoạn thẳng và tỉ số lượng giác, Chuyên đề Toán học 10, PTNK]. 146

Nhận xét. Chú ý rằng bổ đề vẫn đúng khi điểm P nằm phía trong góc BAC chứ không nhất thiết điểm P phải nằm trong tam giác ABC. Ta sẽ điểm qua một số bài toán có thể áp dụng được bổ đề 1 sau: Ví dụ 5 [P5, ELMO 01]. Cho ABC là một tam giác nhọn với AB < AC, và D và E là các điểm nằm trên cạnh BC sao cho BD = CE và D nằm giữa B và E. Giả sử có điểm P nằm bên trong tam giác ABC sao cho P D AE và P AB = ÊAC. Chứng minh rằng P BA = P CA. Lời giải.. Dựng hình bình hành BPCQ [xem h..3]. Hình. Khi đó dễ thấy rằng từ BD = CE, suy ra QE PD AE suy ra ba điểm A, E và Q thẳng hàng. Áp dụng bổ đề, ta suy ra được ÂBQ = ÂCQ. Và do đó ÂBP = ÂCP. Ví dụ 6 [IMOSL G, 01]. Cho ABCD là một tứ giác nội tiếp với hai đường chéo AC và BD cắt nhau tại E. Các đường thẳng AD và BC kéo dài về phía A và B cắt nhau tại F. Cho G là điểm sao cho ECGD là hình bình hành, và H là đối xứng của E qua AD. Chứng minh rằng D, H, F, G đồng viên. Hướng dẫn. [xem h..3] Từ FCE = FDE. Do đó theo bổ đề, ta có ĈFG = DFE = DFH. Do đó tứ giác DGFH nội tiếp khi và chỉ khi DGH + DHF =180. Tuy nhiên, chú ý rằng FHG = FED [tính chất đối xứng trục] và FEB + FED =180. Do đó bài táo quy về chứng minh FEB = FGD. Điều này đồng nghĩ với việc chứng minh FEB FGD, tức là ta chỉ cần chứng minh FBE = FDG. Thật vậy, vì tứ giác ABCD và DG AC nội tiếp nên FBE = FBD = F AC = FDG. 147

Hình.3 Nhận xét. Rõ ràng, nếu chúng ta biết đến bổ đề thì ví dụ 6 thì thấy rõ được hướng đi của lời giải một cách nhanh chóng. Ví dụ 7 [ G4, IMOSL 009 ]. Cho tứ giác nội tiếp ABCD có các đường chéo AC và BD cắt nhau tại E, và các đường thẳng AD và BC cắt nhau tại F. Gọi G và H theo thứ tự là trung điểm của AB và CD. Chứng minh rằng EF là tiếp tuyến tại E của đường tròn đi qua các điểm E, G và H. Lời giải.. Dựng các hình bình hành AEBM và CEDN [xem h..4]. Hình.4 148

Khi đó G, H lần lượt là trung điểm của EM và FN. Do đó GH MN. Suy ra ÊHG = ÊNM. [8] Áp dụng bổ đề cho tam giác F AB với chú ý F AE = FBE, và cho tam giác FCD với chú ý FCE = FDE. Suy ra MFA = ÊFB = ÊFC = NFD. Suy ra ba điểm A, M, N thẳng hàng. Dễ thấy EAB {M; E; G} FCD {E; H; N}. Suy ra FEG = FNH. [9] Từ [8] và [9], suy ra FEG = ÊFG, tức là EF tiếp xúc với đường tròn ngoại tiếp tam giác EFG. Nhận xét. Bài toán chỉ đúng khi E,F,G không thẳng hàng. Điều này tương đương với AB và CD không song song với nhau. Ví dụ 8 [Đài Loan TST 014]. Cho tam giác ABC và điể P nằm trong tam giác, và các đường thẳng AP, BP, CP theo thứ tự cắt lại đường tròn ngoại tiếp tam giác ABC tại T,S,R. Cho U là một điểm nằm trên đoạn thẳng PT. Một đường thẳng qua U song song với AB cắt CR tại W, và đường thẳng khác qua U song song với AC cắt BS tại V. Các đường thẳng qua B song song với CP, qua C song song với BP cắt nhau tại Q. Giả sử rằng RS và VW song song với nhau, chứng minh rằng ĈAP = BAQ. Hướng dẫn. Gọi E = BP AC và F = CP AB [xem h..5]. Hình.5 Theo bổ đề, bài toán quy về chứng minh bốn điểm B,C,E,F đồng viên. 149

Áp dụng định lí Thales, ta có PV PE = PU = PW PA PF. Suy ra VW EF. Do đó RS VW EF. Ta có tứ giác BCSR nội tiếp với chú ý VW RS nên theo định lí Reim đảo, ta có tứ giác BCWV nội tiếp. Tiếp tục áp dụng định lí Reim đảo cho tứ giác BCSR với chú ý EF RS, ta có tứ giác BCEF nội tiếp. 3. Bổ đề 3. Định lý Koutras Dưới đây tác giả xin trình bày một tiêu chuẩn vuông góc của hai đường thẳng được biết đến với tên gọi là định lí Koutras. Kết quả này mặt dù đã biết đến trước năm 1980, tuy nhiên Stathis Koutras, một nhà nghiên cứu người Hy Lạp đã có những áp dụng đẹp cho tiêu chuẩn đơn giản này [hơn 300 bài toán có thể áp dụng nó] và phổ biến kết quả này. Nội dung của định lí này như sau: Bổ đề 3 [Định lí Koutras]. Cho tam giác ABC và đoạn thẳng MN. Gọi M 1,N 1 lần lượt là hình chiếu của M,N lên AB; M,N lần lượt là hình chiếu của M,N lên AC. Khi đó MN BC AB khi và chỉ khi = AC. M N M 1 N 1 Chứng minh. Có rất nhiều chứng minh cho bổ đề 3[xem thêm tại [5]], ở đây tác giả xin giới thiệu một cách chứng minh bằng trục đẳng phương [xem h. 3.1]. Hình 3.1 Đường thẳng qua M theo thứ tự song song với AB, AC cắt NN 1,NN tương ứng tại H, K và cắt BC tại P, Q. Gọi [I], [J] là đường tròn ngoại tiếp các tam giác HPN,KQN. Dễ thấy hai tam giác ABC và MPQ có các cặp cạnh tương ứng đôi một hoặc song song hoặctrùng nhau nên tồn tại một phép vị tự hoặc một phép tịnh tiến biến tam giác này thành tam 150

giác kia. Suy ra Mặt khác, lại có AB MP = AC MQ. [10] MH = M 1 N 1 và MK = M N. [11] Dễ thấy I,J là trung điểm của NP,NQ nên Vậy AB = AC M N M 1 N 1 IJ PQ BC. [1] [vì [10] và [11]] MH.MP = MK.MQ P M/[I] = P M/[J] MN là trục đẳng phương của [I] và [J] MN IJ MN BC [vì [1]]. Ta đi xem xét một số ví dụ minh họa cho sự áp dụng của bổ đề 3: Ví dụ 9 [Định lí Brahmagupta]. Đường thẳng nối giao điểm của hai đường chéo vuông góc trong tứ giác nội tiếp đường tròn với trung điểm của một cạnh bên thì luôn vuông góc với cạnh bên đối diện. Lời giải. Xét tứ giác ABCD nội tiếp có AC BD tại E. M là trung điểm của AB. Ta cần chứng minh ME BC [xem h. 3.] Hình 3. Gọi H, K lần lượt là hình chiếu vuông góc của M lên AE, BE. Khi đó H, K lần lượt là trung điểm của AE, BE.Chú ý tứ giác ABCD nội tiếp nên EA.EC = EB.ED = EH EK = EA EB = ED EC. Theo bổ đề 3, ta suy ra ME BC. 151

Nhận xét. Cách chứng minh định lí Brahmagupta ở trên là một hướng đi mới lạ. Không những thế, chúng ta còn có thể mở rộng kết quả này cho đường đối song của tam giác ở ví dụ tiếp theo sau đây mà cách chứng minh hoàn toàn tương tự. Ví dụ 10. Cho tam giác ABC nội tiếp đường tròn [O]. Chứng minh rằng đường đối song ứng với đỉnh A của tam giác vuông góc với AO [đường đối song ứng với đỉnh A là đường thẳng có ảnh đối xứng qua đường phân giác đỉnh A của tam giác song song với BC]. Hướng dẫn. Cho l là một đường đối song của ứng với đỉnh A của tam giác, ta cần chứng minh AO l. Giả sử CA l = E và AB l = F. Khi đó B,C,E,F đồng viên. Phần tiếp theo bạn đọc chứng minh hoàn toàn tương tự như ví dụ trước. Nhận xét. Ví dụ này là một kết quả cơ bản của đường đối song trong tam giác. Kết quả này cũng có rất nhiều áp dụng và bạn đọc nên nhớ. Ví dụ 11. Cho hai đường thẳng x và x không song song với nhau. Các điểm phân biệt A, B, C thuộc x và các điểm phân biệt A,B,C thuộc x. Gọi a, b, c là các đường thẳng lần lượt đi qua A, B, C và cùng vuông góc với x; các đường thẳng a,b,c lần lượt đi qua A,B,C và cùng vuông góc với x. Gọi M = a a, N = b b và P = c c. Chứng minh rằng M,N,P thẳng hàng khi và chỉ khi AB = A B. AC A C Lời giải.. Gọi Q = x x ; tồn tại ít nhất cặp điểm D x và D x sao cho D D và DD MN [xem h. 3.3] Hình 3.3 Áp dụng bổ đề 3, từ MN DD suy ra Ta có M,N,P thẳng hàng PM DD AC = A C QD QD AB QD = A B QD. [13] 15 AB = A B AC QD [vì [13]].

Nhận xét. Ví dụ này cũng là một kết quả cơ bản có nhiều áp dụng và bạn đọc cũng nên nhớ. Hơn nữa chúng ta còn có thể chứng minh kết quả tổng quát như sau: Cho góc BAC và các đoạn thẳng MN và KL. Gọi M 1,N 1,K 1,L 1 theo thứ tự là hình chiếu của M,N,K,L lên AB, vàm,n,k,l là hình chiếu vuông góc của M,N,K,L lên AC. Khi đó M 1N 1 = M N khi và chỉ khi hai đường thẳng MN và KL cùng phương [song song hoặc K 1 L 1 K L trùng nhau]. Ví dụ 1 [P5, IMO 1985]. Một đường tròn tâm O đi qua các đỉnh A và C của tam giác ABC và cắt AB và BC lần lượt tại các điểm thứ hai K và N. Gọi M là giao điểm khác B của các đường tròn ngoại tiếp các tam giác ABC và KBN. Chứng minh rằng ÔMB =90. Lời giải. Vì M khác B nên tam giác ABC không cân tại B. Không mất tính tổng quát, giả sử AB < BC [xem h. 3.4]. Hình 3.4 Ta có BM, KN, AC đồng quy tại tâm đẳng phương của ba đường tròn [O], [ABC], [BKN]. Gọi P := BM KN AC; BB là đường kính của [ABC] và L = BB KN. Áp dụng tính chất điểm Miquel cho tứ giác toàn phần ACNKBF, ta có M [CNP]. Và áp dụng tính chất đường đối song [xem ví dụ 10] cho tam giác ABC, ta có BB KN tại L nên B LN =90 = B CN. Suy ra bốn điểm B,C,N,Lđồng viên. Do đó BL.BB = P B/ [B CNL] = BC.BN = P B/ [CNP] = BP.BM. Suy ra B,L,M,P đồng viên hay B M BP. Do đó bài toán sẽ được chứng minh nếu ta chỉ ra được O B M hay B O BP. Thật vậy, gọi O là tâm của [ABC]; G, H lần lượt là trung điểm của AC, CN; và R, S lần lượt là hình chiếu của B,B lên AC. Khi đó O là trung điểm của BB và OO H là trung trực 153

của AC nên AR O H BS nên RS =HS. [14] Mặt khác, lại có NC =GC. [15] Áp dụng bổ đề 3 với chú ý BB PN. Ta có RS = BC. Kết hợp với [14] và [15], suy ra NC PC HS = BC. Do đó lại theo bổ đề 3, ta có GC PC B O BP. Bài toán được chứng minh. Nhận xét. Bài toán được xử lí theo hướng sử dụng bổ đề 3 cho ta một cách nhìn khá thú vị và tự nhiên. Ví dụ 13 [IMOSL G5, 005]. Cho tam giác nhọn ABC với AB AC, H là trực tâm của tam giác và M là trung điểm của BC. Các điểm D trên AB và E trên AC và sao cho AE = AD và D, H, E thẳng hàng. Chứng minh rằng HM vuông góc với dây cung chung của các đường tròn ngoại tiếp các tam giác ABC và ADE. Lời giải.. Gọi [O], [K] là các đường tròn ngoại tiếp các tam giác ABC, ADE; G là giao điểm khác A của [O] và [K]. Ta cần chứng minh HM AG [xem h. 3.5] Hình 3.5 Dễ thấy OK là trung trực của AG nên OK AG. Do đó ta chỉ cần chứng minh HM OK. Thật vậy, gọi O 1,K 1,M 1,H 1 lần lượt là hình chiếu của O, K, M, H lên AB, O,K,M,H lần lượt là hình chiếu của O, K, M, H lên AC. Ta có O 1 K 1 = O 1 A K 1 A = CA EA = CE. Tương tự, O K = BD. Suy ra O 1 K 1 = CE O K BD. [16] 154

Mặt khác, lại có ÂEH = ÂDH [vì AE = AD] vàêch = DBH nên ECH DBH. Do đó ÊHC = DHB, kết hợp với D, H, E thẳng hàng. Suy ra DE là phân giác ngoài tại đỉnh H của tam giác BCH. Do đó HE và HD theo thứ tự là phân giác trong của các tam giác H 1 CH và H BH. [17] Dễ dàng có được hai tam giác H 1 CH và H BH đồng dạng ngược hướng. [18] Từ [17] và [18] suy ra CE BD = CH 1 BH = M 1H 1 M H = M 1H 1 M H. [19] Từ [16] và [19] suy ra O 1K 1 O K = M 1H 1 M H. Do đó MH OK.VậyMH AG [đpcm]. Nhận xét. Có thể áp dụng kết quả ví dụ 1 để giải ví dụ 13. Và ngược lại có thể sử dụng cách chứng minh của ví dụ 13 để giải ví dụ 1. Ví dụ 14 [Định lí "Bướm đơn"]. Cho tứ giác ABCD nội tiếp đường tròn [O] có hai đường chéo AC và BD cắt nhau tại E. Đường thẳng qua E vuông góc với OE cắt AD, BC thứ tự tại P và Q. Chứng minh rằng EP = EQ. Lời giải. Gọi M,N thứ tự là trung điểm của AD, BC; H, K thứ tự là hình chiếu của E lên AD, BD; và gọi F = AD BC [điểm F luôn tồn tại vì do hai điểm P và Q xác định] [xem h. 3.6] Hình 3.6 155

Vì OE PQnên áp dụng bổ đề 3, ta có MH KN = FQ FP. [0] Vì EAD EBC với chú ý M,N là các trung điểm của các cặp cạnh AD, BC và EH,EK là hai đường cao của hai tam giác này. Do đó EAD {H; P } EBC {K; N}. Do đó EH EK = MH KN. [1] Từ [0] và [1], suy ra EH.FP = EK.FQ. Tức là diện tích hai tam giác EFP và EFQ bằng nhau. Do đó E là trung điểm của PQ. Nhận xét. Chứng minh định lí "Bướm đơn" có rất nhiều cách khác nhau, tuy nhiên dưới góc nhìn bằng cách sử dụng bổ đề 3 là một hướng đi rất mới lạ. 4. Bổ đề 4. Bổ đề góc thứ hai Tại facebook [6] của một nhóm hình học PERU GEOMETRICO, trong đó có một bài toán của tác giả Miguel Ochoa Sanchez như sau: Bổ đề 4. Cho tam giác ABC và các điểm P thuộc AB và Q thuộc AC. Đường tròn ngoại tiếp các tam giác ABQ và ACP cắt nhau tại D khác A. Gọi M,N lần lượt là trung điểm của BC và PQ. Khi đó [AB, NM] [AC, AD][ mod π]. Chứng minh. Gọi G là trung điểm của BD [xem h. 4.1]. Dễ thấy [PB,PD] [P A, P D] [CA, CD] [CQ,CD][ mod π] và [BP,BD] [BA, BD] [QA, QD] [QC, QD][ mod π]. Do đó Suy ra hai tam giác DBP và DQC đồng dạng cùng hướng. [] DB DQ = BP QC = GN GM = GN GM, và [ GM, GN] [ QC, BP] [ DQ, DB][vì []][ mod π]. Do đó hai tam giác GMN và DQB đồng dạng cùng hướng. Suy ra [MN, AB] [NM,NG] [BQ,BD] [AC, AD][ mod π]. Nhận xét. Nếu MN cắt AB tại T. Khi đó hoặc BTM = DAC hoặc BTM + DAC =180. 156

Hình 4.1 Bổ đề 4 ở dưới dạng tổng quát, do đó đặc biệt hóa kết quả này sẽ thu được một số bài toán có ý nghĩa. Nếu cho MN đi qua đỉnh A của tam giác thì khi đó PQ BC. Theo bổ đề 4 thì khi đó AD là đường đối trung đỉnh A của tam giác. Do đó chúng ta thu được bài toán sau Ví dụ 15 [P, Balkan MO, 009]. Cho tam giác ABC, các điểm M và N lần lượt nằm trên các canhj AB và AC sao cho MN BC. Gọi P là giao điểm của BN và CM. Đường tròn ngoại tiếp các tam giác BMP và CNP cắt nhau tại hai điểm phân biệt P và Q. Chứng minh rằng BAQ = ĈAP. Lời giải. Dễ thấy Q là điểm Miquel của tứ giác toàn phần AMP NBC. Do đó P [ABN] [ACM] [BPM] [CPN] [xem h. 4.]. Gọi D := AP BC và F := AP MN. Khi đó, từ MN BC suy ra D, E là trung điểm của BC và MB. Áp dụng bổ đề 4, ta có BAQ = ÊAC = P AC. Bây giờ, ở cấu hình của bổ đề 4, nếu cho AD là phân giác trong của BAC. Khi đó, DB = DQ, DC = DP và vì DCQ DPB. Suy ra BP = CQ. Hơn nữa, theo kết quả bổ đề 4 thì MN tạo với hai đường thẳng AB, AC một cặp góc bằng nhau nên MN song song với AD. Ta thu được một kết quả kinh điển như sau Ví dụ 16. Cho tam giác ABC. Các điểm E và F theo thứ tự nằm trên các tia CA và BA sao cho BF = CE. Gọi M và N theo thứ tự là trung điểm của BC và EF. Chứng minh rằng MN song song với phân giác trong góc A của tam giác ABC. 157

Hình 4. Hình 4.3 158

Lời giải. Gọi G = CF BE; K = MN AC và D là điểm Miquel của tứ giác AF GEBC. Áp dụng bổ đề 4, ta có BAD = MKC. [3] Dễ thấy DBF DEC, mà BF = CE nên DB = DE. Do đó AD là phân giác của BAE. Suy ra BAD = BAC. [4] Từ [3] và [4] suy ra MKC = DAC. Do đó MN MK AD. Nhận xét. Bài toán này có nhiều cách giải đơn giản hơn trên, tuy nhiên chúng ta đi từ bổ đề 4 cho ta một góc nhìn khác để dẫn đến ví dụ này. Ở cấu hình bổ đề 4, nếu chọn các điểm P, Q sao cho PQlà đường đối song của tam giác ABC thì chúng ta thấy rằng bổ đề 4 là mở rộng của bổ đề. Bây giờ, nếu từ cấu hình của bổ đề 4, cho đường thẳng AD đi qua tâm đường tròn ngoại tiếp thì chúng ta thu được MN BC. Do đó N thuộc trung trực của BC. Ta đi đến bài toán sau Ví dụ 17. Cho tam giác ABC nội tiếp đường tròn [O]. Điểm D nằm trên AO khác A. Đường tròn ngoại tiếp các tam giác ABD, ACD theo thứ tự cắt AB, AC tại E,F. Chứng minh rằng trung điểm của EF cách đều hai đỉnh B và C. Lời giải. Gọi M,N lần lượt là trung điểm của BC,EF;vàH là hình chiếu của A lên BC [xem h. 4.4]. Hình 4.4 159

Áp dụng bổ đề 4 với chú ý rằng AH và AO là đẳng giác nhau qua BAC, ta có [[MN, AB] [AC, AD] [AC, AO] [AH, AB][ mod π]. Suy ra MN AH BC. Do đó MN là trung trực của BC nên NB = NC [đpcm]. Cuối cùng, chúng ta sẽ đi đến một áp dụng đẹp của bổ đề 4 thông qua bài toán sau đây Ví dụ 18. Cho tam giác không cân ABC nội tiếp đường tròn Γ, và đường tròn nội tiếp tâm I tiếp xúc với các cạnh BC, CA, AB theo thứ tự tại D, E, F. Tia AI cắt Γ tại L, và đường thẳng EF cắt BC tại G. Các đường tròn ngoại tiếp của các tam giác GBE và GCF cắt nhau tại K khác G. Giả sử đường tròn ngoại tiếp tam giác DKL cắt lại các đường thẳng BC và AL thứ tự tại P và Q. Chứng minh rằng đường thẳng PQchia đôi các đoạn thẳng AG, BE và CF. Lời giải. Gọi H, M, N lần lượt là trung điểm của AG, BE, CF thì H, M, N là cùng nằm trên đường thẳng Gauss của tứ giác BCEF; ta định nghĩa lại điểm P và Q là giao điểm của đường thẳng HMN lần lượt với BC và AL. Do đó bài toán trở thành việc chứng minh năm điểm D, K, L, P, Q cùng nằm trên một đường tròn [xem h. 4.5]. Hình 4.5 Trước hết, gọi S là giao điểm khác A của đường tròn [AEF ] và Γ. Khi đó S là điểm Miquel của tứ giác toàn phần BCEF AG. Do đó S [CEG]. Áp dụng bổ đề 4, ta có [PQ,PD] [HM,GC] [CA, CS][ mod π]. [5] Mặt khác, dễ thấy có một phép vị tự quay S 1 biến SBF thành SCE. Chú ý rằng BF = BD và CE = CD. Suy ra SB = BF = BD. Do đó SD là phân giác của tam giác SBC nên SD đi SC CE CD qua điểm L là trung điểm của cung BC không chứa A của Γ. Do đó [CA, CS] [LA, LS] [LQ, LD][ mod π]. [6] 160

Từ [5] và [6] suy ra [PQ,PD] [LQ, LD][ mod π].vậy bốn điểm D, L, P, Q đồng viên. [7] Gọi R = AL EF thì E là trung điểm của EF. Khi đó RM, RN là các đường trung bình của các tam giác BEF và CEF. Do đó RM AB, RN AC và RM = BF và RN = CE. Kết hợp với AQ là phân giác trong của BAC, suy ra RQ là phân giác của tam giác RMN. Suy ra QM QN = RM RN = BF = BD CE CD. Chú ý các điểm D, Q cùng nằm trong đoạn BC và MN nên QM QN = DB DC. [8] Gọi J := BE CF. Áp dụng định lí Miquel cho tứ giác toàn phần BCEFGJ ta có K là điểm Miquel của tứ giác này. Do đó K là giao điểm khác J của hai đường tròn [JBC] và [JEF]. Do đó tồn tại một phép vị tự quay S biến KBE thành KCF. Chú ý rằng M,N theo thứ tự là trung điểm của BE và CF. Do đó qua phép biến hình S thì M biến thành N. Vìvậy [KM,KN] [BE,CF] [IB,IC] [IM,IN][ mod π]. Do đó K [IMN]. Do đó tồn tại một phép vị tự quay S 3 biến KBC thành KMN. [9] Từ [8] và [9] suy ra qua phép biến hình S 3 biến D thành Q. Do đó [DP, DK] [DC, DK] [QN, QK] [QP, QK][ mod π]. Do đó bốn điểm D, K, P, Q đồng viên. [30] Từ [7] và [30] suy ra năm điểm D, K, L, P, Q cùng nằm trên một đường tròn. Ta có đpcm. 5. Bài tập Bài toán 1 [IMOSL G1, 015]. Cho tam giác nhọn ABC với trực tâm H. Lấy điểm G sao cho tứ giác ABGH là hình bình hành. Gọi I là điểm trên đường thẳng GH sao cho AC chia đôi HI. Giả sử đường thẳng AC cắt đường tròn ngoại tiếp GCI tại C và J. Chứng minh rằng IJ = AH. Bài toán [P, AMPO 017]. Cho tam giác ABC với AB < AC. Gọi D là giao điểm thứ hai của phân giác trong của góc BAC và đường tròn ngoại tiếp tam giác ABC. Gọi Z là giao điểm của trung trực của AC với phân giác ngoài của góc BAC. Chứng minh rằng trung điểm của đoạn thẳng AB nằm trên đường tròn ngoại tiếp tam giác ADZ. Bài toán 3 [P1, Advanced, IGO 018]. Hai đường tròn ω 1, ω cắt nhau tại A và B. Gọi PQlà một tiếp tuyến chung của hai đường tròn với P ω 1 và Q ω. Một điểm X bất kì nằm trên ω 1. Đường thẳng AX cắt ω tại điểm thứ hai Y. Điểm Y Y nằm trên ω sao cho QY = QY. Đường thẳng Y B cắt ω 1 tại điểm thứ hai X. Chứng minh rằng PX = PX. 161

Bài toán 4 [P4, Iranian TST 018]. Cho ABC là một tam giác [Â 90 ]. BE,CF là các đường cao của tam giác. Phân giác góc A cắt EF, BC lần lượt tại M,N. Cho P là một điểm sao cho MP EF và NP BC. Chứng minh rằng AP đi qua trung điểm của BC. Bài toán 5 [P3, USAMO 005]. Cho ABC là một ttam giác nhọn, và P và Q là hai điểm nằm trên cạnh BC. Lấy điểm C 1 sao cho tứ giác lồi AP BC 1 nội tiếp, QC 1 CA và các điểm C 1 và Q 1 nằm cùng phía so với AB. Lấy điểm B 1 sao cho tứ giác lồi AP CB 1 nội tiếp, QB 1 BA và các điểm B 1 và Q 1 nằm cùng phía so với AC. Chứng minh rằng các điểm B 1,C 1,P và Q cùng nằm trên một đường tròn. Bổ đề 5 [P3, China TST 018]. Cho đường tròn ω tiếp xúc với các cạnh AB, AC của tam giác ABC lần lượt tại D, E, sao cho D B, E C và BD + CE < BC. Các điểm F, G nằm trên cạnh BC sao cho BF = BD và CG = CE. Các đường thẳng DG và EF cắt nhau tại K. Điểm L nằm trên cung nhỏ DE của đường tròn ω, sao cho tiếp tuyến của ω tại L song song với BC. Chứng minh rằng tâm đường tròn nội tiếp của tam giác ABC nằm trên đường thẳng KL. Bài toán 6 [P5, China TST 018]. Cho tam giác ABC với BAC > 90, và cho O là tâm đường tròn ngoại tiếp và ω là đường tròn ngoại tiếp của tam giác. Tiếp tuyến của ω tại A cắt tiếp tuyến của ω tại B và C lần lượt tại P và Q. Gọi E,F lần lượt là chân đường cao kẻ từ P, Q đến BC. F, G là hai điểm nằm trên đoạn PQ khác A sao cho A, F, B, E và A, G, C, D là các bộ điểm đồng viên. Gọi M là trung điểm của DE. Chứng minh rằng DF, OM, EG đồng quy. Bài toán 7 [P3, Intermediate, IGO 015]. Trong tam giác ABC, M,N,P theo thứ tự là trung điểm của các cạnh BC, CA, AB. Dựng các nữa đường tròn đường kính AB, AC ra bên ngoài tam giác ABC. MK,MN cắt các nữa đường tròn thứ tự tại X, Y. Tiếp tuyến của các nữa đường tròn tại X, Y cắt nhau tại Z. Chứng minh rằng AZ BC. Bài toán 8 [Bài O86, Mathematical reflections]. Cho tam giác ABC với trực tâm H. Gọi HM là đường trung tuyến và HS là đường đối trung của tam giác BHC. Gọi P là hình chiếu vuông góc của A trên HS. Chứng minh rằng đường tròn ngoại tiếp của các tam giác MPS và ABC tiếp xúc với nhau. Bài toán 9 [P4, IMO 013]. Cho tam giác nhọn ABC với trực tâm H, W là một điểm tùy ý nằm trên cạnh BC và W nằm giữa B và C. Các điểm M và N lần lượt là chân đường vuông cao hạ từ các đỉnh B và C. Kí hiệu ω 1 là đường tròn ngoại tiếp tam giác BWN, và gọi X là điểm trên ω 1 sao cho WX là đường kính của ω 1. Tương tự kí hiệu ω là đường tròn ngoại tiếp tam giác CWM, và gọi Y là điểm trên ω sao cho WY là đường kính của ω. Chứng minh rằng ba điểm X, Y, H thẳng hàng. Bài toán 10 [P, EGMO 016]. Cho tứ giác nội tiếp ABCD, và các đường chéo AC và BD cắt nhau tại X. Gọi C 1,D 1 và M theo thứ tự là trung điểm của các đoạn thẳng CX,DX và CD. Các đường thẳng AD 1 và BC 1 cắt nhau tại Y, và đường thẳng MY cắt các đường chéo AC và BD thứ tự tại các điểm phân biệt E và F. Chứng minh rằng đường thẳng XY tiếp xúc với đường tròn đi qua E,F và X. Bài toán 11 [Việt Nam TST 006]. Cho tam giác nhọn ABC và trực tâm H. Phân giác ngoài của tam giác ABC cắt các cạnh AB, AC thứ tự tại D và E. Đường phân giác trong của góc BAC cắt đường tròn ngoại tiếp tam giác ADE tại K. Chứng minh rằng HK đi qua trung điểm của BC. 16

Bài toán 1. Cho tam giác cân ABC với AB = AC. T là một điểm trong đoạn CA. Trên tia BT lấy điểm S sao cho BS = BA. Gọi O, I theo thứ tự tâm đường tròn ngoại tiếp và tâm đường tròn nội tiếp tam giác ABT. Chứng minh rằng OI vuông góc CS. Bài toán 13. Cho tam giác không cân ABC có O, I theo thứ tự là tâm đường tròn ngoại tiếp và tâm đường tròn nội tiếp của tam giác. Gọi M,N theo thứ thứ tự là đối xứng của các đỉnh B,C tương ứng qua CI,BI. Chứng minh rằng OI vuông góc với MN. Bài toán 14. Cho tam giác ABC không cân tại A và nội tiếp đường tròn [O]. Đường trung tuyến kẻ từ đỉnh A của tam giác cắt BC tại M và [O] tại G khác A. Gọi H, K thứ tự là hình chiếu của G lên AB, AC. Gọi N và P theo thứ tự là trung điểm của BK và CH. Chứng minh rằng AG tiếp xúc với đường tròn đi qua M,N,P. Bài toán 15 [P7, USA TSTST 01]. Cho tam giác ABC nội tiếp đường tròn Ω. Phân giác trong góc A cắt BC và Ω thứ tự tại D và L [khác với A]. Gọi M là trung điểm cạnh BC. Đường tròn ngoại tiếp tam giác ADM cắt các cạnh AB và AC thứ tự tại Q và P khác A. Gọi N là trung điểm của PQ,vàH là chân đường vuông góc hạ từ N xuống đường thẳng ND. Chứng minh rằng đường thẳng ML tiếp xúc với ngoại tiếp của tam giác HMN. Lời kết Bài viết đã giới thiệu một số bổ đề điển hình để bạn đọc có thêm công cụ trong việc giải các bài toán hình học ở các kì thi HSG. Tất nhiên để giải quyết các bài toán hình học, chúng ta cần phải khéo léo, tư duy trừu tượng nhạy bén. Để đạt được những điều này thì việc rèn luyện thường xuyên để tăng thêm kinh nghiệm cho bản thân. Vẫn còn rất nhiều bổ đề khác nữa tác giả sẽ giới thiệu cho bạn đọc vào một bài viết khác. Rất mong nhận được sự góp ý của bạn đọc về nội dung. Cuối cùng, tác giả gửi lời cảm ơn đến thầy Trần Nam Dũng và các bạn trong Ban biên tập đã đọc lại bản thảo và đưa ra những góp ý xác đáng để bài viết được hoàn thiện hơn. Tài liệu [1] //jl.ayme.pagesperso-orange.fr/docs/anton%0reim.pdf. [] //www.cut-the-knot.org/m/geometry/reim1.shtml. [3] //www.imo-official.org/. [4] Diễn đàn AoPS, topic //artofproblemsolving.com/community/c6_high_school_ olympiads. [5] //www.cut-the-knot.org/m/geometry/stathiskoutras.shtml. [6] //www.facebook.com/groups/perugeometrico1. 163

BÀI TOÁN HAY LỜI GIẢI ĐẸP Nguyễn Duy Liên [Trường THPT Chuyên Vĩnh Phúc] GIỚI THIỆU Giải được bài toán Số học hay và khó, ta đã cảm thích thú rồi. Nhưng nếu một bài toán Số học hay và khó mà giải được bằng nhiều cách mà từ đó ta có thể giải được, hay tạo ra một số bài toán cùng lớp bài toán đó thì niềm vui còn nhân lên nhiều lần. Bài viết này, tôi xin giới thiệu với các bạn 4 cách giải cho bài toán số về Số học khá hay và khó trong kỳ thi Olympic Toán học Hoa Kỳ 005 [USAMO 005]. Bài toán. Chứng minh rằng hệ phương trình [ x 6 C x 3 C x 3 y C y D 147 157 không có nghiệm nguyên x; y; z: x 3 C x 3 y C y C y C z 9 D 157 147 Lời giải 1. Ta sử dụng modulo 19 để chứng minh hệ phương trình này không có nghiệm nguyên. Ta có 15 D 8 19; 157 147.mod 19/: Bảng giá trị của 5 n.n N / theo.mod 19/: n 1 3 4 5 6 7 8 9 10 11 5 n 5 6 8 9 7 3 4 1 5 6 Do đó 147 157. 5/ 13 5 4.mod 19/ và 157 147 5 3 8.mod 19/: Viết lại hệ phương trình [ x 6 C x 3 C x 3 y C y D 147 157 x 3 C x 3 y C y C y C z 9 D 157 147 Cộng tương ứng theo vế ta được x 3 C y C 1 C z 9 D 147 157 C 157 147 C 1: Lại thấy 147 157 C 157 147 C 1 5.mod 19/; từ đó suy ra.x 3 C y C 1/ C z 9 5.mod 19/: [1] 164

Mặt khác theo định lý Femat s ta có z 18 0; 1.mod 19/ suy ra z 9 1; 0; 1.mod 19/ do đó để [1] xảy ra thì.x 3 C y C 1/ 4; 5; 6.mod 19/ vô lý, vì với mọi số nguyên a thì a 8; 3; ; 0; 1; 4; 5; 6; 7; 9.mod 19/: Vậy [1] không thể xảy ra hay là hệ phương trình không có nghiệm nguyên x; y và z: Lời giải. Ta sử dụng modulo 13: Cộng vế theo vế ta được x 3 C y C 1 C z 9 D 147 157 C 157 147 C 1: Theo định lý Femat s ta có a 1 1.mod 13/; 8a Z; a6 0.mod 13/; ta có 147 157 4 157 4 1.mod 13/; 157 147 1 147 1.mod 13/: Lại có 147 157 C 157 147 C 1 6.mod 13/; từ đó suy ra x 3 C y C 1 C z 9 6.mod 13/: [] Mặt khác với mọi số nguyên a thì a 3 5; 1; 0; 1; 5.mod 13/: Viết phương trình đầu tiên của hệ thành x 3 C 1 x 3 C y D 147 157 4.mod 13/; suy ra x 3 6 1.mod 13/ nên chỉ có thể x 3 0; 1; 5; 5.mod 13/ khi đó sẽ tương ứng với x 3 C y 4; ; 5; 1.mod 13/; suy ra x 3 C y C 1 1; 9; 10; 0.mod 13/: Vậy để xảy ra [] thì z 9 7; 10; 9; 6.mod 13/ vô lý, vì z 3 5; 1; 0; 1; 5.mod 13/; suy ra z 9 5; 1; 0; 1; 8.mod 13/: Vậy [] không thể xảy ra, ta có điều phải chứng minh. Lời giải 3. Ta sử dụng modulo 7; modulo 8 và định lý Mihailescu s. Viết lại hệ phương trình [ x 3 C y x 3 C 1 D 147 157.1/ x 3 C y.y C 1/ D 157 147 z 9./ Từ phương trình.1/ ta được x 3 C y ˇˇ147 157 : Ta xét ba trường hợp sau: x 3 C y D 1 W Từ.1/ suy ra x 3 C 1 D 147 157 ; cho nên 147 157 1 D x 3 _ 147 157 1 D. x/ 3 ; điều này vô lý vì x chẵn x 3. x/ 3 0.mod 8/ mà 147 157 1 D.mod 8/; 147 157 C 1 D 4.mod 8/: 7 ˇˇ x3 C y W Từ phương trình./ suy ra 7 ˇˇ157 147 z 9 nên z 9 157 147.mod 7/; suy ra z 3 3 49 3.mod 7/; vô lý vì z 3 1; 0; 1.mod 7/; 8z Z: 165

3 ˇˇ x3 C y W Từ.1/ suy ra 3 k x 3 C 1 D 147 157 ;k Z; từ đó dẫn đến x 3 C 1 D 147157 3 k ; cho nên 3 157 k 7 157 C 1 D. x/ 3 _ 3 157 k 7 157 1 D x 3 ; suy ra x chẵn, và ta có x 3. x/ 3 0.mod 8/: Phương trình 3 157 k 7 157 C1 D. x/ 3 không xảy ra 3 157 k 7 157 C1 ; 4.mod 8/: Phương trình 3 157 k 7 157 1 D x 3 để xảy ra thì 157 k D r; r N khi đó trở thành 3 r 7 157 1 D x 3 vô lý [định lý Mihailescu s 1 ]. Từ các trường hợp trên đều không xảy ra, ta có điều phải chứng minh. Lời giải 4. Ta sử dụng modulo 9; modulo 37 và Bổ đề LTE. Viết lại hệ phương trình [.x 3 C y/.x C 1/.x x C 1/ D 7 314 3 157.1/ Ta thấy rằng.x 3 C y/.y C 1/ D.157 49 z 3 /.157 98 C 157 49 z 3 C z 6 /./ x x C 1 D.x C 1/ 3.x C 1/ C 3 suy ra gcd x C 1; x x C 1 3.x C 1/ x x C 1 D x 3 C 1 D 7 a 3 b ;a;b N: x x khi jxj tương đương x x C 1 x khi jxj Nếu jxj 1 và x;y > 0 là x D 3; a D ; y D ; b D 3: 166

Nếu xc1 D 3 k ; thì.x C 1/ 3.x C 1/C3 D 3 7 m ; dẫn đến 3 k 3 kc1 C3 D 3 7 m ; cho nên 3 k 1 3 k C 1 D 7 m ; vì thế 3 k 3 k 1 1 D 7 m 1: Theo bổ đề LTE ta có v 3 3 k 3 k 1 1 D v 3.7 m 1/ ; suy ra k D C v 3.m/ C log 3 m: Vì thế 7 m 1 D 3 3 k k 1 1 3 Clog m 3 1Clog m 1 D 9m.3m 1/ : [3] Với m>thì [3] vô lý vì 7 m 1>9m.3m 1/ qua quy nạp. Với 0 m W ı Với m D ; từ phương trình 7 m 1 D 3 k.3 k 1 1/ ta thấy không tồn tại k Z: ı Với m D 1 từ phương trình 7 m 1 D 3 k.3 k 1 1/; suy ra k D 1; x D 4; thế vào.1/;./ ta có 3 155 ˇˇ.x 3 C y/ nên 3 155 ˇˇ 157 49 z 3 157 98 C 157 49 z 3 C z 6 : [4] Để [4] là đúng thì 9 chia hết ít nhất một trong hai thừa số vế phải của [4]. Nếu 9 j 157 49 z 3 ; thì z 3 157 49 4 49 4.mod 9/; vô lý. Còn nếu 9 j 157 98 C 157 49 z 3 C z 6 ; thì 157 98 C 157 49 z 3 C z 6 0.mod 9/: Tuy nhiên 157 98 C 157 49 z 3 C z 6 7 C 4z 3 C z 6 z 3 C C 3 6 0.mod 9/: ı Với m D 0 từ phương trình 7 m 1 D 3 k 3 k 1 1 ; suy ra k D 1; x D cũng giống như trường hợp khi m D 1 không tồn tại nghiệm nguyên của hệ. Vậy do đó không có trường hợp nào hệ có nghiệm nguyên. Bốn lời giải trên cho ta vẻ đẹp của mỗi con số, cách chọn modulo cho nó thật đắt giá như việc chọn modulo 19 ở lời giải 1; chọn modulo 13 ở lời giải : Sự kết hợp nhẹ nhàng và logic giữa các modulo đan xen vào đó là các phép biến đổi đại số, lập luận số học cho một lời giải gần gũi hơn đối với học trò qua các lời giải 3 và 4: Đặc biệt tôi ấn tượng với lời giải 3 đích cuối cùng của lời giải dẫn đến một bài toán mới định lý Mihailescu s, một định lý rất mới được hoàn thiện và công bố năm 004; tình yêu với những con số đã thôi thúc về tìm tòi cái mới lạ đã đưa tôi đên với định lý Mihailescu s. Tôi đã đọc, suy nghĩ, kiểm chứng nội dung định lý, nguồn gốc định lý và hiểu được cách chứng minh định lý này trong một đêm hè đấy. Ngoài cách giải trên các bạn cùng tôi tiếp tục đi tìm những lời giải mới, có thể cho những bài hay và khó trong vốn bài của bạn. Cứ như tôi thiết nghĩ đã mới khi trong đời ta đạt được các lời giải thật ngắn, đẹp và đắt. Nên ta vẫn trân trọng những lời giải của cá nhân tuy có dài dòng chút đỉnh với phương châm Cách giải này dài với bài này nhưng sẽ ngắn với bài khác. 167

HƯỚNG TỚI KỲ THI VMO 018-019 Lê Phúc Lữ [Lớp Cao học Khoa học tự nhiên TP.HCM] LỜI BAN BIÊN TẬP Kỳ thi chọn HSG cấp quốc gia [viết tắt là VMO] năm nay diễn ra vào các ngày 13 và 14 tháng 01/019. Trong bài viết này, tác giả sẽ đưa ra giới thiệu một số nội dung hướng đến rèn luyện chuẩn bị cho kỳ thi này, bao gồm: phân tích một số bài toán trong đề thi HSG các tỉnh, giới thiệu một số bài toán hay, đề thi thử và đáp án chi tiết. 1. Phân tích một số bài toán trong đề thi các tỉnh 018 1.1. Bài Cấp số cộng - Cấp số nhân đề Ninh Bình Ta xét bài toán sau trong đề thi của Ninh Bình Bài toán 1. Cho cấp số cộng [a n ] và cấp số nhân [b n ] đều có các số hạng dương. Biết rằng a 1 = b 1,a n = b n. Chứng minh rằng a k b k với mọi k =, 3,...,n 1. Bài toán này chính là đề chọn kỹ sư tài năng của ĐH Bách Khoa Hà Nội 01, cũng có xuất hiện trong đề thi HSG lớp 11 không chuyên của TP.HCM năm trước với trường hợp n =5. Bài toán đòi hỏi so sánh các số hạng "nằm giữa" của hai CSC, CSN hữu hạn có cùng số hạng đầu và cuối. Như thế ta cần tìm mối liên hệ giữa các số ở giữa đó và các số ở hai đầu? a n = a 1 +[n 1]d; a k = a 1 +[k 1]d hay a k = [n k]a 1 +[k 1]a n. n 1 Tương tự thì b k = n 1 b n k 1 b k 1 n. Áp dụng trực tiếp bất đẳng thức AM-GM cho n 1 số dương, ta có đpcm. Xem lại các bài toán trước đó thì trong đề thi HSG QG 01, đã có một bài đa thức tương tự, thực ra chỉ là giải phương trình bậc hai nhưng có liên quan đến CSC này; và năm đó, nhiều thí sinh đã lúng túng, không tìm được mối liên hệ như trên nên không xử lý được bài toán. Bài toán. [VMO 01] Cho hai CSC [a n ], [b n ] và xét dãy đa thức P k [x] =x + a k x + b k với 1 k n. Giả sử P 1 [x],p n [x] vô nghiệm. Chứng minh rằng các đa thức P k [x] với k n 1 đều vô nghiệm. 168

Tóm tắt lời giải bài VMO 01. Theo giả thiết thì a 1 4b 1 < 0,a n 4b n < 0. Ta cần chứng minh a k 4b k < 0 với 1

Chủ Đề